225
Міністерство освіти і науки України Сумський державний університет Медичний інститут КРОК -1 БІОХІМІЯ Збірник тестових завдань для студентів спеціальностей 222 «Медицина» та 228 «Педіатрія» денної форми навчання Суми Сумський державний університет 2020

КРОК-1 БІОХІМІЯ Збірник тестових завданьlib.sumdu.edu.ua/library/docs/rio/2020/m4695.pdf · 2 Крок-1. Біохімія Н: збірник тестових

  • Upload
    others

  • View
    18

  • Download
    1

Embed Size (px)

Citation preview

Page 1: КРОК-1 БІОХІМІЯ Збірник тестових завданьlib.sumdu.edu.ua/library/docs/rio/2020/m4695.pdf · 2 Крок-1. Біохімія Н: збірник тестових

Міністерство освіти і науки України

Сумський державний університет

Медичний інститут

КРОК-1

БІОХІМІЯ

Збірник тестових завдань для студентів спеціальностей

222 «Медицина» та 228 «Педіатрія»

денної форми навчання

Суми

Сумський державний університет

2020

Page 2: КРОК-1 БІОХІМІЯ Збірник тестових завданьlib.sumdu.edu.ua/library/docs/rio/2020/m4695.pdf · 2 Крок-1. Біохімія Н: збірник тестових

2

Крок-1. Біохімія : збірник тестових завдань / Н. М. Іншина,

Л. І. Гребеник, Л. О. Прімова. – Суми : Сумський державний

університет, 2020. – 225 с.

Кафедра біофізики, біохімії, фармакології

та біомолекулярної інженерії

Page 3: КРОК-1 БІОХІМІЯ Збірник тестових завданьlib.sumdu.edu.ua/library/docs/rio/2020/m4695.pdf · 2 Крок-1. Біохімія Н: збірник тестових

3

ЗМІСТ

С.

Частина I . . . . . . . . . . . . . . . . . . . . . . . . . . . . . . . . . . . . . . . . . . . . . 4

1.1. Білки . . . . . . . . . . . . . . . . . . . . . . . . . . . . . . . . . . . . . . . . . . . 4

1.2. Ферменти . . . . . . . . . . . . . . . . . . . . . . . . . . . . . . . . . . . . . . . 8

1.3. Біоенергетика . . . . . . . . . . . . . . . . . . . . . . . . . . . . . . . . . . . 14

1.4. Обмін вуглеводів . . . . . . . . . . . . . . . . . . . . . . . . . . . . . . . . 18

1.5. Обмін ліпідів . . . . . . . . . . . . . . . . . . . . . . . . . . . . . . . . . . . 36

1.6. Обмін амінокислот та білків. . . . . . . . . . . . . . . . . . . . . . . 44

1.7. Обмін нуклеотидів та нуклеїнових кислот . . . . . . . . . . . 56

1.8. Гормони . . . . . . . . . . . . . . . . . . . . . . . . . . . . . . . . . . . . . . . 66

1.9. Вітаміни . . . . . . . . . . . . . . . . . . . . . . . . . . . . . . . . . . . . . . . 83

1.10. Обмін порфіринів. Біохімія крові . . . . . . . . . . . . . . . . . . . 98

1.11. Біохімія травної системи. . . . . . . . . . . . . . . . . . . . . . . . . 107

1.12. Біохімія печінки та нирок . . . . . . . . . . . . . . . . . . . . . . . . 113

1.13. Біохімія мязів та сполучної тканини . . . . . . . . . . . . . . . 121

Частина II . . . . . . . . . . . . . . . . . . . . . . . . . . . . . . . . . . . . . . . . . . . 127

2.1. Крок-2007 . . . . . . . . . . . . . . . . . . . . . . . . . . . . . . . . . . . . . . . . 127

2.2. Крок-2008 . . . . . . . . . . . . . . . . . . . . . . . . . . . . . . . . . . . . . . . . 131

2.3. Крок-2009 . . . . . . . . . . . . . . . . . . . . . . . . . . . . . . . . . . . . . . . . 136

2.4. Крок-2010 . . . . . . . . . . . . . . . . . . . . . . . . . . . . . . . . . . . . . . . . 141

2.5. Крок-2011 . . . . . . . . . . . . . . . . . . . . . . . . . . . . . . . . . . . . . . . . 149

2.6. Крок-2012 . . . . . . . . . . . . . . . . . . . . . . . . . . . . . . . . . . . . . . . . 154

2.7. Крок-2013 . . . . . . . . . . . . . . . . . . . . . . . . . . . . . . . . . . . . . . . . 161

2.8. Крок-2014 . . . . . . . . . . . . . . . . . . . . . . . . . . . . . . . . . . . . . . . . 166

2.9. Крок-2015 . . . . . . . . . . . . . . . . . . . . . . . . . . . . . . . . . . . . . . . . 176

2.10. Крок-2016 . . . . . . . . . . . . . . . . . . . . . . . . . . . . . . . . . . . . . . . 186

2.11. Крок-2017 . . . . . . . . . . . . . . . . . . . . . . . . . . . . . . . . . . . . . . . 195

2.12. Крок-2018 . . . . . . . . . . . . . . . . . . . . . . . . . . . . . . . . . . . . . . . 206

2.13. Крок-2019 . . . . . . . . . . . . . . . . . . . . . . . . . . . . . . . . . . . . . . . 215

Page 4: КРОК-1 БІОХІМІЯ Збірник тестових завданьlib.sumdu.edu.ua/library/docs/rio/2020/m4695.pdf · 2 Крок-1. Біохімія Н: збірник тестових

4

ЧАСТИНА I

1.1. БІЛКИ

1. Жінка 30 років хворіє

близько року: біль у

суглобах, їх припухлість,

почервоніння шкіри над

ними. Попередній діагноз –

ревматоїдний артрит.

Одна з ймовірних причин

цього захворювання –

зміна у структурі білка

сполучної тканини:

А. Міозину.

В. Муцину.

С. Тропоніну.

D. Овоальбуміну.

Е. *Колагену.

2. Відомо, що молекула ко-

лагену містить амінокис-

лоти – гідроксипролін,

гідроксилізин. Яка з

перелічених речовин бере

участь у гідроксилюванні

проліну та лізину під час

синтезу ?

А. Аспарагінова кислота.

В. Фолієва кислота.

С. Пантотенова кислота.

D. Глутамінова кислота.

Е. *Аскорбінова кислота.

3. Лікар, перш ніж

призначити виснаженому

хворому білкове

парентеральне харчуван-

ня, направив до

лабораторії кров для

визначення електрофоре-

тичного спектра білків. На

яких фізико-хімічних

властивостях білків

ґрунтується цей метод ?

А. В’язкість.

В. Оптична активність.

С. Нездатність до

денатурації.

D. Гідрофільність і здатність

до набрякання.

Е. *Наявність заряду.

4. Хвора 36 років страждає

на колагеноз. Збільшення

вмісту якого метаболіту

найбільш імовірно буде

встановлено в сечі ?

А. Індикану.

В. Уробіліногену.

С. Креатиніну.

D. Сечовини.

Е. *Гідроксипроліну.

5. При пародонтозі

відбувається деструкція

білкових та полісахарид-

них компонентів сполучної

тканини. Який із

Page 5: КРОК-1 БІОХІМІЯ Збірник тестових завданьlib.sumdu.edu.ua/library/docs/rio/2020/m4695.pdf · 2 Крок-1. Біохімія Н: збірник тестових

5

наведених білків входить

до складу сполучної

тканини ?

А. Антитрипсин.

В. Альбумін.

С. Трансферин.

D. Церулоплазмін.

Е. *Колаген.

6. В організмі людини є

пептид, в утворенні якого

бере участь γ-карбо-

ксильна група глутамі-

нової кислоти. Як

називається цей пептид ?

А. Вазопресин.

В. Карнозин.

С. Ансерин.

D. Окситоцин.

Е. *Глутатіон.

7. Яка речовина надає

слині в’язкого, слизистого

характеру, виконує

захисну роль, запобігає

механічному пошкоджен-

ню слизової оболонки

ротової порожнини ?

А. Лізоцим.

В. Глюкоза.

С. Калікреїн.

D. Амілаза.

Е. *Муцин.

8. Гемоглобін дорослої

людини (HbA) – білок-

тетрамер, який

складається з двох α- та

двох β-поліпептидних

ланцюгів. Яку назву має

така структура цього

білка ?

А. Первинна.

В. Третинна.

С. Вторинна.

D. Пептидна.

Е. *Четвертинна.

9. Споживання забрудне-

них овочів і фруктів

упродовж тривалого часу

призвело до отруєння

пацієнта нітратами і

утворення в крові

похідного гемоглобіну:

A. Hb NHCOOH.

B. Hb СО.

C. Hb O2.

D. Hb CN.

E. *Hb-OH.

10. У хворого з тяжкою

формою гемолітичної

анемії еритроцити мають

форму серпа. У чому

виявляється молекулярна

причина виникнення

даного захворювання ?

A. Порушення синтезу гему.

Page 6: КРОК-1 БІОХІМІЯ Збірник тестових завданьlib.sumdu.edu.ua/library/docs/rio/2020/m4695.pdf · 2 Крок-1. Біохімія Н: збірник тестових

6

B. Порушення синтезу

порфіринів.

C. Порушення синтезу

ланцюга гемоглобіну.

D. Порушення синтезу

ланцюгів гемоглобіну.

E. *Заміна глутамінової

кислоти на валін у ланцюгу

гемоглобіну.

11. Робітник цеху з вироб-

ництва нітросполук

звернувся до лікаря із

скаргами на задишку та

швидку втомлюваність.

При обстеженні хворого

виявлено ціаноз нижніх

кінцівок. Яка причина

цього стану ?

A. Авітаміноз.

B. Гіповітаміноз.

C. Гіпервітаміноз.

D. Жирова інфільтрація

печінки.

E. *Посилене метгемогло-

біноутворення.

12. У сечі хворого

виявлено гідроксипролін і

пролін у підвищених

концентраціях. Пору-

шення метаболізму якого

білка можна припустити у

цього хворого:

A. Протромбіну.

B. Гемоглобіну.

C. Міозину.

D. Фібриногену.

Е. *Колагену.

13. Хворий прийнятий у

реанімаційне відділення з

підозрою на отруєння чад-

ним газом (монооксидом

карбону). Яка сполука

гемоглобіну буде вияв-

лена при спектральному

аналізі ?

A. Дезоксигемоглобін.

B. Карбгемоглобін.

C. Метгемоглобін.

D. Оксигемоглобін.

E. *Карбоксигемоглобін.

14. Хворому з печінковою

недостатністю проведено

дослідження електрофоре-

тичного спектра білків

сироватки крові. Які

фізико-хімічні властивості

білкових молекул лежать в

основі цього методу ?

A. Нездатність до діалізу.

B. Гідрофільність.

C. Здатність набрякати.

D. Оптична активність.

E. *Наявність заряду.

15. Поряд із нормальними

типами гемоглобіну в

Page 7: КРОК-1 БІОХІМІЯ Збірник тестових завданьlib.sumdu.edu.ua/library/docs/rio/2020/m4695.pdf · 2 Крок-1. Біохімія Н: збірник тестових

7

організмі дорослої людини

можуть бути патологічні.

Назвіть один із них:

A. HbO2.

B. Hb.

C. HbA1.

D. HbA2.

E. *HbS.

16. У хворого виявлена

серпоподібноклітинна ане-

мія. Заміна якої

амінокислоти в полі-

пептидному ланцюгу Hb

на валін призводить до

цього захворювання ?

A. Треоніну.

B. Аспарагінової кислоти.

C. Лейцину.

D. Аргініну.

E. *Глутамінової кислоти.

17. Катіонні глікопротеїни

є основними компо-

нентами слини. Які

амінокислоти обумов-

люють їх позитивний

заряд ?

A. Цистеїн, гліцин, пролін.

B. Аспартат, глутамат,

гліцин.

C. Аспартат, аргінін,

глутамат.

D. Глутамат, валін, лейцин.

E. *Лізин, аргінін, гістидин.

18. Назвіть, з яким білком

зв'язується гемоглобін для

транспорту в ретикуло-

ендотеліальну систему

печінки ?

A. Церулоплазміном.

B. Альбуміном.

C. Феритином.

D. Трансферином.

E. * Гаптоглобіном.

19. Після ремонту автомо-

біля в гаражному

приміщенні водій

потрапив у лікарню з

симптомами отруєння ви-

хлопними газами.

Концентрація якого

гемоглобіну в крові буде

підвищена ?

A. Глікозильованого

гемоглобіну.

B. Метгемоглобіну.

C. Карбгемоглобіну.

D. Оксигемоглобіну.

E. *Карбоксигемоглобіну.

20. Наявність білка в роз-

чині можна виявити за

допомогою кольорових

реакцій. Яка з реакцій

дасть негативний резуль-

тат при повному гідролізі

білка ?

A. Сакагучі.

Page 8: КРОК-1 БІОХІМІЯ Збірник тестових завданьlib.sumdu.edu.ua/library/docs/rio/2020/m4695.pdf · 2 Крок-1. Біохімія Н: збірник тестових

8

B. Нінгідринова.

C. Ксантопротеїнова.

D. Фоля.

E. *Біуретова.

21. Під час дослідження

первинної структури

молекули гемоглобіну

виявлено заміну

глутамінової кислоти на

валін. Для якої спадкової

патології це характерно ?

А. Таласемії.

В. Гемоглобінозу.

С. Хвороби Мінковського-

Шоффара.

D. Фавізму.

Е. *Серпоподібноклітинної

анемії.

1.2. ФЕРМЕНТИ

1. Захисна функція слини

зумовлена декількома ме-

ханізмами, у тому числі

наявністю ферменту, який

має бактерицидну дію,

зумовлює лізис

полісахаридного комплек-

су оболонки стафілококів,

стрептококів. Який це

фермент ?

А. Колагеназа.

В. α-Амілаза.

С. Оліго-1,6-глюкозидаза.

D. β-Глюкуронідаза.

Е. *Лізоцим.

2. При захворюваннях під-

шлункової залози пору-

шуються утворення та

секреція трипсину. Назвіть

речовини, гідроліз яких

при цьому зазнає змін ?

А. Фосфоліпіди.

В. Ліпіди.

С. Вуглеводи.

D. Нуклеїнові кислоти.

Е. *Білки.

3. У крові хворого – підви-

щення активності ЛДГ4,

ЛДГ5, аланінамінотранс-

ферази, карбамоїлорнітин-

трансферази. В якому

органі можна передбачити

розвиток патологічного

процесу ?

А. У скелетних м’язах.

В. У серцевому м’язі

(можливий інфаркт

міокарда).

С. У сполучній тканині.

D. У нирках.

Е. *У печінці (можливий

гепатит).

4. У хворого виявлено під-

вищення активності ЛДГ1,

Page 9: КРОК-1 БІОХІМІЯ Збірник тестових завданьlib.sumdu.edu.ua/library/docs/rio/2020/m4695.pdf · 2 Крок-1. Біохімія Н: збірник тестових

9

ЛДГ2, аспартатаміно-

трансферази, креатин-

фосфокінази. В якому

органі імовірний розвиток

патологічного процесу ?

А. У печінці та нирках.

В. У скелетних м’язах (дис-

трофія, атрофія).

С. У нирках та надниркових

залозах.

D. У сполучній тканині.

Е. *У серцевому м’язі

(інфаркт міокарда).

5. При патологічних про-

цесах, що супроводжую-

ться гіпоксією, відбуваю-

ться неповне відновлення

молекули кисню в

дихальному ланцюзі і

накопичення пероксиду

гідрогену. Назвіть

фермент, який забезпечує

його руйнування:

А. Аконітаза.

В. Цитохромоксидаза.

С. Сукцинатдегідрогеназа.

D. α-Кетоглутаратдегідроге-

наза.

Е. *Каталаза.

6. Універсальною біологіч-

ною системою окиснення

неполярних сполук (ба-

гатьох лікарських засобів,

токсичних сполук, стероїд-

них гормонів, холестерину)

є мікросомальне окиснен-

ня. Який цитохром

входить до складу

оксигеназного ланцюга

мікросом ?

А. Цитохром с.

В. Цитохром а3.

С. Цитохром b.

D. Цитохром а.

Е. *Цитохром Р450.

7. У відділення реанімації

прийнятий хворий 47

років з діагнозом інфаркт

міокарда. Яка з фракцій

лактатдегідрогенази (ЛДГ)

буде переважати в

сироватці крові хворого

упродовж перших двох

діб ?

А. ЛДГ4.

В. ЛДГ2.

С. ЛДГ3.

D. ЛДГ5.

Е. *ЛДГ1.

8. У відділення інтенсив-

ної терапії доставлено жін-

ку 50 років з діагнозом

інфаркт міокарда. Ак-

тивність якого ферменту

буде найбільш підвищена

упродовж перших двох діб:

Page 10: КРОК-1 БІОХІМІЯ Збірник тестових завданьlib.sumdu.edu.ua/library/docs/rio/2020/m4695.pdf · 2 Крок-1. Біохімія Н: збірник тестових

10

А. Аланінамінопептидази.

В. Аланінамінотрансферази.

С. ЛДГ5.

D. ЛДГ4.

Е.*Аспартатамінотрансфе-

рази.

9. В організмі людини

хімотрипсин секретується

підшлунковою залозою і в

порожнині кишечника

зазнає обмеженого

протеолізу з перетворен-

ням на активний

хімотрипсин під дією:

А. Амінопептидази.

В. Ентерокінази.

С. Пепсину.

D. Карбоксипептидази.

Е. *Трипсину.

10. У хворого через 12 год

після гострого нападу

загруднинного болю в

сироватці крові різко

підвищилася активність

аспартатамінотрансферази

Назвіть патологію, для

якої характерні ці зміни:

А. Колагеноз.

В. Вірусний гепатит.

С. Нецукровий діабет.

D. Цукровий діабет.

Е. *Інфаркт міокарда.

11. У хворого 27 років

виявлено патологічні

зміни печінки і головного

мозку. У плазмі крові –

різке зниження, а в сечі –

підвищення вмісту міді.

Встановлено діагноз:

хвороба Вільсона. Актив-

ність якого ферменту в

сироватці крові необхідно

дослідити для підтверд-

ження діагнозу:

А. Ксантиноксидази.

В. Карбоангідрази.

С. Алкогольдегідрогенази.

D. Лейцинамінопептидази.

Е. *Церулоплазміну.

12. Діагностичним тестом

при гострих панкреатитах

є визначення в сечі

активності ферменту:

А. Аланінамінопептидази.

В. Лактатдегідрогенази.

С. Креатинкінази.

D. Альдолази.

Е. *Амілази.

13. Назвіть фермент,

визначення активності

якого в крові є найбільш

інформативним у перші

години після інфаркту

міокарда:

А. Глутаматдегідрогеназа.

Page 11: КРОК-1 БІОХІМІЯ Збірник тестових завданьlib.sumdu.edu.ua/library/docs/rio/2020/m4695.pdf · 2 Крок-1. Біохімія Н: збірник тестових

11

В. Аспартатамінотрансфе-

раза.

С. Аланінамінотрансфераза.

D. Лактатдегідрогеназа.

Е. *Креатинфосфокіназа.

14. У новонародженої

дитини в шлунку

відбувається звурджуван-

ня молока, тобто

перетворення розчинних

білків молока – казеїнів -

на нерозчинні –

параказеїни – за участі

іонів кальцію і ферменту.

Який фермент бере участь

у цьому процесі ?

А. Секретин.

В. Пепсин.

С. Гастрин.

D. Ліпаза.

Е. *Реннін.

15. У хворого гострий

панкреатит. Які препарати

повинен призначити лікар,

щоб уникнути аутолізу

підшлункової залози ?

А. Активатори протеаз.

В. Амілазу.

С. Трипсин.

D. Хімотрипсин.

Е. *Інгібітори протеаз.

16. При гепатиті, інфаркті

міокарда в плазмі крові у

хворих різко зростає

активність аланін- і

аспартатамінотрансфераз.

Які причини підвищення

активності цих ферментів

у крові ?

А. Недостатність

піридоксину.

В. Активація ферментів

гормонами.

С. Збільшення швидкості

розпаду амінокислот у

тканинах.

D. Зростання швидкості син-

тезу амінокислот у

тканинах.

Е. *Пошкодження мембран

клітин і вихід ферментів у

кров.

17. У слині міститься фер-

мент, здатний руйнувати

α-1,4-глікозидні зв’язки в

молекулі крохмалю.

Назвіть цей фермент:

А. Лізоцим.

В. Фосфатаза.

С. Фруктофуранозидаза.

D. β-Галактозидаза.

Е. *α-Амілаза.

18. 60-річний чоловік

звернувся до лікаря після

Page 12: КРОК-1 БІОХІМІЯ Збірник тестових завданьlib.sumdu.edu.ua/library/docs/rio/2020/m4695.pdf · 2 Крок-1. Біохімія Н: збірник тестових

12

появи болю в грудній

клітці. У сироватці крові

виявлено значне підви-

щення активності фермен-

тів: креатинфосфокінази

та її МВ-ізоформи,

аспартатамінотрансферази

Про розвиток патологіч-

ного процесу в якій

тканині свідчать ці зміни ?

A. У гладеньких м’язах.

B. У тканині легень.

C. У скелетних м’язах.

D. У тканині печінки.

E. *У серцевому м’язі.

19. При вивченні власти-

востей ферменту до

системи фермент-субстрат

було додано невідому

речовину. В результаті

константа Міхаеліса

збільшилася в 2 рази. Яке

явище мало місце ?

A. Незворотне інгібування.

B. Неконкурентне інгібу-

вання.

C. Безконкурентне інгібу-

вання.

D. Алостерична активація.

E. *Конкурентне

інгібування.

20. Хворому під час курсу

хіміотерапії пухлини

призначено структурний

аналог глутаміну –

антибіотик азасерин –

сильний інгібітор синтезу

пуринових нуклеотидів.

Який тип інгібування

характерний для цього

препарату ?

A. Алостеричний.

B. Незворотний.

C. Неконкурентний.

D. Безконкурентний.

E. *Конкурентний.

21. У сироватці крові

хворого виявлено високу

активність ізоферменту

ЛДГ1. Патологічний

процес в якому органі має

хворий ?

A. Нирках.

B. Печінці.

C. Скелетних м'язах.

D. Підшлунковій залозі.

E. *Серці.

22. Хворого доставила в

стаціонар швидка

допомога із попереднім

діагнозом – гострий

панкреатит. Активність

якого ферменту в крові та

сечі необхідно визначити

для підтвердження цього

діагнозу ?

Page 13: КРОК-1 БІОХІМІЯ Збірник тестових завданьlib.sumdu.edu.ua/library/docs/rio/2020/m4695.pdf · 2 Крок-1. Біохімія Н: збірник тестових

13

A. Холінестерази.

B. АлАТ.

C. АсАТ.

D. Лактатдегідрогенази.

E. *α-Амілази.

23. У діагностиці інфаркту

міокарда важлива роль

належить методам

ензимодіагностики. Визна-

чення вмісту в крові якого

ферменту є вирішальним у

перші 2-4 год після

інфаркту ?

А. Альдолази.

В. Ліпопротеїнліпази.

С. Аланінамінотрансферази.

D. Ацетилхолінестерази.

Е. *Креатинфосфокінази.

24. Один із методів

лікування при отруєнні

метанолом полягає в тому,

що хворому призначають

етанол у кількості, що

спричиняє у здорової

людини інтоксикацію.

Чому таке лікування є

ефективним ?

А. Етанол розщеплюється

швидше, ніж метанол, у

результаті чого

утворюються менш токсичні

сполуки.

В. Етанол інгібує

алкогольдегідрогеназу.

С. Етанол активує

алкогольдегідрогеназу.

D. Етанол зв’язує алостерич-

ний центр алкоголь-

дегідрогенази, яка при

цьому інактивується.

Е. *Етанол конкурує з мета-

нолом за активний центр

алкогольдегідрогенази.

25. При дослідженні сиро-

ватки крові у хворого

виявлено підвищення

активності лактатдегідро-

генази. Які зміни в

організмі на клітинному

рівні призводять до

подібних порушень ?

А. Інгібування ферментних

систем.

В. Порушення генетичного

апарату.

С. Порушення міжклітинних

взаємодій.

D. Порушення процесів

енергозабезпечення.

Е. *Пошкодження плазмати-

чних мембран.

Page 14: КРОК-1 БІОХІМІЯ Збірник тестових завданьlib.sumdu.edu.ua/library/docs/rio/2020/m4695.pdf · 2 Крок-1. Біохімія Н: збірник тестових

14

1.3. БІОЕНЕРГЕТИКА

1. Центральним проміж-

ним продуктом усіх

обмінів (білків, ліпідів,

вуглеводів) є:

А. Сукциніл-КоА.

В. Цитрат.

С. Оксалоацетат.

D. Лактат.

Е. *Ацетил-КоА.

2. Судмедексперт під час

розтину тіла 20-річної

дівчини встановив, що

смерть настала унаслідок

отруєння ціанідами.

Активність якого

ферменту найбільшою

мірою гальмується

ціанідами ?

А. Малатдегідрогенази.

В. Карбамоїлфосфатсинте-

тази.

С. Гемсинтетази.

D. Аспартатамінотрансфе-

рази.

Е. *Цитохромоксидази.

3. Підвищену стійкість

“моржів” до холодної води

пояснюють активацією

синтезу гормонів, що

посилюють процеси окис-

нення і утворення тепла в

мітохондріях шляхом

роз’єднання біологічного

окиснення та окисного

фосфорилювання. Які це

гормони ?

А. Глюкагон.

В. Адреналін і норадреналін.

С. Кортикостероїди.

D. Інсулін.

Е. *Йодовмісні гормони щи-

товидної залози.

4. Посилення перекисного

окиснення ліпідів і

біополімерів є одним із

основних механізмів

пошкодження структури

та функції клітинних

мембран і загибелі

клітини. Причиною цього

є:

А. Гіпервітаміноз В12.

В. Гіповітаміноз В1.

С. Гіпервітаміноз В1.

D. Гіповітаміноз В12.

Е. *Посилене утворення

вільних радикалів кисню та

пригнічення антиоксидант-

них систем.

5. При тиреотоксикозі під-

вищується продукція тире-

оїдних гормонів Т3 і Т4,

спостерігаються схуднен-

ня, тахікардія, психічна

Page 15: КРОК-1 БІОХІМІЯ Збірник тестових завданьlib.sumdu.edu.ua/library/docs/rio/2020/m4695.pdf · 2 Крок-1. Біохімія Н: збірник тестових

15

збудливість та ін. Як саме

впливають тиреоїдні гор-

мони на енергетичний

обмін у мітохондріях

клітин ?

А. Блокують дихальний

ланцюг.

В. Активують субстратне

фосфорилювання.

С. Блокують субстратне

фосфорилювання.

D. Активують окисне

фосфорилювання.

Е. .*Роз’єднують біологічне

окиснення та окиснювальне

фосфорилювання.

6. При отруєнні ціанідами

настає миттєва смерть. У

чому полягає механізм дії

ціанідів на молекулярному

рівні ?

А. Інгібують цитохром b.

В. Зв’язують субстрати

ЦТК.

С. Блокують сукцинатде-

гідрогеназу.

D. Інактивують кисень.

Е. *Інгібують цитохром-

оксидазу.

7. Біоенергетика мозку

значно залежить від

постачання кисню. Який

субстрат окиснення має

найбільше значення для

забезпечення мозку

енергією ?

А. Жирні кислоти.

В. Фосфоенолпіруват

С. Кетонові тіла.

D. Гліцерол-3-фосфат.

Е. *Глюкоза.

8. Як називається процес

синтезу АТФ, що

пов'язаний з реакціями

окиснення за участі

системи дихальних

ферментів мітохондрій ?

A. Відновлювальне

фосфорилювання.

B. Cубстратне фосфорилю-

вання.

C. Вільне окиснення.

D. Фотосинтетичне

фосфорилювання.

E. *Окиснювальне

фосфорилювання.

9. При нанесенні стомато-

логом пероксиду гідрогену

на слизову оболонку

порожнини рота з'явилася

інтенсивна піна. Наявність

якого ферменту може

спричинити такий ефект ?

A. Метгемоглобінредуктази.

B. Холінестерази.

C. Ацетилтрансферази.

Page 16: КРОК-1 БІОХІМІЯ Збірник тестових завданьlib.sumdu.edu.ua/library/docs/rio/2020/m4695.pdf · 2 Крок-1. Біохімія Н: збірник тестових

16

D. Глюкозо-6-фосфат-

дегідрогенази

E. *Каталази.

10. Ціаніди є надзвичайно

потужними клітинними

отрутами, які при

надходженні в організм

людини можуть спричи-

нити смерть. Блокування

якого ферменту тканин-

ного дихання лежить в

основі їх дії ?

A. Глюкозо-6-фосфат-

дегідрогенази.

B. Ферохелатази.

C. Каталази.

D. Гемоглобінредуктази.

E. *Цитохромоксидази.

11. При підвищенні

функції щитовидної залози

спостерігається втрата

ваги та підвищення

температури тіла. Який

біохімічний процес при

цьому активується ?

A. Стероїдогенез.

B. Анаболізм.

C. Неоглюкогенез.

D. Ліпогенез.

E. *Катаболізм.

12. Біологічне окиснення

та знешкодження ксено-

біотиків відбувається за

рахунок гемовмісних

ферментів. Який метал є

обов`язковою складовою

цих ферментів ?

A. Mn.

B. Zn.

C. Co.

D. Mg.

E. *Fe.

13. У процесі метаболізму в

організмі людини

утворюються активні

форми кисню, у тому числі

супероксидний аніон-

радикал О-2. Цей аніон

інактивується за

допомогою ферменту:

A. Глутатіонредуктази.

B. Каталази.

C. Пероксидази.

D. Глутатіонпероксидази.

E. * Супероксиддисмутази.

14. Знешкодження хворо-

ботворних бактерій та

розщеплення чужорідних

компонентів у лейкоцитах

здійснюється за типом

реакції окиснення:

A. Анаеробного.

B. Оксидазного.

C. Оксигеназного.

D. Аеробного.

Page 17: КРОК-1 БІОХІМІЯ Збірник тестових завданьlib.sumdu.edu.ua/library/docs/rio/2020/m4695.pdf · 2 Крок-1. Біохімія Н: збірник тестових

17

E. *Пероксидазного.

15. Для нормального мета-

болізму клітинам необхідні

макроергічні сполуки. Яка

сполука належить до

макроергів ?

A. Аденозинмонофосфат.

B. Креатин.

C. Креатинін.

D. Глюкозо-6-фосфат.

E. *Креатинфосфат.

16. Пацієнт звернувся зі

скаргами на напади утруд-

неного дихання, запаморо-

чення. З’ясувалося, що він

працює на хімічному під-

приємстві з виробництва

синильної кислоти. З пору-

шенням функції якого

ферменту можуть бути

пов’язані зазначені зміни ?

A. Піруватдегідрогенази.

B. Лактатдегідрогенази.

C. Сукцинатдегідрогенази.

D. Каталази.

E. * Цитохромоксидази.

17. При окисненні органіч-

них речовин в організмі

утворюється ендогенна

вода, яка в умовах “сухого

голодування” частково

компенсує водний дефіцит.

Яка із зазначених речовин

при окисненні дає

найбільшу кількість води

(на одиницю маси цієї

речовини) ?

А. Глюкоза.

В. Білки.

С. Глікоген.

D. Гліцерин.

Е. *Жири.

18. Ті організми, які в

процесі еволюції не

створили систему захисту

від Н2О2, можуть існувати

лише в анаеробних умовах.

Які з наведених ферментів

можуть руйнувати гідро-

гену пероксид ?

А. Оксигенази та

гідроксилази.

В. Флавінзалежні оксидази.

С. Цитохромоксидаза, цито-

хром b5.

D. Оксигеназа та каталаза.

Е. *Пероксидаза та каталаза.

19. Жінка 38 років

скаржиться на підвищену

пітливість, серцебиття,

підвищення температури

тіла у вечірні години.

Основний обмін

збільшений на 60 %. Лікар

встановив діагноз -

Page 18: КРОК-1 БІОХІМІЯ Збірник тестових завданьlib.sumdu.edu.ua/library/docs/rio/2020/m4695.pdf · 2 Крок-1. Біохімія Н: збірник тестових

18

тиреотоксикоз. Які

властивості тироксину

приводять до посилення

теплопродукції ?

А. Сприяє накопиченню

ацетил-КоА.

В. Зменшує дезамінування

амінокислот.

С. Зменшує -окиснення

жирних кислот.

D. Підвищує спряження

окиснення та

фосфорилювання.

Е. *Роз’єднує біологічне

окиснення та окисне

фосфорилювання.

20. Ціаністий калій – отру-

та, яка призводить до

миттєвої загибелі орга-

нізму. На який фермент в

мітохондріях діє ціаністий

калій ?

А. Цитохром Р450.

В. Флавінові ферменти.

С. Цитохром b.

D. НАД+-залежні дегідро-

генази.

Е. *Цитохромоксидаза [аа3].

1.4. ОБМІН

ВУГЛЕВОДІВ

1. Хвора 46 років скар-

житься на сухість у роті,

спрагу, часте сечовипус-

кання, загальну слабість.

При біохімічному дослід-

женні в крові виявлено

гіперглікемію, гіперкето-

немію, у сечі – глюкозу,

кетонові тіла. На електро-

кардіограмі – дифузні

зміни в міокарді. У хворої

імовірно:

А. Ішемічна хвороба серця.

В. Аліментарна гіпер-

глікемія.

С. Гострий панкреатит.

D. Нецукровий діабет.

Е. *Цукровий діабет.

2. У 3-річної дитини із під-

вищеною температурою

тіла після приймання

аспірину спостерігається

посилений гемоліз

еритроцитів. Спадковий

дефіцит якого ферменту

зумовлює гемолітичну

анемію ?

А. Гліцеролфосфатдегідро-

генази.

В. Глюкозо-6-фосфатази.

С. Глікогенфосфорилази.

Page 19: КРОК-1 БІОХІМІЯ Збірник тестових завданьlib.sumdu.edu.ua/library/docs/rio/2020/m4695.pdf · 2 Крок-1. Біохімія Н: збірник тестових

19

D. γ-Глутамінілтрансферази.

Е. *Глюкозо-6-фосфат-

дегідрогенази.

3. У регуляції активності

ферментів важливе місце

належить їх постсинтетич-

ній ковалентній моди-

фікації. Яким із зазначених

механізмів здійснюється

регуляція активності

глікогенфосфорилази і

глікогенсинтетази ?

А. АДФ-рибозилюванням.

В. Метилюванням.

С. Аденілуванням.

D. Обмеженим протеолізом.

Е. *Фосфорилюванням –

дефосфорилюванням.

4. Через деякий час після

інтенсивного фізичного

тренування у спортсмена

активується глюконео-

генез, основним субстра-

том якого є:

А. Серин.

В. Аспарагінова кислота.

С. Глутамінова кислота.

D. α-Кетоглутарат.

Е. *Лактат.

5. Концентрація глюкози в

плазмі крові здорової

людини варіює в таких

межах:

А. 2-4 ммоль/л.

В. 1-2 ммоль/л.

С. 10-25 ммоль/л.

D. 6-9,5 ммоль/л.

Е. *3,3-5,5 ммоль/л.

6. У хворого 34 років

знижена витривалість до

фізичних навантажень, у

той час як у скелетних

м’язах вміст глікогену

підвищений. Зниженням

активності якого ферменту

це пояснюється ?

А. Фосфофруктокінази.

В. Глюкозо-6-фосфат-

дегідрогенази.

С. Глюкозо-6-фосфатази.

D. Глікогенсинтази.

Е. *Глікогенфосфорилази.

7. У дитини з точковою

мутацією генів виявлено

відсутність глюкозо-6-фос-

фатази, гіпоглікемію і

гепатомегалію. Визначте

вид патології, для якої

характерні ці ознаки:

А. Хвороба Мак-Ардля.

В. Хвороба Корі.

С. Хвороба Аддісона.

D. Хвороба Паркінсона.

Е. *Хвороба Гірке.

Page 20: КРОК-1 БІОХІМІЯ Збірник тестових завданьlib.sumdu.edu.ua/library/docs/rio/2020/m4695.pdf · 2 Крок-1. Біохімія Н: збірник тестових

20

8. У хворого, який прохо-

дить курс лікувального го-

лодування, нормальний рі-

вень глюкози у крові під-

тримується за рахунок

глюконеогенезу. З якої

амінокислоти у печінці

людини найбільш активно

синтезується глюкоза ?

А. Глутамінова кислота.

В. Валін.

С. Лейцин.

D. Лізин.

Е. *Аланін.

9. У новонародженої дити-

ни після годування

молоком спостерігалися

диспептичні явища

(пронос, блювання). При

годуванні розчином

глюкози ці явища зникали.

Назвіть фермент, що бере

участь у перетравленні

вуглеводів, недостатня ак-

тивність якого призводить

до зазначених розладів:

А. Сахараза.

В. Амілаза.

С. Мальтаза.

D. Ізомальтаза.

Е. *Лактаза.

10. Під час бігу на короткі

дистанції у нетренованих

людей спостерігається м’я-

зова крепатура унаслідок

накопичення лактату. З

посиленням якого

біохімічного процесу це

може бути пов’язано ?

А. Глюконеогенезу.

В. Глікогенезу.

С. Пентозофосфатного

шляху.

D. Ліпогенезу.

Е. *Гліколізу.

11. У хлопчика 2 років

спостерігається збільшен-

ня розмірів печінки та

селезінки, катаракта. У

крові - підвищена

концентрація цукру, однак

тест толерантності до

глюкози в нормі. Спадкове

порушення обміну якої

речовини є причиною

цього стану ?

А. Глюкози.

В. Фруктози.

С. Сахарози.

D. Мальтози.

Е. *Галактози.

12. У лікарню прийнята

робітниця хімічного

підприємства з ознаками

отруєння. У волоссі цієї

жінки виявлено підвищену

Page 21: КРОК-1 БІОХІМІЯ Збірник тестових завданьlib.sumdu.edu.ua/library/docs/rio/2020/m4695.pdf · 2 Крок-1. Біохімія Н: збірник тестових

21

концентрацію арсенату,

який звязує ліпоєву

кислоту. Порушення якого

процесу є найімовірнішою

причиною отруєння ?

А. Знешкодження суперок-

сидних іонів.

В. Мікросомального

окиснення.

С. Відновлення мет-

гемоглобіну.

D. Відновлення органічних

оксидів.

Е. *Окисного декарбокси-

лювання піровиноградної

кислоти.

13. У хворого 57 років

який страждає на

цукровий діабет, розви-

нувся кетоацидоз. Біохі-

мічною основою цього

стану є зниження ступеня

утилізації ацетил-КоА

унаслідок дефіциту:

А. 2-Оксоглутарату.

В. Сукцинату.

С. Глутамату.

D. Аспартату.

Е. *Оксалоацетату.

14. У хворої 38 років після

приймання аспірину і

сульфаніламідних препа-

ратів спостерігається

посилений гемоліз

еритроцитів, спричи-

нений недостатністю

глюкозо-6-фосфатдегідро-

генази. Порушенням

утворення якого коферме-

нту зумовлена ця

патологія ?

А. Убіхінону.

В. ФАДН2.

С. Піридоксальфосфату.

D. ФМНН2.

Е. *НАДФН.

15. При глікогенозі –

хворобі Гірке –

порушується перетво-

рення глюкозо-6-фосфату

на глюкозу, що призводить

до накопичення глікогену

в печінці. Дефіцит якого

ферменту є причиною

захворювання ?

А. Фосфорилази.

В. Глікогенсинтетази.

С. Альдолази.

D. Гексокінази.

Е. *Глюкозо-6-фосфатази.

16. Дитина квола,

апатична. Печінка

збільшена і при її біопсії

виявлений надлишок

глікогену. Концентрація

глюкози в крові нижча за

Page 22: КРОК-1 БІОХІМІЯ Збірник тестових завданьlib.sumdu.edu.ua/library/docs/rio/2020/m4695.pdf · 2 Крок-1. Біохімія Н: збірник тестових

22

норму. У чому причина

зниженої концентрації

глюкози в крові ?

А. Дефіцит гена, який відпо-

відає за синтез глюкозо-1-

фосфатуридинтрансферази.

В. Знижена (відсутня) актив-

ність гексокінази.

С. Підвищена активність

глікогенсинтетази.

D. Знижена (відсутня) актив-

ність глюкозо-6-фосфатази.

Е. *Знижена (відсутня)

активність глікогенфосфо-

рилази в печінці.

17. Авідин – сильний

специфічний інгібітор

біотинових ферментів. Яка

з реакцій буде блокуватися

при додаванні авідину до

клітинного гомогенату ?

А. Глюкоза → рибозо-5-

фосфат.

В. Глюкоза → піруват.

С. Оксалоацетат → глюкоза.

D. Лактат → піруват.

Е. *Піруват → оксалоацетат.

18. Чоловік 38 років прохо-

дить курс лікування у

стаціонарі з приводу ши-

зофренії. Вихідний вміст у

крові глюкози, кетонових

тіл, сечовини – у нормі.

Шокова терапія регуляр-

ними ін’єкціями інсуліну

призвела до розвитку

інсулінової коми, після

чого стан хворого

поліпшився. Що було най-

більш імовірною причи-

ною інсулінової коми ?

А. Глюкозурія.

В. Дегідратація тканин.

С. Метаболічний ацидоз.

D. Кетонемія.

Е. *Гіпоглікемія.

19. У дівчинки 7 років -

виражені ознаки анемії.

Лабораторно встановлено

дефіцит піруваткінази в

еритроцитах. Порушення

якого процесу відіграє

головну роль у розвитку

анемії ?

А. Дезамінування аміно-

кислот.

В. Окисного

фосфорилювання.

С. Тканинного дихання.

D. Розкладання пероксидів.

Е. *Анаеробного гліколізу.

20. У реанімаційне відді-

лення було доставлено

немовля із такими

ознаками: блювання,

пронос, порушення росту і

Page 23: КРОК-1 БІОХІМІЯ Збірник тестових завданьlib.sumdu.edu.ua/library/docs/rio/2020/m4695.pdf · 2 Крок-1. Біохімія Н: збірник тестових

23

розвитку, катаракта, ро-

зумова відсталість. Було

встановлено діагноз -

галактоземія. Дефіцит

якого ферменту спостері-

гається ?

А. Глюкозо-6-фосфат-

дегідрогенази.

В. Глюкокінази.

С. УДФ-глюкозо-4-

епімерази.

D. УДФ-глюкозо-

пірофосфорилази.

Е. *Галактозо-1-фосфат-

уридилтрансферази.

21. У людей, які тривалий

час перебували у стані

гіподинамії, після фізич-

ного навантаження з’яв-

ляється інтенсивний біль у

м’язах. Яка найбільш

імовірна причина больо-

вого синдрому ?

А. Зменшення вмісту ліпідів

у м’язах.

В. Посилений розпад

м’язових білків.

С. Накопичення креатиніну

у м’язах.

D. Підвищення вмісту АДФ

у м’язах.

Е. *Накопичення у м’язах

молочної кислоти.

22. У жінки 45 років

відсутні симптоми

цукрового діабету, але

визначається натще

підвищений вміст глюкози

у крові (7,5 ммоль/л). Який

тест необхідно провести ?

А. Визначення концентрації

С-пептиду.

В. Визначення рівня кето-

нових тіл у сечі.

С. Визначення вмісту

залишкового азоту у крові.

D. Визначення рівня гліко-

зильованого гемоглобіну.

Е. *Визначення толерант-

ності до глюкози в крові

натще.

23. У крові пацієнта вміст

глюкози натще становив

5,65 ммоль/л, через 1 год

після цукрового

навантаження –8,55

ммоль/л, а через 2 год –

4,95 ммоль/л. Такі

показники характерні для:

А. Інсулінозалежного цукро-

вого діабету.

В. Прихованого цукрового

діабету.

С. Гіпотиреозу.

D. Інсулінонезалежного цук-

рового діабету.

Е. Тиреотоксикозу.

Page 24: КРОК-1 БІОХІМІЯ Збірник тестових завданьlib.sumdu.edu.ua/library/docs/rio/2020/m4695.pdf · 2 Крок-1. Біохімія Н: збірник тестових

24

Е. *Стану здоров’я.

24. У хворого, виснаженого

голодуванням, у печінці та

нирках посилюється

процес:

А. Синтезу білірубіну.

В. Синтезу сечовини.

С. Синтезу сечової кислоти.

D. Утворення гіпурової

кислоти.

Е. *Глюконеогенезу.

25. Еритроцит для своєї

життєдіяльності потребує

енергію у вигляді АТФ.

Який процес забезпечує цю

клітину необхідною

кількістю АТФ ?

А. Цикл трикарбонових

кислот.

В. Аеробне окиснення

глюкози.

С. Пентозофосфатний цикл.

D. β-Окиснення жирних

кислот.

Е. *Анаеробний гліколіз.

26. При дефіциті тіаміну –

вітаміну В1 – виникає

хвороба бері-бері (полі-

неврит) та порушується

вуглеводний обмін. Який

метаболіт при цьому

накопичується в крові ?

А. Лактат.

В. Малат.

С. Сукцинат.

D. Цитрат.

Е. *Піруват.

27. Анаеробне розщеплен-

ня глюкози до молочної

кислоти регулюється

відповідними ферментами.

Який фермент є головним

регулятором цього

процесу ?

А. Лактатдегідрогеназа.

В. Глюкозо-6-фосфат-

ізомераза.

С. Альдолаза.

D. Енолаза.

Е. *Фосфофруктокіназа.

28. У крові пацієнта –

високий вміст галактози, а

концентрація глюкози –

знижена. Спостерігаються

катаракта, жирове

переродження печінки.

Про яке захворювання це

свідчить ?

А. Цукровий діабет.

В. Фруктоземію.

С. Лактоземію.

D. Стероїдний діабет.

Е. *Галактоземію.

Page 25: КРОК-1 БІОХІМІЯ Збірник тестових завданьlib.sumdu.edu.ua/library/docs/rio/2020/m4695.pdf · 2 Крок-1. Біохімія Н: збірник тестових

25

29. У сечі хворого вияв-

лено цукор, кетонові тіла.

Вміст глюкози в крові

становить 10,1 ммоль/л.

Наявність якого захво-

рювання можна при-

пустити ?

А. Атеросклероз.

В. Інфаркт міокарда.

С. Токсичний гепатит.

D. Панкреатит.

Е. *Цукровий діабет.

30. Унаслідок тривалого

голодування в організмі

людини швидко вичер-

пуються резерви вугле-

водів. Який із процесів

метаболізму відновлює

вміст глюкози у крові ?

А. Аеробний гліколіз.

В. Анаеробний гліколіз.

С. Пентозофосфатний шлях.

D. Глікогеноліз.

Е. *Глюконеогенез.

31. Хвороба Гірке – це за-

хворювання, при якому

спостерігається накопи-

чення глікогену в печінці.

Дефіцит якого ферменту є

причиною цього

захворювання ?

А. Глікогенфосфорилаза.

В. Глюкокіназа.

С. Піруваткіназа.

D. Фосфоглюкомутаза.

Е. *Глюкозо-6-фосфатаза.

32. При цукровому діабеті

збільшується вміст кето-

нових тіл у крові, що

призводить до

метаболічного ацидозу. З

якої речовини синте-

зуються кетонові тіла ?

А. Метилмалоніл-КоА.

В. Сукциніл-КоА.

С. Пропіоніл-КоА.

D. Малоніл-КоА.

Е. *Ацетил-КоА.

33. У хворого спостеріга-

ється кетонурія. При

якому захворюванні в сечі

з’являються кетонові

тіла ?

А. Туберкульозі нирки.

В. Гострому

гломерулонефриті.

С. Сечокам’яній хворобі.

D. Інфаркті нирки.

Е. *Цукровому діабеті.

34. Яка кількість молекул

АТФ може синтезуватися

при повному окисненні

ацетил-КоА в циклі

трикарбонових кислот ?

А. 1.

Page 26: КРОК-1 БІОХІМІЯ Збірник тестових завданьlib.sumdu.edu.ua/library/docs/rio/2020/m4695.pdf · 2 Крок-1. Біохімія Н: збірник тестових

26

В. 3.

С. 5.

D. 8.

Е. *12.

35. У пацієнта 40 років

непереносимість кисло-

молочних продуктів. Не-

достатністю якого

ферменту травлення

можна пояснити це

явище ?

А. Амілази.

В. Лактатдегідрогенази.

С. Мальтази.

D. Ліпази.

Е. *Лактази.

36. Хворий на цукровий

діабет після введення інсу-

ліну знепритомнів, спосте-

рігаються судоми. Який

результат дав біохімічний

аналіз крові на вміст

цукру ?

А. 10 ммоль/л.

В. 3,3 ммоль/л.

С. 8 ммоль/л.

D. 5,5 ммоль/л.

Е. *1,5 ммоль/л.

37. В ендокринологічному

відділенні з діагнозом

цукрового діабету пере-

буває жінка 40 років зі

скаргами на спрагу,

підвищений апетит. Які

патологічні компоненти

виявлено при лаборатор-

ному дослідженні сечі

пацієнтки ?

А. Кров.

В. Білок, амінокислоти.

С. Білок, креатин.

D. Білірубін, уробілін.

Е. *Глюкоза, кетонові тіла.

38. При обстеженні хворого

в крові виявлено цукру

16 ммоль/л, кетонових тіл

– 0,52 ммоль/л; в сечі -

цукру 2%, ацетонові тіла

+++, діурез 10 л на добу.

Який діагноз можна

припустити ?

A. Хвороба Іценка-Кушинга.

B. Нецукровий діабет.

C. Стероїдний діабет.

D. Нирковий діабет.

E. * Цукровий діабет.

39. У жінки 40 років

виявлена гемолітична

анемія, що зумовлена

генетичним дефектом

ферменту глюкозо-6-фос-

фатдегідрогенази в еритро-

цитах. Утворення якої

речовини пентозофосфат-

Page 27: КРОК-1 БІОХІМІЯ Збірник тестових завданьlib.sumdu.edu.ua/library/docs/rio/2020/m4695.pdf · 2 Крок-1. Біохімія Н: збірник тестових

27

ного шляху буде порушено

при цьому найбільше ?

A. Диоксиацетонфосфату.

B. Фосфоенолпірувату.

C. ФАДН2.

D. Глюкозо-6-фосфату.

E. *НАДФН.

40. Новонароджена дитина

відмовляється від їжі, в неї

виникли блювання,

пронос, а з часом

розвинулося помутніння

кришталика. При

обстеженні: цукор у крові -

8,5 ммоль/л, а в сечі – 1 %.

Який найбільш імовірний

діагноз ?

A. Цистинурія.

B. Алкаптонурія.

C. Тирозиноз.

D. Фенілкетонурія.

E. *Галактоземія.

41. У дитини виявили

галактоземію. Концентра-

ція глюкози в крові

суттєво не змінюється.

Дефіцитом якого ферменту

зумовлене це захворю-

вання ?

A. Гексокінази.

B. Амілопектин-1,6-

глюкозидази.

C. Фосфоглюкомутази.

D. Галактокінази.

E.*Галактозо-1-фосфат-

уридилтрансферази.

42. При хворобі Іценка-Ку-

шинга (гіперфункція кори

наднирників з підвищеною

продукцією кортикостерої-

дів) виникає гіперглікемія.

Який процес при цьому

стимулюється ?

A. Гліколіз.

B. Фосфороліз глікогену.

C. Цикл Кребса.

D. Пентозофосфатний шлях.

E. * Глюконеогенез.

43. Використання глюкози

відбувається шляхом її

транспорту з екстрацелю-

лярного простору через

плазматичну мембрану

всередину клітини. Цей

процес стимулюється

гормоном:

A. Адреналіном.

B. Глюкагоном.

C. Тироксином.

D. Альдостероном.

E. *Інсуліном.

44. Який полісахарид

природного походження є

антикоагулянтом ?

A. Декстран.

Page 28: КРОК-1 БІОХІМІЯ Збірник тестових завданьlib.sumdu.edu.ua/library/docs/rio/2020/m4695.pdf · 2 Крок-1. Біохімія Н: збірник тестових

28

B. Гіалуронова кислота.

C. Дерматансульфат.

D. Хондроїтинсульфат.

E. *Гепарин.

45. Для синтезу полі-

сахаридних ланцюгів

глікогену використовує-

ться попередник – активна

форма глюкози. Без-

посереднім донором

залишків глюкози в

процесі синтезу гліконегу

є:

A. Глюкозо-3-фосфат.

B. Глюкозо-1-фосфат.

C. АДФ-глюкоза.

D. Глюкозо-6-фосфат.

E. *УДФ-глюкоза.

46. Еритроцити людини не

містять мітохондрій. Який

основний шлях утворення

АТФ у цих клітинах ?

A. Аденілаткіназна реакція.

B. Аеробний гліколіз.

C. Пентозофосфатний шлях.

D. Креатинкіназна реакція.

E. *Анаеробний гліколіз.

47. У хворого у печінці спо-

стерігається накопичення

надмірної кількості гліко-

гену. Клінічно це виявля-

ється:

A. Фруктозурією.

B. Гіперглікемією.

C. Кетонурією.

D. Галактоземією.

E. *Гіпоглікемією.

48. У цитоплазмі міоцитів

розчинена велика

кількість метаболітів

окиснення глюкози.

Назвіть один із них, який

безпосередньо перетво-

рюється в лактат:

A. Фруктозо-6-фосфат.

B. Оксалоацетат.

C. Гліцерофосфат.

D. Глюкозо-6-фосфат.

E. *Піруват.

49. При хронічному

передозуванні глюко-

кортикоїдів у хворого

развивається гіпер-

глікемія. Назвіть процес

вуглеводного обміну, за

рахунок якого збільшує-

ться концентрація

глюкози:

A. Глікогенез.

B. Глікогеноліз.

C. Аеробний гліколіз.

D. Пентозофосфатний цикл.

E. *Глюконеогенез.

Page 29: КРОК-1 БІОХІМІЯ Збірник тестових завданьlib.sumdu.edu.ua/library/docs/rio/2020/m4695.pdf · 2 Крок-1. Біохімія Н: збірник тестових

29

50. Під час бігу на довгі

дистанції скелетна

мускулатура тренованої

людини використовує

глюкозу з метою

отримання енергії АТФ

для м’язового скорочення.

Назвіть основний процес

утилізації глюкози за цих

умов:

A. Глікогенез.

B. Анаеробний гліколіз.

C. Глікогеноліз.

D. Глюконеогенез.

E. *Аеробний гліколіз.

51. У жінки 40 років

хвороба Іценка-Кушинга –

стероїдний діабет. При

біохімічному обстеженні:

гіперглікемія, гіпо-

хлоремія. Який із пере-

лічених нижче процесів

активується в першу

чергу ?

A. Гліколіз.

B. Глікогеноліз.

C. Реабсорбція глюкози.

D. Транспорт глюкози в

клітину.

E. *Глюконеогенез.

52. Біоенергетика мозку

характеризується значною

залежністю від постачання

киснем. Який субстрат

окиснення має найбільше

значення для забезпечення

енергією мозку ?

A. Фосфоенолпіруват.

B. Жирні кислоти.

C. Кетонові тіла.

D. Гліцерол-3-фосфат.

E. *Глюкоза.

53. У вагітної жінки віком

28 років досліджували

ферменти в клітинах

амніотичної рідини. При

цьому виявили

недостатню активність

-глюкуронідази. Який

патологічний процес

спостерігається у жінки ?

A. Ліпідоз.

B. Глікогеноз.

C. Аглікогеноз.

D. Колагеноз.

E. *Мукополісахаридоз.

54. При перетворенні

глюкози в пентозо-

фосфатному циклі утворю-

ються фосфати різних

моносахаридів. Яка з цих

речовин може бути

використана для синтезу

нуклеїнових кислот ?

A. Ксилулозо-5-фосфат.

B. Рибулозо-5-фосфат.

C. Еритрозо-4-фосфат.

Page 30: КРОК-1 БІОХІМІЯ Збірник тестових завданьlib.sumdu.edu.ua/library/docs/rio/2020/m4695.pdf · 2 Крок-1. Біохімія Н: збірник тестових

30

D. Седогептулозо-7-фосфат.

E. *Рибозо-5-фосфат.

55. Одним із факторів, що

призводять до карієсу, є

органічні кислоти, які

виробляються мікро-

організмами зубного

нальоту. Який із

ферментів призводить до

утворення молочної

кислоти ?

A. Алкогольдегідрогеназа.

B. Гексокіназа.

C. Альдолаза.

D. Фосфофруктокіназа.

E. *Лактатдегідрогеназа.

56. У дитини 6 років

знижена активність, є

ознаки порушення

координації рухів. При

обстеженні встановлений

генетичний дефект одного

з ферментів ПВК- дегідро-

геназного комплексу.

Назвіть, який із переліче-

них нижче лабораторних

показників був вирішаль-

ним в обґрунтуванні

діагнозу:

A. Лактат нижче норми.

B. Аланін нижче норми.

C. Піруват нижче норми.

D. Аланін вище норми.

E. *Піруват вище норми.

57. При недостатньому

кровообігу в період

інтенсивноі м’язової

роботи у м’язах в резуль-

таті анаеробного гліколізу

накопичується молочна

кислота. Яка її подальша

участь ?

A. Використовується в тка-

нинах для синтезу вищих

жирних кислот.

B. Видаляється через нирки

із сечею.

C. Використовується у

м’язах для синтезу глюкози.

D. Використовується

іншими тканинами для

синтезу кетонових тіл.

E. *Залучається в

глюконеогенез у печінці.

58. У жінки 62 років розви-

нулася катаракта (помут-

ніння кришталика) на

фоні цукрового діабету.

Назвіть тип модифікації

білків, який має місце при

діабетичній катаракті:

A. Обмежений протеоліз.

B. Фосфорилювання.

C. АДФ-рибозилювання.

D. Метилювання.

E. *Глікозилювання.

Page 31: КРОК-1 БІОХІМІЯ Збірник тестових завданьlib.sumdu.edu.ua/library/docs/rio/2020/m4695.pdf · 2 Крок-1. Біохімія Н: збірник тестових

31

59. У 8-місячної дитини

спостерігаються блювання

та діарея після прийому

фруктових соків. Наванта-

ження фруктозою приз-

вело до гіпоглікемії.

Зазначте, спадковий дефі-

цит якого ферменту є

причиною стану дитини:

A. Фруктозо-1,6-

дифосфатази.

B. Фруктокінази.

C. Гексокінази.

D. Фосфофруктокінази.

E. *Фруктозо -1- фосфат-

альдолази.

60. При дослідженні крові

у хворого виявлена

виражена гіпоглікемія

натще. При дослідженні

біоптату печінки

виявилося, що в клітинах

печінки не відбувається

синтезу глікогену. Дефіцит

якого ферменту є

причиною захворювання ?

A. Альдолази.

B. Фосфорилази а.

C. Фруктозодифосфатази.

D. Піруваткарбоксилази.

E. *Глікогенсинтази.

61. Повне окиснення

молекули глюкози і

спряження його з фосфори-

люванням еквівалентне

утворенню такої сумарної

кількості молекул АТФ:

A. 58.

B. 8.

C. 12.

D. 52.

E. *38.

62. У хворої 58 років стан

тяжкий, свідомість затьма-

рена, шкіра суха, очі

запалі, ціаноз, запах

ацетону з рота. Результати

аналізів: глюкоза крові

15,1 ммоль/л, в сечі 3,5 %

глюкози. Причиною

такого стану є:

A. Гіповолемічна кома.

B. Гіпоглікемічна кома.

C. Анафілактичний шок.

D. Уремічна кома.

E. * Гіперглікемічна кома.

63. Однорічна дитина

відстає в розумовому

розвитку від своїх

однолітків. Уранці: блюва-

ння, судоми, втрата

свідомості. У крові –

гіпоглікемія натще. З

дефектом якого ферменту

це пов’язано:

A. Лактази.

Page 32: КРОК-1 БІОХІМІЯ Збірник тестових завданьlib.sumdu.edu.ua/library/docs/rio/2020/m4695.pdf · 2 Крок-1. Біохімія Н: збірник тестових

32

B. Фосфорилази.

C. Аргінази.

D. Сахарази.

E. *Глікогенсинтази.

64. У клініку доставлено

пацієнта з ознаками

гострого алкогольного

отруєння. Які зміни

вуглеводного обміну

характерні для цього

стану ?

A. У м’язах посилюється

аеробний розпад глюкози.

B. У печінці посилюється

розпад глікогену.

C. У м’язах переважає

анаеробний розпад глюкози.

D. У печінці посилюється

глюконеогенез.

E. *У печінці знижується

швидкість глюконеогенезу.

65. Дитина 10 років

постійно скаржиться на

біль і судоми у м'язах після

фізичного навантаження.

При обстеженні крові

вміст глюкози, лактату і

креатину відповідає

фізіологічній нормі. В сечі

визначається міоглобін. У

біоптаті м'язів виявлений

дефіцит фосфорилази

глікогену – ферменту, що

каталізує перетворення:

A. “Затравки” глікогену в

глікоген.

B. Глюкозо-6-фосфату в

глюкозу.

C. Глюкозо-6-фосфату в

глікоген.

D. Глюкози в глюкозо-6-

фосфат.

E. *Глікогену в глюкозо-1-

фосфат.

66. У хворого 57 років,

який страждає на

цукровий діабет,

розвинувся кетоацидоз.

Біохімічною основою

цього стану є зменшення

ступеня утилізації ацетил-

КоА через дефіцит в

гепатоцитах:

A. Сукцинату.

B. 2-Оксоглутарату.

C. Глутамату.

D. Аспартату.

E. *Оксалоацетату.

67. Цикл Кребса вiдiграє

важливу роль у реалiзацiї

глюкопластичного ефекту

амiнокислот. Це зумовлено

перетворенням безазо-

тистого залишку:

A. Цитрату.

Page 33: КРОК-1 БІОХІМІЯ Збірник тестових завданьlib.sumdu.edu.ua/library/docs/rio/2020/m4695.pdf · 2 Крок-1. Біохімія Н: збірник тестових

33

B. Малату.

C. Сукцинату.

D. Фумарату.

E. *Оксалоацетату.

68. У хворих на алкоголізм

часто спостерігається

гіповітаміноз B1, який є

наслідком порушення

харчування. Симптомами

гіповітамінозу B1 є розлади

нервової системи, психози,

втрата пам'яті. Чому до

дефіциту вітаміну B1

особливо чутливі клітини

нервової тканини ?

A. Знижується інтенсивність

гліколізу.

B. Посилюється ліполіз у

жировій тканини.

C. Порушується окиснення

жирних кислот.

D. Підвищується

інтенсивність гліколізу.

E. *Порушується аеробний

розпад глюкози.

69. У хворого в коматоз-

ному стані відчувається

запах ацетону з рота. Вміст

глюкози в плазмі крові –

18 ммоль/л. Яку з ком

найбільш віро-гідно

запідозрити в даному

випадку ?

А. Гіпоглікемічну.

В. Токсичну.

С. Гіперосмолярну.

D. Лактатацидемічну.

Е. *Кетоацидемічну.

70. У реанімаційне відді-

лення доставлено немовля

з такими симптомами:

блювання, діарея, пору-

шення росту і розвитку,

катаракта, розумова

відсталість. Був встанов-

лений діагноз – галакто-

земія. Дефіцит якого

ферменту має місце ? А. УДФ-глюкозо-4-

епімерази.

В. Глюкокінази.

С. Глюкозо-6-фосфат-

дегідрогенази.

D. УДФ-глюкозо-

пірофосфорилази.

Е. *Галактозо-1-фосфат-

уридилтрансферази.

71. У хворого, що страж-

дає на ішемічну хворобу

серця, після проведеного

лікування в серцевому

м’язі значно зменшився

вміст лактату. Актива-

цією якого біохімічного

процесу в міокарді можна

це пояснити ?

Page 34: КРОК-1 БІОХІМІЯ Збірник тестових завданьlib.sumdu.edu.ua/library/docs/rio/2020/m4695.pdf · 2 Крок-1. Біохімія Н: збірник тестових

34

А. Глікогенезу.

В. Глюконеогенезу.

С. Глікогенолізу.

D. Пентозофосфатного

циклу.

Е. *Аеробного окиснення

глюкози.

72. У новонародженої

дитини розвивається

жирове переродження

печінки, спостерігаються

галактозурія і аміно-

ацидурія. У крові під-

вищений загальний

білірубін. Яку речовину

слід ви-ключити з раціону

дитини ?

А. Сахарозу.

В. Фенілаланін.

С. Холестерин.

D. Жирні кислоти.

Е. *Молочний цукор.

73. У клініку доставлено

пацієнта з ознаками

гострого алкогольного

отруєння. Які зміни

вуглеводного обміну

характерні для такого

стану ?

А. У печінці посилюється

розпад глікогену.

В. У м’язах посилюється

аеробний розпад глюкози.

С. У печінці посилюється

глюконеогенез.

D. У м’язах переважає

анаеробний розпад глюкози.

Е. *У печінці знижується

швидкість глюконеогенезу.

74. У немовляти

спостерігаються блювання

і пронос, загальна

дистрофія, гепато- і

спленомегалія. При

припиненні годування

молоком симптоми

зникають. Який спадковий

дефект у немовляти ?

А. Гіперсекреція залоз

зовнішньої секреції.

В. Недостатність глюкозо-6-

фосфатдегідрогенази.

С. Порушення обміну

фенілаланіну.

D. Порушення обміну

тирозину.

Е. *Порушення обміну

галактози.

75. У лікарню доставили

хворого на цукровий

діабет у непритомному

стані. Дихання типу

Кусмауля, АТ - 80/50 мм

рт. ст., запах ацетону з

рота. Накопиченням в

організмі яких речовин

Page 35: КРОК-1 БІОХІМІЯ Збірник тестових завданьlib.sumdu.edu.ua/library/docs/rio/2020/m4695.pdf · 2 Крок-1. Біохімія Н: збірник тестових

35

можна пояснити

виникнення таких

порушень ?

А. Молочної кислоти.

В. Вугільної кислоти.

С. Складних вуглеводів.

D. Модифікованих

ліпопротеїдів.

Е. *Кетонових тіл.

76. Хворому, що має

непереносимість анти-

біотиків, для лікування

пневмонії призначено

сульфален. Через кілька

днів у хворого розвинувся

гемоліз еритроцитів.

Недостатність якого

ферменту в організмі

хворого сприяла розвитку

цього побічного ефекту ?

А. Ацетальдегід-

дегідрогенази.

В. Аланінамінотрансферази.

С. N-Ацетилтрансферази.

D. Холінестерази.

Е. *Глюкозо-6-фосфат-

дегідрогенази.

77. Хворий на цукровий

діабет після ін’єкції

інсуліну знепритомнів,

почалися судоми. Який

результат може дати

біохімічний аналіз крові на

вміст цукру ?

А. 10,0 ммоль/л.

В. 8,0 ммоль/л.

С. 5,5 ммоль/л.

D. 3,3 ммоль/л.

Е. *1,5 ммоль/л.

78. Жінка 26 років скар-

житься на сухість у роті,

спрагу. Під час обсте-

ження встановлено:

глюкоза в крові –

6,5 ммоль/л, глюкозурія.

Для якого із зазначених

станів найбільш

характерні ці симптоми ?

А. Стероїдний діабет.

В. Аліментарна глюкозурія.

С. Нирковий діабет.

D. Нецукровий діабет.

Е. *Цукровий діабет.

79. У хворої 38 років після

прийому аспірину і

сульфаніламідів спо-

стерігається посилений

гемоліз, викликаний недо-

статністю глюкозо-6-

фосфатдегідрогенази.

Порушенням утворення

якого коферменту

зумовлена ця патологія ?

А. Піридоксальфосфат.

В. Убіхінон.

Page 36: КРОК-1 БІОХІМІЯ Збірник тестових завданьlib.sumdu.edu.ua/library/docs/rio/2020/m4695.pdf · 2 Крок-1. Біохімія Н: збірник тестових

36

С. ФМНН2.

D. ФАДН2.

Е. *НАДФН.

80. У людини вміст

глюкози в крові становить

15 ммоль/л (поріг

реабсорбції – 10 ммоль/л),

наслідком цього буде:

А. Зменшення діурезу.

В. Зменшення секреції

вазопресину.

С. Зменшення реабсорбції

глюкози.

D. Зменшення секреції

альдостерону.

Е. *Глюкозурія.

81. У дівчинки 7 років

ознаки анемії. Лабора-

торно встановлений

дефіцит піруваткінази в

еритроцитах. Порушення

якого процесу відіграє

головну роль у розвитку

анемії у дівчинки ?

А. Окиснювальне фосфо-

рилювання.

В. Розкладання пероксидів.

С. Тканинне дихання.

D. Дезамінування аміно-

кислот.

Е. *Анаеробний гліколіз.

1.5. ОБМІН ЛІПІДІВ

1. В основі ліполізу

(мобілізації жирних кислот

з жирових депо) лежить

ферментативний процес

гідролізу жирів до жирних

кислот та гліцерину.

Утворені жирні кислоти

надходять у кров і

транспортуються в складі:

А. ЛПНЩ.

В. Глобулінів.

С. ЛПВЩ.

D. Хіломікронів.

Е. *Альбумінів.

2. Після споживання

жирної їжі хворий відчуває

дискомфорт. У випорож-

неннях містяться непере-

травлені краплини жиру.

Реакція в сечі на жовчні

кислоти позитивна.

Причиною такого стану є

дефіцит у травному тракті:

А. Фосфоліпідів.

В. Жирних кислот.

С. Хіломікронів.

D. Тригліцеридів.

Е. *Жовчних кислот.

3. Хворий страждає на

артеріальну гіпертензію,

атеросклеротичне уражен-

Page 37: КРОК-1 БІОХІМІЯ Збірник тестових завданьlib.sumdu.edu.ua/library/docs/rio/2020/m4695.pdf · 2 Крок-1. Біохімія Н: збірник тестових

37

ня судин. Вживання якого

ліпіду йому необхідно

обмежити ?

А. Лецитину.

В. Олеїнової кислоти.

С. Фосфатидилсерину.

D. Моноолеатгліцериду.

Е. *Холестерину.

4. У чоловіка 35 років

феохромоцитома. У крові –

підвищений рівень адрена-

ліну та норадреналіну,

концентрація вільних жир-

них кислот збільшена в 11

разів. Активація якого

ферменту під впливом

адреналіну прискорює

ліполіз ?

А. Холестеролестерази.

В. Ліпопротеїдліпази.

С. Фосфоліпази А2.

D. Фосфоліпази С.

Е. *ТАГ-ліпази.

5. Для серцевого м’яза ха-

рактерним є аеробний

характер окиснення

субстратів. Назвіть основ-

ний із них:

А. Амінокислоти.

В. Триацилгліцероли.

С. Гліцерол.

D. Глюкоза.

Е. *Жирні кислоти.

6. В організмі людини

основним місцем депону-

вання триацилгліцеролів

(ТАГ) є жирова тканина.

Разом з тим їх синтез

відбувається і в

гепатоцитах. У вигляді

якої речовини ТАГ

транспортуються з печінки

в жирову тканину ?

А. Хіломікронів.

В. Комплексу з альбуміном.

С. ЛПНЩ.

D. ЛПВЩ.

Е. *ЛПДНЩ.

7. При обстеженні хворого

виявлено підвищений

вміст у сироватці крові

ЛПНЩ. Яке захворювання

можна передбачити у

хворого ?

А. Гастрит.

В. Ураження нирок.

С. Гострий панкреатит.

D. Запалення легень.

Е. *Атеросклероз.

8. Для підвищення ре-

зультатів спортсмену ре-

комендували застосову-

вати препарат, який

містить карнітин. Який

процес найбільшою мірою

активується карнітином ?

Page 38: КРОК-1 БІОХІМІЯ Збірник тестових завданьlib.sumdu.edu.ua/library/docs/rio/2020/m4695.pdf · 2 Крок-1. Біохімія Н: збірник тестових

38

А. Синтез стероїдних

гормонів.

В. Синтез кетонових тіл.

С. Синтез ліпідів.

D. Тканинне дихання.

Е. *Транспорт жирних

кислот у мітохондрії.

9. При жировій інфільт-

рації печінки порушується

синтез фосфоліпідів. Яка з

нижченаведених речовин

може підсилювати процеси

метилювання в синтезі

фосфоліпідів ?

А. Цитрат.

В. Аскорбінова кислота.

С. Глюкоза.

D. Гліцерин.

Е. *Метіонін.

10. Дані суб’єктивного й

об’єктивного обстеження

дозволяють припустити

наявність у хворого

запального процесу в

жовчному міхурі,

порушення колоїдних

властивостей жовчі,

імовірність утворення

жовчних каменів. Що

може спричинити їх

утворення ?

А. Оксалати.

В. Урати.

С. Фосфати.

D. Хлориди.

Е. *Холестерин.

11. Хворому 65 років, з

ознаками загального ожи-

ріння, небезпекою жирової

дистрофії печінки

рекомендовано дієту,

збагачену ліпотропними

речовинами, серед яких

важливе значення має

вміст у продуктах:

А. Вітаміну С.

В. Холестерину.

С. Глюкози.

D. Гліцину.

Е. *Метіоніну.

12. Арахідонова кислота як

незамінний компонент їжі

є попередником біологічно

активних речовин. Яка

сполука синтезується з

неї ?

А. Етаноламін.

В. Холін.

С. Норадреналін.

D. Трийодтиронін.

Е. *Простагландин Е1.

13. При дослідженні

плазми крові пацієнта

через 4 год після вживання

жирів встановлено її

Page 39: КРОК-1 БІОХІМІЯ Збірник тестових завданьlib.sumdu.edu.ua/library/docs/rio/2020/m4695.pdf · 2 Крок-1. Біохімія Н: збірник тестових

39

каламутність. Найбільш

імовірною причиною цього

стану є підвищення

концентрації в плазмі:

А. ЛПВЩ.

В. Фосфоліпідів.

С. ЛПНЩ.

D. Холестерину.

Е. *Хіломікронів.

14. Недостатня секреція

якого ферменту зумовлює

неповне перетравлювання

жирів у травному тракті та

появу великої кількості

нейтральних жирів у

калових масах ?

А. Пепсину.

В. Фосфоліпази.

С. Ентерокінази.

D. Амілази.

Е. *Панкреатичної ліпази.

15. У клініку потрапила

дитина 1 року з ознаками

ураження м’язів. Після

обстеження виявлено де-

фіцит карнітину в м’язах.

Біохімічним підґрунтям

цієї патології є порушення

процесу:

А. Синтезу актину й

міозину.

В. Регуляції Са2+ в

мітохондріях.

С. Субстратного фосфори-

лювання.

D. Утилізації молочної

кислоти.

Е. *Транспорту жирних

кислот у мітохондрії.

16. Унаслідок укусу змії

людина почала задиха-

тися, в сечі з'явився

гемоглобін. У крові

виявлений гемоліз. Дія

токсичої зміїної отрути

призводить до:

A. Утворення тригліцеридів.

B. Ацидозу.

C. Поліурії.

D. Розвитку алкалозу.

E. *Утворення лізо-

лецитину.

17. Пацієнт голодує 48 год.

Які речовини викори-

стовуються м’язовою

тканиною як джерела

енергії за цих умов?

A. Амінокислоти.

B. Гліцерин.

C. Піруват.

D. Лактат.

E. *Кетонові тіла.

18. При дефіциті

ліпотропних факторів

розвивається жирове

Page 40: КРОК-1 БІОХІМІЯ Збірник тестових завданьlib.sumdu.edu.ua/library/docs/rio/2020/m4695.pdf · 2 Крок-1. Біохімія Н: збірник тестових

40

переродження печінки.

Яка з наведених речовин

належить до ліпотропних

факторів ?

A. Рибофлавін.

B. Холестерин.

C. Триацилгліцериди.

D. Жирні кислоти.

E. *Холін.

19. У крові хворих на

цукровий діабет

спостерігається підвищен-

ня вмісту вільних жирних

кислот (НЕЖК). Причи-

ною цього може бути:

A. Зниження активності

лецитинхолестеринацил-

трансферази плазми крові.

B. Накопичення в цитозолі

пальмітоїл-КоА.

C. Активація утилізації кето-

нових тіл.

D. Активація синтезу

аполіпопротеїнів А-1, А-2,

А-4.

E. *Підвищення активності

тригліцеридліпази адипо-

цитів.

20. Пацієнтці з високим

ступенем ожиріння як

харчову добавку

рекомендовано карнітин

для покращення “спа-

лювання” жиру. Яку

безпосередню участь бере

карнітин у процесі

окиснення жирів ?

A. Активація внутрішньоклі-

тинного ліполізу.

B. Транспорт ВЖК із

жирових депо в тканини.

C. Бере участь в одній із

реакцій бета-окиснення.

D. Активація ВЖК.

E. *Транспорт ВЖК з цито-

плазми в мітохондрії.

21. Під час аналізу крові

виявлено високий вміст

холестерину в β-ліпо-

протеїновій фракції. Який

можливий наслідок цього

явища ?

A. Жовтяниця.

B. Цукровий діабет.

C. Ожиріння.

D. Гіпертонія.

E. *Атеросклероз.

22. Простагландини вико-

ристовуються в клініці як

терапевтичні засоби. Що є

субстратом для їх синтезу ?

A. Глутамінова кислота.

B. Фосфатидна кислота.

C. Пальмітинова кислота.

D. Стеаринова кислота.

E. * Арахідонова кислота.

Page 41: КРОК-1 БІОХІМІЯ Збірник тестових завданьlib.sumdu.edu.ua/library/docs/rio/2020/m4695.pdf · 2 Крок-1. Біохімія Н: збірник тестових

41

23. У хворої дитини при

аналізі крові встановлено

спадкову гіперліпо-

протеїнемію. Генетичний

дефект синтезу якого

ферменту обумовлює цю

патологію ?

A. Фенілаланінгідроксилази.

B. Глікозидази.

C. Протеїнази.

D. Гемсинтетази.

E. *Ліпопротеїнліпази.

24. Який із перелічених

гормонів знижує

швидкість ліполізу в

жировій тканині ?

A. Норадреналін.

B. Адреналін.

C. Гідрокортизон.

D. Соматотропін.

E. *Інсулін.

25. При обстеженні

підлітка, який страждає на

ксантоматоз, виявлена

сімейна гіперхолесте-

ринемія. Концентрація

яких ліпопротеїнів значно

підвищена в крові при

даній патології ?

A. НЕЖК.

B. Хіломікронів.

C. ЛПДНГ.

D. ЛПВГ.

E. *ЛПНГ.

26. Експериментальній

тварині давали надлиш-

кову кількість глюкози,

міченої вуглецем, упро-

довж тижня. В якій

сполуці можна виявити

мітку ?

A. У арахідоновій кислоті.

B. У метіоніні.

C. У вітаміні А.

D. У холіні.

E. *У пальмітиновій кислоті.

27. Дефіцит якого

ферменту є причиною

неповного перетравлення

жирів у шлунково-

кишковому тракті і

збільшення кількості

нейтральних жирів у калі?

А. Шлункової ліпази.

В. Кишкової ліпази.

С. Печінкової ліпази.

D. Ентерокінази.

Е. *Панкреатичної ліпази.

28. Хворий після вживання

жирної їжі відчував

нудоту, в’ялість, з часом

з’явилися ознаки

стеатореї. У крові

холестерин – 9,2 ммоль/л.

Причиною такого стану є

нестача:

Page 42: КРОК-1 БІОХІМІЯ Збірник тестових завданьlib.sumdu.edu.ua/library/docs/rio/2020/m4695.pdf · 2 Крок-1. Біохімія Н: збірник тестових

42

А. Хіломікронів.

В. Тригліцеридів.

С. Фосфоліпідів.

D. Жирних кислот.

Е. *Жовчних кислот.

29. При копрологічному

дослідженні встановлено,

що кал знебарвлений, у

ньому виявлені краплини

нейтрального жиру.

Найбільш вірогідною

причиною цього є

порушення:

А. Секреції підшлункового

соку.

В. Секреції кишкового соку.

С. Кислотності шлункового

соку.

D. Процесів всмоктування в

кишечнику.

Е. *Надходження жовчі в

кишечник.

30. Чоловік 67 років

страждає на атеросклероз

судин головного мозку. Під

час обстеження було

виявлено гіперліпідемію.

Вміст якого класу ліпо-

протеїнів плазми крові

буде значно підвищеним

при біохімічному

дослідженні ?

А. Ліпопротеїнів високої

щільності.

В. Комплексів жирних

кислот з альбумінами.

С. Хіломікронів.

D. Ліпопротеїнів дуже

низької щільності.

Е. *Ліпопротеїнів низької

щільності.

31. При обстеженні

хворого, який страждає на

гострий панкреатит, у

крові виявлений підви-

щений вміст хіломікронів.

Активність якого

ферменту значно знижена

при даній патології ?

А. Панкреатичної

фосфоліпази.

В. Тканинної дигліцерид-

ліпази.

С. Тканинної тригліцерид-

ліпази.

D. Ліпази підшлункової

залози.

Е. *Ліпопротеїнліпази.

32. Пацієнту похилого віку

рекомендовано з метою

попередження розвитку

жирової інфільтрації пе-

чінки вживати в їжу сир.

Яка незамінна аміно-

кислота, необхідна для

Page 43: КРОК-1 БІОХІМІЯ Збірник тестових завданьlib.sumdu.edu.ua/library/docs/rio/2020/m4695.pdf · 2 Крок-1. Біохімія Н: збірник тестових

43

синтезу фосфоліпідів,

міститься в сирі ?

А. Пролін.

В. Валін.

С. Лізин.

D. Аргінін.

Е. *Метіонін.

33. При обстеженні у

хворого виявлено

підвищений вміст

ліпопротеїнів низької

щільності в сироватці

крові. Яке захворювання

можна очікувати у цього

хворого ?

А. Гострий панкреатит.

В. Запалення легень.

С. Ушкодження нирок.

D. Гастрит.

Е. *Атеросклероз.

34. У крові хворих на

цукровий діабет

спостерігається підвищен-

ня вмісту вільних жирних

кислот. Причиною цього

може бути:

А. Активація утилізації

кетонових тіл.

В. Зниження активності

фосфатидилхолін-холесте-

рин-ацилтрансферази крові.

С. Накопичення в цитозолі

пальмітоїл-КоА.

D. Активація синтезу

аполіпопротеїнів А-1, А-2,

А-4.

Е. *Підвищення активності

тригліцеридліпази

адипоцитів.

35. Для покращення

результатів спортсмену

рекомендовано застосову-

вати препарат, що містить

карнітин. Який процес

найбільшою мірою

активується карнітином ?

А. Синтез кетонових тіл.

В. Синтез стероїдних

гормонів.

С. Синтез ліпідів.

D. Тканинне дихання.

Е. *Транспорт жирних

кислот у мітохондрії.

36. У хворих на цукровий

діабет і під час тривалого

голодування в крові

збільшується вміст

кетонових тіл. З якої

речовини вони

синтезуються ?

А. Цитрату.

В. Малату.

С. Кетоглутарату.

D. Сукциніл-КоА.

Е. *Ацетил-КоА.

Page 44: КРОК-1 БІОХІМІЯ Збірник тестових завданьlib.sumdu.edu.ua/library/docs/rio/2020/m4695.pdf · 2 Крок-1. Біохімія Н: збірник тестових

44

37. Людина 28 років

споживає надмірну

кількість вуглеводів (600 г

на добу), що перевищує її

енергетичні потреби. Який

процес буде активуватися

у даному випадку ?

А. Глюконеогенез.

В. Гліколіз.

С. Окиснення жирних

кислот.

D. Ліполіз.

Е. *Ліпогенез.

1.6. ОБМІН

АМІНОКИСЛОТ

ТА БІЛКІВ

1. Альбіноси погано

переносять вплив

ультрафіолету – під час

засмагання отримують

опіки. Порушення

метаболізму якої

амінокислоти лежить в

основі цього явища ?

А. Триптофану.

В. Метіоніну.

С. Гістидину.

D. Глутамінової.

Е. *Фенілаланіну.

2. У крові хворого на рак

сечового міхура високий

вміст серотоніну й

оксиантранілової кислоти.

З надмірним надходжен-

ням в організм якої аміно-

кислоти це пов’язано?

А. Тирозину.

В. Аланіну.

С. Гістидину.

D. Метіоніну.

Е. *Триптофану.

3. Немовля відмовляється

від материнського молока,

збуджене, дихання не-

ритмічне, сеча має запах

пивної закваски або клено-

вого сиропу. Природжений

дефект якого ферменту

зумовив цю патологію ?

А. Аспартатаміно-

трансферази.

В. Глюкозо-6-фосфат-

дегідрогенази.

С. Гліцеролкінази.

D. УДФ-глюкуроніл-

трансферази.

Е. *Дегідрогенази розгалуд-

жених α-кетокислот.

4. У немовляти на 6-й день

у сечі виявлено надлишок

фенілпірувату та фенілаце-

тату. Обмін якої

амінокислоти порушено в

організмі людини ?

Page 45: КРОК-1 БІОХІМІЯ Збірник тестових завданьlib.sumdu.edu.ua/library/docs/rio/2020/m4695.pdf · 2 Крок-1. Біохімія Н: збірник тестових

45

А. Метіоніну.

В. Триптофану.

С. Аргініну.

D. Гістидину.

Е. *Фенілаланіну.

5. У хворого з черепною

травмою спостерігаються

судомні напади, що

регулярно повторюються.

Утворення якого біоген-

ного аміну порушено при

цьому стані ?

А. Гістаміну.

В. Дофаміну.

С. Адреналіну.

D. Серотоніну.

Е. *ГАМК.

6. При алкаптонурії в сечі

хворого виявлено велику

кількість гомогентизи-

нової кислоти (сеча темніє

на повітрі). Природжений

дефект якого ферменту

спостерігається ?

А.Тирозинамінотрансферази

В. Аланінамінотрансферази.

С. Тирозинази.

D. Фенілаланін-4-моно-

оксигенази.

Е. *Оксидази гомогентизи-

нової кислоти.

7. У хворих на колагеноз

діагностують процес

деструкції сполучної

тканини. Підвищення

вмісту яких сполук у крові

це підтверджує ?

А. Ізоферментів ЛДГ.

В. Креатину та креатиніну.

С. Уратів.

D. Трансаміназ.

Е. *Гідроксипроліну та

гідроксилізину.

8. У чоловіка 53 років

діагностовано хворобу

Педжета. У добовій сечі

різко підвищений рівень

гідроксипроліну, що

свідчить передусім про

посилення розпаду:

А. Кератину.

В. Фібриногену.

С. Альбуміну.

D. Еластину.

Е. *Колагену.

9. У психіатрії для ліку-

вання деяких захворювань

ЦНС використовують

біогенні аміни. Назвіть

препарат цієї групи, який є

медіатором гальмування:

А. Дофамін.

В. Гістамін.

С. Серотонін.

Page 46: КРОК-1 БІОХІМІЯ Збірник тестових завданьlib.sumdu.edu.ua/library/docs/rio/2020/m4695.pdf · 2 Крок-1. Біохімія Н: збірник тестових

46

D. Таурин.

Е. *γ-Аміномасляна кислота.

10. При хворобі Вільсона

(гепатоцеребральна дис-

трофія) в крові знижений

вміст церулоплазміну.

Який процес порушується

внаслідок нестачі цього

транспортного білка ?

А. Декарбоксилювання

амінокислот.

В. Розпад тканинних білків.

С. Трансамінування аміно-

кислот.

D. Синтез сечовини.

Е. *Комплексоутворення

амінокислот із міддю.

11. У дитини грудного віку

спостерігається потем-

ніння склер, слизових

оболонок, вушних

раковин. Сеча набуває

темного кольору на

повітрі. У крові та сечі

виявлено гомогентизинову

кислоту. Який діагноз

можна припустити ?

А. Гемолітична анемія.

В. Альбінізм.

С. Цистинурія.

D. Порфірія.

Е. *Алкаптонурія.

12. До лікарні потрапив 9-

річний хлопчик, розумово

й фізично відсталий. При

біохімічному аналізі крові

виявлено підвищену кіль-

кість фенілаланіну. Блоку-

вання якого ферменту

може призвести до такого

стану ?

А. Глутаматдекарбоксилази.

В. Оксидази

гомогентизинової кислоти.

С. Глутамінтрансамінази.

D. Аспартатамінотрансфе-

рази.

Е. *Фенілаланін-4-моно-

оксигенази.

13. Мати виявила занадто

темне забарвлення сечі її 5-

річної дитини. Дитина

скарг ніяких не

висловлює. Жовчних

пігментів у сечі немає.

Встановлено діагноз -

алкаптонурія. Дефіцит

якого ферменту спостері-

гається ?

А. Оксидази оксифенілпі-

рувату.

В. Фенілаланін-4-

гідроксилази.

С. Тирозинази.

D. Декарбоксилази феніл-

пірувату.

Page 47: КРОК-1 БІОХІМІЯ Збірник тестових завданьlib.sumdu.edu.ua/library/docs/rio/2020/m4695.pdf · 2 Крок-1. Біохімія Н: збірник тестових

47

Е. *Оксидази гомогентизи-

нової кислоти.

14. Аміак – дуже отруйна

речовина, особливо для

нервової системи. Яка

сполука бере активну

участь у знешкодженні

аміаку в тканинах мозку ?

А. Лізин.

В. Аланін.

С. Пролін.

D. Гістидин.

Е. *Глутамінова кислота.

15. Біогенні аміни: гіста-

мін, серотонін, дофамін та

ін. – дуже активні

речовини, які впливають

на різноманітні фізіоло-

гічні функції організму. У

результаті якого процесу

утворюються біогенні

аміни в тканинах

організму ?

А. Відновного

дезамінування амінокислот.

В. Гідролітичного

дезамінування амінокислот.

С. Трансамінування аміно-

кислот.

D. Окиснення амінокислот.

Е. *Декарбоксилювання амі-

нокислот.

16. У сечі новонародженого

визначаються цитрулін та

високий рівень аміаку.

Утворення якої речовини

найімовірніше порушено ?

А. Аміаку.

В. Сечової кислоти.

С. Креатину.

D. Креатиніну.

Е. *Сечовини.

17. Добовий раціон дорос-

лої здорової людини

повинні становити жири,

білки, вуглеводи, вітаміни,

мінеральні солі та вода.

Назвіть кількість білка,

яка забезпечує нормальну

життєдіяльність

організму:

А. 40-50 г на добу.

В. 50-60 г на добу.

С.10-20 г на добу.

D. 70-80 г на добу.

Е. *100 – 120 г на добу.

18. Хворий 13 років скар-

житься на загальну

слабість, запаморочення,

швидку втомлюваність.

Спостерігається відставан-

ня в розумовому розвитку.

При обстеженні виявлено

високу концентрацію

валіну, ізолейцину, лей-

Page 48: КРОК-1 БІОХІМІЯ Збірник тестових завданьlib.sumdu.edu.ua/library/docs/rio/2020/m4695.pdf · 2 Крок-1. Біохімія Н: збірник тестових

48

цину в крові та сечі. Сеча

має специфічний запах.

Що може бути причиною

такого стану ?

А. Гістидинемія.

В. Хвороба Аддісона.

С. Тирозиноз.

D. Дифузний ендемічний

зоб.

Е. *Хвороба кленового

сиропу.

19. Яка із сполук є акцеп-

тором аміногруп у

реакціях трансамінування

амінокислот ?

А. Аргініносукцинат.

В. Орнітин.

С. Лактат.

D. Цитрулін.

Е. *α-Кетоглутарат.

20. До лікарні швидкої

медичної допомоги

доставили дитину 7 років у

стані алергічного шоку, що

розвинувся після укусу

оси. У крові – підвищена

концентрація гістаміну. У

результаті якої реакції

утворюється цей амін ?

А. Відновлення.

В. Гідроксилювання.

С. Дегідрування.

D. Дезамінування.

Е. *Декарбоксилювання.

21. Дитина 10-місячного

віку, батьки якої брюнети,

має світле волосся, світлу

шкіру і блакитні очі. Зовні

при народженні не було

виявлено ніякої патології,

але упродовж останніх 3

місяців спостерігалися

порушення мозкового

кровообігу, відставання в

розумовому розвитку.

Причиною такого стану є:

А. Гістидинемія.

В. Галактоземія.

С. Глікогеноз.

D. Гостра порфірія.

Е. *Фенілкетонурія.

22. У хворої 63 років

внаслідок крововиливу в

шлунково-кишковий

тракт білки крові

виявилися доступними для

дії мікроорганізмів ки-

шечника, тобто піддалися

гниттю. Концентрація якої

речовини збільшилася у

крові хворої ?

A. Триптофан.

B. Креатин.

C. Ціанокобаламін.

D. Тіамін.

E. *Індол.

Page 49: КРОК-1 БІОХІМІЯ Збірник тестових завданьlib.sumdu.edu.ua/library/docs/rio/2020/m4695.pdf · 2 Крок-1. Біохімія Н: збірник тестових

49

23. При декарбоксилю-

ванні амінокислоти

гістидину утворюється

надзвичайно активний

амін – медіатор запалення

та алергії, а саме:

А. Серотонін.

В. Триптамін.

С. Дофамін.

D. γ-Аміномасляна кислота.

Е. *Гістамін.

24. Одна з форм спадкової

патології супровод-

жується гальмуванням

перетворення фенілалані-

ну на тирозин. Біохімічною

ознакою хворого є

накопичення в організмі

деяких органічних кислот,

у тому числі:

А. Глутамінової.

В. Цитратної.

С. Піровиноградної.

D. Молочної.

Е. *Фенілпіровиноградної.

25. У юнака 19 років ви-

ражені ознаки депігмен-

тації шкіри, що зумовлено

порушенням синтезу

меланіну. Порушенням

обміну якої амінокислоти

це спричинено ?

А. Гістидину.

В. Триптофану.

С. Гліцину.

D. Проліну.

Е. *Тирозину.

26. Під дією опромінення

ультрафіолетовими про-

менями в людини

темнішає шкіра, що є

захисною реакцією

організму. Яка захисна

речовина – похідне

амінокислот – синтезує-

ться в клітинах під

впливом ультрафіолету ?

А. Аргінін.

В. Тироксин.

С. Метіонін.

D. Фенілаланін.

Е. *Меланін.

27. При повторній дії

ультрафіолетових проме-

нів шкіра темнішає

внаслідок синтезу в ній

меланіну, що захищає

клітини від пошкодження.

Основним механізмом

залучення цього захисту є:

А. Пригнічення тирозинази.

В. Пригнічення фенілала-

нінгідроксилази.

С. Активація оксидази

гомогентизинової кислоти.

Page 50: КРОК-1 БІОХІМІЯ Збірник тестових завданьlib.sumdu.edu.ua/library/docs/rio/2020/m4695.pdf · 2 Крок-1. Біохімія Н: збірник тестових

50

D. Пригнічення оксидази

гомогентизинової кислоти.

Е. *Активація тирозинази.

28. Аміак особливо токсич-

ний для ЦНС людини. Наз-

віть головний шлях його

знешкодження у нервовій

тканині:

A. Утворення парних

сполук.

B. Синтез солей амонію.

C. Синтез сечовини.

D. Трансамінування.

E. *Синтез глутаміну.

29. У 12-річного хлопчика

в сечі виявлено високий

вміст усіх амінокислот

аліфатичного ряду. При

цьому відмічена найбільш

висока екскреція цистину

та цистеїну. Крім того,

УЗД нирок показало наяв-

ність каменів у них. Вибе-

ріть можливу патологію:

A. Хвороба Хартнупа.

B. Алкаптонурія.

C. Цистит.

D. Фенілкетонурія.

E. *Цистинурія.

30. До лікаря звернувся

пацієнт зі скаргами на

непереносимість сонячної

радіації. З’являються

опіки шкіри і порушення

зору. Попередній діагноз -

альбінізм. Порушення

обміну якої амінокислоти

відмічається у цього

пацієнта ?

A. Триптофану.

B. Проліну.

C. Лізину.

D. Аланіну.

E. *Тирозину.

31. Основна маса азоту з

організму виводиться у

вигляді сечовини. Знижен-

ня активності якого

ферменту в печінці

призводить до гальму-

вання синтезу сечовини і

накопичення аміаку в

крові і тканинах ?

A. Пепсину.

B. Аспартатаміно-

трансферази.

C. Уреази.

D. Амілази.

E. *Карбамоїлфосфат-

синтетази.

32. У чоловіка 32 років

діагностована гостра

променева хвороба.

Лабораторно встановлено

різке зниження рівня

Page 51: КРОК-1 БІОХІМІЯ Збірник тестових завданьlib.sumdu.edu.ua/library/docs/rio/2020/m4695.pdf · 2 Крок-1. Біохімія Н: збірник тестових

51

серотоніну в тромбоцитах.

Найбільш вірогідною

причиною зниження

тромбоцитарного серотоні-

ну є порушення процесу

декарбоксилювання:

A. Гістидину.

B. Серину.

C. Тирозину.

D. Піровиноградної кислоти.

E. *5-Окситриптофану.

33. Кухар у результаті

необережності обпік руку

парою. Підвищення кон-

центрації якої речовини

викликало почервоніння,

набряклість, болючість

ураженої ділянки ?

A. Галактозаміну.

B. Тіаміну.

C. Глутаміну.

D. Лізину.

E. *Гістаміну.

34. У дитини 3 років після

перенесеної тяжкої

вірусної інфекції відмічаю-

ться часте блювання,

втрата свідомості, судоми.

При дослідженні крові

виявлена гіперамоніємія. З

чим може бути пов’язана

зміна біохімічних показ-

ників крові у цієї дитини ?

A. Із пригніченням

активності ферментів

трансамінування.

B. Із активацією процесів де-

карбоксилювання амінокис-

лот.

C. Із порушенням знешкод-

ження біогенних амінів.

D. Із посиленням гниття

білків у кишечнику.

E. *Із порушенням знешкод-

ження аміаку в

орнітиновому циклі.

35. Причиною захво-

рювання на пелагру може

бути переважне харчу-

вання кукурудзою і

зниження в раціоні

продуктів тваринного

походження. Відсутність у

раціоні якої амінокислоти

призводить до цієї

патології ?

A. Гістидину.

B. Ізолейцину.

C. Фенілаланіну.

D. Метіоніну.

E. *Триптофану.

36. У дитини упродовж

перших трьох місяців

після народження розвину-

лася тяжка форма гіпоксії

(задуха, синюшність).

Page 52: КРОК-1 БІОХІМІЯ Збірник тестових завданьlib.sumdu.edu.ua/library/docs/rio/2020/m4695.pdf · 2 Крок-1. Біохімія Н: збірник тестових

52

Який із процесів гемо-

глобіноутворення зазнав

порушення ?

A. Заміна фетгемоглобіну на

гемоглобін М.

B. Заміна фетгемоглобіну на

гемоглобін S.

C. Заміна фетгемоглобіну на

глікозильований гемоглобін.

D. Заміна фетгемоглобіну на

метгемоглобін.

E. *Заміна фетгемоглобіну

на гемоглобін А.

37. При інтенсивній роботі

в м’язах утворюється

значна кількість аміаку.

Яка амінокислота відіграє

основну роль у транс-

портуванні аміаку в

печінку та використовує-

ться в синтезі глюкози ?

A. Аспартат.

B. Аргінін.

C. Лізин.

D. Орнітин.

E. *Аланін.

38. Центральну роль в

обміні амінокислот у

нервовій тканині відіграє

глутамінова кислота. Це

пов’язано з тим, що вона:

A. Використовується для

синтезу кетонових тіл.

B. Використовується для

синтезу ліків.

C. Використовується для

синтезу глюкози.

D. Використовується для

синтезу нейроспецифічних

білків.

E. *Зв’язує аміак з утворен-

ням глутаміну.

39. Травма мозку

спричинила збільшення

утворення аміаку. Яка

амінокислота бере участь у

видаленні аміаку з цієї

тканини ?

A. Лізин.

B. Тирозин.

C. Валін.

D. Триптофан.

E. *Глутамінова кислота.

40. При декарбокси-

люванні глутамату в ЦНС

утворюється медіатор

гальмування. Назвіть

його:

A. Аспарагін.

B. Глутатіон.

C. Гістамін.

D. Серотонін.

E. *ГАМК.

41. При катаболізмі

гістидину утворюється

Page 53: КРОК-1 БІОХІМІЯ Збірник тестових завданьlib.sumdu.edu.ua/library/docs/rio/2020/m4695.pdf · 2 Крок-1. Біохімія Н: збірник тестових

53

біогенний амін, що має

сильну судино-

розширювальну дію.

Назвіть його:

A. Дофамін.

B. Серотонін.

C. ДОФА.

D. Норадреналін.

E. *Гістамін.

42. У хворого діагносто-

вана алкаптонурія.

Назвіть фермент, дефект

якого є причиною цієї

патології:

A. ДОФА-декарбоксилаза.

B. Фенілаланінгідроксилаза.

C. Глутаматдегідрогеназа.

D. Піруватдегідрогеназа.

E. *Оксидаза гомогентизи-

нової кислоти.

43. У клініку госпіта-

лізовано хворого з

діагнозом карциноїд

кишечника. Аналіз вия-

вив підвищену продукцію

серотоніну. Відомо, що ця

речовина утворюється з

амінокислоти триптофану.

Який біохімічний механізм

лежить в основі цього

процесу ?

A. Утворення парних

сполук.

B. Дезамінування.

C. Мікросомальне

окиснення.

D. Трансамінування.

E. *Декарбоксилювання.

44. За клінічними показни-

ками хворому призначено

піридоксальфосфат. Для

корекції яких процесів

рекомендований цей

препарат ?

A. Синтезу білка.

B. Окисного декарбоксилю-

вання кетокислот.

C. Дезамінування пуринових

нуклеотидів.

D. Синтезу пуринових і

піримідинових основ.

E. *Трансамінування і

декарбоксилювання

амінокислот.

45. У дитини в крові підви-

щена кількість феніл-

піровиноградної кислоти.

Який вид лікування

потрібен при феніл-

кетонемії ?

A. Гормонотерапія.

B. Вітамінотерапія.

C. Ферментотерапія.

D. Антибактеріальна

терапія.

E. *Дієтотерапія.

Page 54: КРОК-1 БІОХІМІЯ Збірник тестових завданьlib.sumdu.edu.ua/library/docs/rio/2020/m4695.pdf · 2 Крок-1. Біохімія Н: збірник тестових

54

46. У хворого діагностова-

но мієломну хворобу.

Загальний білок крові -

180 г/л. Гіперпротеїнемія є

наслідком підвищення

вмісту в крові:

A. Трансферину.

B. Альбумінів.

C. Гаптоглобіну.

D. Імуноглобулінів.

E. *Білка Бенс-Джонса.

47. У експериментальної

тварини, яка була на

безбілковому раціоні,

розвинулася жирова

інфільтрація печінки

внаслідок дефіциту мети-

люючих агентів. Назвіть

метаболіт, синтез якого

порушений:

A. Лінолева кислота.

B. ДОФА.

C. Холестерин.

D. Ацетоацетат.

E. *Холін.

48. У новонародженого на

пелюшках виявлені темні

плями, які свідчать про

утворення гомогентизи-

нової кислоти. З

порушенням обміну якої

речовини це пов’язано ?

A. Триптофану.

B. Галактози.

C. Метіоніну.

D. Холестерину.

E. *Тирозину.

49. У немовляти спосте-

рігаються судоми,

спричинені дефіцитом

вітаміну В6. Це зумовлено

зниженням вмісту у

нервовій тканині

гальмівного медіатора -

γ-аміномасляної кислоти.

Активність якого

ферменту знижена ?

A. Глутаматсинтетази.

B. Аланінамінотрансферази.

C. Глутаматдегідрогенази.

D. Піридоксалькінази.

E.*Глутаматдекарбоксилази.

50. Дитина, хвора на

фенілкетонурію, має

розумову відсталість. Який

механізм буде головним у

розвитку пошкодження

центральної нервової

системи ?

А. Збільшення екскреції з

сечею фенілкетонових тіл.

В. Підвищення синтезу

тирозину.

С. Зниження синтезу

тиреоїдних гормонів.

Page 55: КРОК-1 БІОХІМІЯ Збірник тестових завданьlib.sumdu.edu.ua/library/docs/rio/2020/m4695.pdf · 2 Крок-1. Біохімія Н: збірник тестових

55

D. Зниження синтезу

меланіну.

Е. *Накопичення в крові

фенілаланіну і фенілкетонів.

51. У дитини 6 місяців

встановлена затримка

моторного і психо-

моторного розвитку,

посвітління шкіри,

волосся, райдужної обо-

лонки очей, позитивна

проба Фелінга. Яке

спадкове захворювання

виявлено у дитини ?

А. Галактоземія.

В. Алкаптонурія.

С. Хвороба Дауна.

D. Альбінізм.

Е. *Фенілкетонурія.

52. При обстеженні

чоловіка 45 років, що

перебував тривалий час на

рослинній дієті, виявили

негативний азотистий

баланс. Яка особливість

раціону стала причиною

цього явища ?

А. Недостатня кількість

жирів.

В. Надмірна кількість води.

С. Недостатня кількість

жирів і білків.

D. Надмірна кількість вугле-

водів.

Е. *Недостатня кількість

білків.

53. При дії ультрафіоле-

тового випромінювання у

людини темнішає шкіра,

що є захисною реакцією.

Яка захисна речовина

синтезується в клітинах

під впливом зазначеного

фактора ?

А. Триптофан.

В. ДНК.

С. Аргінін.

D. Вітамін D.

Е. *Меланін.

54. У дитини з розумовою

відсталістю виявлене зеле-

не забарвлення сечі після

додавання 5 % розчину

FeCl3. Порушення обміну

якої амінокислоти під-

тверджує позитивний ре-

зультат цієї діагностичної

проби ?

А. Аргініну.

В. Тирозину.

С. Глутаміну.

D. Триптофану.

Е. *Фенілаланіну.

Page 56: КРОК-1 БІОХІМІЯ Збірник тестових завданьlib.sumdu.edu.ua/library/docs/rio/2020/m4695.pdf · 2 Крок-1. Біохімія Н: збірник тестових

56

55. У хворого з діагнозом

“злоякісний карциноїд”

різко підвищений вміст

серотоніну в крові. Із якої

амінокислоти може утво-

рюватися цей біогенний

амін ?

А. Лейцину.

В. Метіоніну.

С. Треоніну.

D. Аланіну.

Е. *Триптофану.

1.7. ОБМІН

НУКЛЕОТИДІВ ТА

НУКЛЕЇНОВИХ КИСЛОТ

1. Уотсон і Крік

установили, що подвійна

спіраль ДНК стабілізує-

ться за рахунок зв’язків

між комплементарними

азотистими основами. Які

це зв’язки ?

A. Складно-ефірні.

B. N- глікозидні.

C. Фосфодиефірні.

D. Пептидні.

E. *Водневі.

2. На судово-медичну екс-

пертизу надійшла кров

дитини та чоловіка для

встановлення батьківства.

Які хімічні компоненти

необхідно ідентифікувати в

дослідній крові ?

А. мяРНК.

В. тРНК.

С. мРНК.

D. рРНК.

Е. *ДНК.

3. Антибіотик рифампіцин,

який використовується

для лікування туберкульо-

зу, впливає на певні

біохімічні процеси. Назвіть

їх:

А. Інгібує дію білкових

факторів у синтезі білка.

В. Інгібує ДНК-полімеразу

на стадії ініціації.

С. Інгібує ДНК-лігазу.

D. Інгібує аміноацил-тРНК-

синтетазу.

Е. *Інгібує РНК-полімеразу

на стадії ініціації.

4. У хворого 50 років

діагностовано подагру, а в

крові виявлено гіпер-

урикемію. Обмін яких

речовин порушений ?

А. Піримідину.

В. Жирів.

С. Амінокислот.

D. Вуглеводів.

Е. *Пуринів.

Page 57: КРОК-1 БІОХІМІЯ Збірник тестових завданьlib.sumdu.edu.ua/library/docs/rio/2020/m4695.pdf · 2 Крок-1. Біохімія Н: збірник тестових

57

5. У клініку прийнятий

хворий із підозрою на

подагру. Який біохімічний

аналіз слід провести для

уточнення діагнозу ?

А. Визначення вмісту амі-

нокислот у крові.

В. Визначення концентрації

сечовини в крові та сечі.

С. Визначення рівня кре-

атину в крові.

D. Визначення активності

уреази в крові.

Е. *Визначення рівня

сечової кислоти в крові та

сечі.

6. Для лікування урогені-

тальних інфекцій

використовують хінолони

– інгібітори ферменту

ДНК-гірази. Який процес

порушується під дією

хінолонів у першу чергу ?

А. Зворотна транскрипція.

В. Репарація.

С. Ампліфікація генів.

D. Рекомбінація генів.

Е. *Реплікація.

7. Із нітратів, нітритів і

нітрозамінів в організмі

утворюється нітратна кис-

лота, яка зумовлює окисне

дезамінування азотистих

основ нуклеотидів. Це

може призвести до

точкової мутації – заміни

цитозину на:

А. Тимін.

В. Гуанін.

С. Інозин.

D. Аденін.

Е. *Урацил.

8. У хворих із пігментною

ксеродермою шкіра

надзвичайно чутлива до

сонячного світла, може

розвиватися рак шкіри.

Причиною є спадкова

недостатність ферменту

УФ-ендонуклеази. Вна-

слідок цього дефекту

порушується процес:

А. Трансляції.

В. Реплікації ДНК.

С. Транскрипції.

D. Зворотної транскрипції.

Е. *Репарації ДНК.

9. У крові 12-річного

хлопчика – зниження

концентрації сечової

кислоти і накопичення

ксантину та гіпоксантину.

Про генетичний дефект

якого ферменту це

свідчить ?

А. Аргінази.

Page 58: КРОК-1 БІОХІМІЯ Збірник тестових завданьlib.sumdu.edu.ua/library/docs/rio/2020/m4695.pdf · 2 Крок-1. Біохімія Н: збірник тестових

58

В. Гліцеролкінази.

С. Уреази.

D. Орнітин-карбамоїлтранс-

ферази.

Е. *Ксантиноксидази.

10. Хворий 46 років зве-

рнувся до лікаря зі

скаргою на біль у суглобах,

який посилюється на зміну

погоди. У крові –

підвищення концентрації

сечової кислоти.

Посилений розпад якої

речовини є причиною цих

змін ?

А. УТФ.

В. ЦМФ.

С. ТМФ.

D. УМФ.

Е. *АМФ.

11. У хлопчика 8 років

хвороба Леша-Ніхана. У

крові – збільшена кон-

центрація сечової кислоти.

Порушення якого процесу

є причиною цього спадко-

вого захворювання ?

А.Утворення

дезоксирибонуклеотидів.

В. Синтезу пуринових

нуклеотидів.

С. Синтезу піримідинових

нуклеотидів.

D. Розпаду піримідинових

нуклеотидів.

Е. *Розпаду пуринових

нуклеотидів.

12. Чоловік 58 років пе-

реніс операцію з приводу

раку передміхурової

залози. Через 3 місяці йому

провели курс променевої

та хіміотерапії. До

комплексу лікарських

препаратів входив 5-фтор-

дезоксиуридин – інгібітор

тимідилатсинтази. Синтез

якої речовини в першу

чергу блокується під дією

цього препарату ?

А. Білка.

В. іРНК.

С. рРНК.

D. тРНК.

Е. *ДНК.

13. Чоловік 65 років, який

страждає на подагру, скар-

житься на біль у ділянці

нирок. При ультра-

звуковому обстеженні

виявлені камені у нирках.

Підвищення концентрації

якої речовини є найбільш

імовірною причиною

утворення каменів у

Page 59: КРОК-1 БІОХІМІЯ Збірник тестових завданьlib.sumdu.edu.ua/library/docs/rio/2020/m4695.pdf · 2 Крок-1. Біохімія Н: збірник тестових

59

нирках ?

А. Цистину.

В. Холестерину.

С. Білірубіну.

D. Сечовини.

Е. *Сечової кислоти.

14. Біосинтез пуринового

кільця відбувається за

рахунок рибозо-5-фосфату

шляхом поступового наро-

щення атомів нітрогену і

карбону та замикання

кілець. Джерелом рибозо-

5-фосфату є:

А. Глікогеноліз.

В. Гліколіз.

С. Глікогенез.

D. Глюконеогенез.

Е. *Пентозофосфатний цикл.

15. Хворому на подагру

лікар призначив

алопуринол. Яка фарма-

кологічна властивість

алопуринолу забезпечує

терапевтичний ефект у

цьому випадку ?

А. Прискорення катаболізму

піримідинових нуклеотидів.

В. Збільшення швидкості

виведення азотовмісних

речовин.

С. Прискорення синтезу

нуклеїнових кислот.

D. Уповільнення реути-

лізації піримідинових

нуклеотидів.

Е. *Конкурентне інгібування

ксантиноксидази.

16. Для утворення транс-

портної форми

амінокислоти в ході

синтезу білка на рибосомах

необхідна:

А. мРНК.

В. ГТФ.

С. Ревертаза.

D. Рибосома.

Е. *Аміноацил-тРНК-

синтетаза.

17. При отруєнні аманіти-

ном – токсином блідої

поганки – блокується

РНК-полімераза (ІІ). При

цьому припиняється:

А. Дозрівання мРНК.

В. Синтез тРНК.

С. Зворотна транскрипція.

D. Синтез праймерів.

Е. *Синтез мРНК.

18. Виродженість гене-

тичного коду – здатність

декількох триплетів

кодувати одну аміно-

кислоту. А яка

Page 60: КРОК-1 БІОХІМІЯ Збірник тестових завданьlib.sumdu.edu.ua/library/docs/rio/2020/m4695.pdf · 2 Крок-1. Біохімія Н: збірник тестових

60

амінокислота кодується

одним триплетом ?

А. Лейцин.

В. Серин.

С. Аланін.

D. Лізин.

Е. *Метіонін.

19. Генетичний апарат

людини містить близько

30 тис. генів, а кількість

варіантів антитіл досягає

мільйонів. Який механізм

використовується для

утворення нових генів, що

відповідають за синтез

такої кількості антитіл ?

А. Утворення фрагментів

Оказакі.

В. Ампліфікація генів.

С. Реплікація ДНК.

D. Репарація ДНК.

Е. *Рекомбінація генів.

20. Локалізована в цито-

плазмі карбамоїлфосфат-

синтетаза ІІ каталізує

реакцію утворення

карбамоїлфосфату не з

вільного аміаку, а з

глутаміну. Цей фермент

постачає карбамоїлфосфат

для синтезу ?

A. Амінокислот.

B. Пуринів.

C. Сечовини.

D. Ліпідів.

E. *Піримідинів.

21. Для лікування злоякіс-

них пухлин призначають

метотрексат – структур-

ний аналог фолієвої

кислоти, який є

конкурентним інгібітором

дигідрофолатредуктази і

тому пригнічує синтез ?

A. Глікогену.

B. Моносахаридів.

C. Жирних кислот.

D. Гліцерофосфатидів.

E. *Нуклеотидів.

22. Жінка 43 років,

робітниця лакофарбного

підприємства, скаржиться

на загальну слабість,

зниження ваги, апатію,

сонливість. Хронічна

свинцева інтоксикація

підтверджена лабораторно

– виявлена гіпохромна

анемія. У крові:

підвищення вмісту прото-

порфірину і зниження

рівня дельтаамінолевулі-

нової кислоти, що свідчить

про порушення синтезу:

A. Мевалонової кислоти.

B. ДНК.

Page 61: КРОК-1 БІОХІМІЯ Збірник тестових завданьlib.sumdu.edu.ua/library/docs/rio/2020/m4695.pdf · 2 Крок-1. Біохімія Н: збірник тестових

61

C. РНК.

D. Білка.

E. *Гему.

23. У хворого в крові збіль-

шений вміст сечової

кислоти, що клінічно

виявляється больовим

синдромом унаслідок

відкладання уратів у

суглобах. У результаті

якого процесу утворюється

ця кислота:

A. Реутилізації пуринових

основ.

B. Розпаду піримідинових

нуклеотидів.

C. Катаболізму гему.

D. Розщеплення білків.

E. *Розпаду пуринових

нуклеотидів.

24. У всіх живих організмів

одні й ті самі триплети

кодують одні й ті самі

амінокислоти, що дозволяє

перенести E.Coli ген

інсуліну людини. Як

називається ця влас-

тивість генетичного коду ?

A. Неперервність.

B. Виродженість.

C. Надлишковість.

D. Триплетність.

E. *Універсальність.

25. При спадковій орот-

ацидурії виділення

оротової кислоти в багато

разів перевищує норму.

Синтез яких речовин буде

порушений при цій

патології ?

A. Сечовини.

B. Пуринових нуклеотидів.

C. Біогенних амінів.

D. Сечової кислоти.

E. *Піримідинових нуклео-

тидів.

26. Жінка 40 років

потрапила до лікарні в

тяжкому стані з

симптомами отруєння

Amanita phalloides (блідою

поганкою). Відомо, що

один із токсинів цих грибів

блокує синтез

попередників мРНК. Цим

токсином є:

A. Бікулін.

B. Актиноміцин.

C. Таурин.

D. Рибофорин.

E. *Аманітин.

27. Похідні птерину –

аміноптерин і метотрексат

– є конкурентними

інгібіторами дигідрофолат-

редуктази, унаслідок чого

Page 62: КРОК-1 БІОХІМІЯ Збірник тестових завданьlib.sumdu.edu.ua/library/docs/rio/2020/m4695.pdf · 2 Крок-1. Біохімія Н: збірник тестових

62

вони пригнічують регене-

рацію тетрагідрофолієвої

кислоти з дигідрофолату.

Ці лікарські засоби

призводять до гальму-

вання міжмолекулярного

транспорту одновуглеце-

вих груп. Біосинтез якого

нуклеотиду при цьому

пригнічується ?

A. АМФ.

B. ІМФ.

C. УМФ.

D. ОМФ.

E. *дТМФ.

28. У пацієнта візуально

виявлені пухирі та

посилена пігментація

після дії УФ-променів.

Сеча після стояння

набуває червоного

кольору. Виявлення у сечі

якого з перелічених показ-

ників дозволить діагносту-

вати хворобу Гюнтера ?

A. Ацетон.

B. Гемоглобін.

C. Білірубін.

D. Креатинін.

E. *Уропорфіриноген І.

29. У крові 12-річного

хлопчика виявлено

зниження концентрації

сечової кислоти і

накопичення ксантину та

гіпоксантину. Генетичний

дефект якого ферменту

має місце в організмі

дитини ?

A. Гліцеролкінази.

B. Аргінази.

C. Уреази.

D. Орнітинкарбамоїл-

трансферази.

E. *Ксантиноксидази.

30. В експерименті було

показано, що опромінені

ультрафіолетом клітини

шкіри хворих пігментною

ксеродермою повільніше

відновлюють нативну

структуру ДНК, ніж

клітини нормальних

людей через дефект

ферменту репарації.

Виберіть фермент цього

процесу:

A. ДНК-гіраза.

B. РНК-лігаза.

C. Праймаза.

D. ДНК-полімераза ІІІ.

E. *Ендонуклеаза.

31. У дитини спостері-

гається відставання у

фізичному і розумовому

розвитку, з сечею

Page 63: КРОК-1 БІОХІМІЯ Збірник тестових завданьlib.sumdu.edu.ua/library/docs/rio/2020/m4695.pdf · 2 Крок-1. Біохімія Н: збірник тестових

63

виділяється велика

кількість оротової

кислоти. Ця спадкова

хвороба розвивається

внаслідок порушень:

A. Перетворення рибо-

нуклеотидів у дезоксирибо-

нуклеотиди.

B. Розпаду піримідинових

нуклеотидів.

C. Синтезу пуринових

нуклеотидів.

D. Розпаду пуринових

нуклеотидів.

E. *Синтезу піримідинових

нуклеотидів.

32. У хворого 50 років

діагностовано подагру,

виявлено гіперурикемію.

Обмін яких речовин

порушений ?

A. Піримідину.

B. Жирів.

C. Амінокислот.

D. Вуглеводів.

E. * Пуринів.

33. РНК вірусу СНІДу

проникла всередину

лейкоциту і за допомогою

ферменту ревертази

примусила клітину

синтезувати вірусну ДНК.

Цей процес має назву:

А. Депресія оперону.

В. Конваріантна реплікація.

С. Зворотна трансляція.

D. Репресія оперону.

Е. *Зворотна транскрипція.

34. Жінка 40 років зверну-

лася до лікаря зі скаргами

на біль у дрібних суглобах

ніг і рук. Суглоби збіль-

шені, мають вигляд

потовщених вузлів. У

сироватці крові виявлено

підвищений рівень уратів.

Причиною такого стану є

порушення обміну:

А. Амінокислот.

В. Ліпідів.

С. Вуглеводів.

D. Піримідинів.

Е. *Пуринів.

35. У процесі транскрипції

здійснюється синтез комп-

лементарної молекули

РНК на матриці ДНК.

Який фермент каталізує

цей процес ?

А. Хеліказа.

В. Праймаза.

С. ДНК-полімераза.

D. Топоізомераза.

Е. *ДНК-залежна РНК-

полімераза.

Page 64: КРОК-1 БІОХІМІЯ Збірник тестових завданьlib.sumdu.edu.ua/library/docs/rio/2020/m4695.pdf · 2 Крок-1. Біохімія Н: збірник тестових

64

36. У хворого на

сечокам’яну хворобу в

крові і сечі виявлено

підвищений вміст сечової

кислоти. Реакція сечі

виявилася сильно кислою.

Наявність якого типу

каменів можна очікувати у

пацієнта ?

А. Фосфатні.

В. Оксалатні.

С. Холестеринові.

D. Кальцієві.

Е. *Уратні.

37. Згідно з моделлю

подвійної спіралі ДНК,

запропонованою Уотсоном

і Кріком, було

встановлено, що один із

ланцюгів зберігається при

реплікації, а інший синте-

зується комплементарно

першому. Як називається

цей спосіб реплікації ?

А. Ідентичний.

В. Дисперсний.

С. Аналогічний.

D. Консервативний.

Е. *Напівконсервативний.

38. У чоловіка 42 років,

який страждає на подагру,

в крові підвищена кон-

центрація сечової кисло-

ти. Для зниження рівня

сечової кислоти йому

призначили алопуринол.

Конкурентним інгібітором

якого ферменту є

алопуринол ?

А. Гіпоксантин-фосфо-

рибозилтрансферази.

В. Аденозиндезамінази.

С. Аденінфосфорибозил-

трансферази.

D. Гуаніндезамінази.

Е. *Ксантиноксидази.

39. При отруєнні аманіти-

ном (отрутою блідої поган-

ки) блокується РНК-

полімераза ІІ. При цьому

припиняється:

А. Синтез т-РНК.

В. Синтез праймерів.

С. Дозрівання м-РНК.

D. Зворотна транскрипція.

Е. *Синтез м-РНК.

40. У хворого виявлений

підвищений вміст сечової

кислоти в крові та

відкладання уратів у

суглобах. У результаті

якого процесу утворюється

сечова кислота ?

А. Розщеплення білків.

В. Катаболізм гему.

Page 65: КРОК-1 БІОХІМІЯ Збірник тестових завданьlib.sumdu.edu.ua/library/docs/rio/2020/m4695.pdf · 2 Крок-1. Біохімія Н: збірник тестових

65

С. Розпад піримідинових

нуклеотидів.

D. Реутилізація пуринових

основ.

Е. *Розпад пуринових

нуклеотидів.

41. Хворому на сечо-

кам’яну хворобу після

обстеження призначили

алопуринол – конкурент-

ний інгібітор ксантин-

оксидази. Підставою для

цього був хімічний аналіз

ниркових каменів, які

складалися переважно з:

А. Фосфату кальцію.

В. Дигідрату оксалату

кальцію.

С. Сульфату кальцію.

D. Моногідрату оксалату

кальцію.

Е. *Урату натрію.

42. Для вивчення

локалізації біосинтезу

білка в клітинах миші

ввели мічені амінокислоти

- аланін та триптофан.

Біля яких органел буде

спостерігатися накопичен-

ня мічених амінокислот ?

А. Лізосоми.

В. Апарат Гольджі.

С. Гладенька ЕПС.

D. Клітинний центр.

Е. *Рибосоми.

43. Унаслідок впливу

γ-випромінювання ділянка

ланцюга ДНК повернулася

на 180 градусів. Яка з

перелічених видів мутацій

відбулася в ланцюзі ДНК ?

А. Транслокація.

В. Дуплікація.

С. Реплікація.

D. Делеція.

Е. *Інверсія.

44. У дитини 5-ти років

підвишена концентрація

сечової кислоти в крові,

спостерігаються психічні

розлади, схильність до

пошкодження власного

тіла. Попередній діагноз –

Синдром Леша-Ніхана.

Спадковий дефіцит якого

ферменту є причиною

розвитку даного

захворювання ?

А. Ксантиноксидаза.

В. Аденозиндезаміназа.

С. Гуанозиндезаміназа.

D. ДНК-лігаза.

Е. *Гіпоксантин-гуанін-

фосфорибозилтрансфераза.

Page 66: КРОК-1 БІОХІМІЯ Збірник тестових завданьlib.sumdu.edu.ua/library/docs/rio/2020/m4695.pdf · 2 Крок-1. Біохімія Н: збірник тестових

66

1.8. ГОРМОНИ

1. У пацієнта, який

проживає на специфічній

геохімічній території,

встановлено діагноз -

ендемічний зоб. Який вид

посттрансляційної моди-

фікації тиреоглобуліну

порушений в організмі

хворого ?

А. Глікозилювання.

В. Метилювання.

С. Ацетилювання.

D. Фосфорилювання.

Е. *Йодування.

2. Хворий перебуває в

стані гіпоглікемічної коми.

Передозування якого гор-

мону може призвести до

такої ситуації:

А. Кортикотропіну.

В. Прогестерону.

С. Кортизолу.

D. Соматотропіну.

Е. *Інсуліну.

3. Яка сполука є поперед-

ником у синтезі проста-

гландинів в організмі

людини ?

А. Лінолева кислота.

В. Пальмітинова кислота.

С. Ліноленова кислота.

D. Олеїнова кислота.

Е. *Арахідонова кислота.

4. Хворий звернувся до

лікаря зі скаргами на

часте та надмірне

сечовипускання, спрагу.

При аналізі сечі

встановлено: добовий

діурез – 19 л, густина сечі –

1,001. Для якого

захворювання характерні

ці показники ?

А. Цукровий діабет.

В. Стероїдний діабет.

С. Хвороба Аддісона.

D. Тиреотоксикоз.

Е. *Нецукровий діабет.

5. Під час операції на

щитовидній залозі з

приводу захворювання на

дифузний токсичний зоб

помилково були видалені

паращитовидні залози.

Виникли судоми, тетанія.

Обмін якого біоелемента

було порушено ?

А. Магнію.

В. Натрію.

С. Калію.

D. Заліза.

Е. *Кальцію.

Page 67: КРОК-1 БІОХІМІЯ Збірник тестових завданьlib.sumdu.edu.ua/library/docs/rio/2020/m4695.pdf · 2 Крок-1. Біохімія Н: збірник тестових

67

6. Порушення функції

панкреатичних острівців

Лангерганса призводить

до зниження продукції:

А. Паратгормону та

кортизону.

В. Тироксину та

кальцитоніну.

С. Інсуліну та адреналіну.

D. Калікреїнів та ангіо-

тензину.

Е. *Глюкагону та інсуліну.

7. До лікаря звернувся

хворий зі скаргами на

постійну спрагу.

Встановлено гіпер-

глікемію, поліурію та

підвищений вміст 17-кето-

стероїдів у сечі. Яке

захворювання імовірне ?

А. Аддісонова хвороба.

В. Інсулінозалежний діабет.

С. Мікседема.

D. Глікогеноз І типу.

Е. *Стероїдний діабет.

8. У хворого спостеріга-

ється різке схуднення,

підвищена дратівливість,

незначне підвищення

температури тіла,

екзофтальм, гіперглікемія,

азотемія. Яке це

захворювання ?

А. Невроз.

В. Бронзова хвороба.

С. Мікседема.

D. Туберкульоз надниркових

залоз.

Е. *Дифузний токсичний

зоб.

9. З метою аналгезії може

бути використана

речовина, що імітує

ефекти морфіну, але

виробляється в ЦНС.

Назвіть її:

А. Соматоліберин.

В. Окситоцин.

С. Вазопресин.

D. Кальцитонін.

Е. *Ендорфін.

10. У хворого вміст іонів

калію в плазмі крові

становить 7 ммоль/л. Які

можливі причини такого

стану ?

А. Підвищення рівня

статевих гормонів.

В. Збільшення вмісту

альдостерону.

С. Зменшення вмісту

тиреоїдних гормонів.

D. Підвищення рівня

тиреоїдних гормонів.

Е. *Зменшення концентрації

альдостерону.

Page 68: КРОК-1 БІОХІМІЯ Збірник тестових завданьlib.sumdu.edu.ua/library/docs/rio/2020/m4695.pdf · 2 Крок-1. Біохімія Н: збірник тестових

68

11. У хворого вміст іонів

натрію в плазмі крові

становить 160 ммоль/л.

Зміна вмісту якого

гормону може призвести

до такого стану ?

А. Натрійуретичного гормо-

ну (збільшення).

В. Альдостерону (зменше-

ння).

С. Глюкокортикоїдів (збіль-

шення).

D. Тиреоїдних гормонів

(збільшення).

Е. *Альдостерону

(збільшення).

12. Характерна ознака

холери – втрата

організмом великої

кількості води та іонів

натрію. Основою

біохімічної дії холерного

токсину є:

А. Активація синтезу пе-

редсердного натрійуретич-

ного фактора.

В. Окиснення альдостерону

в кірковій речовині

надниркових залоз.

С. Гальмування синтезу

антидіуретичного гормону в

гіпоталамусі.

D. Посилення секреції

реніну клітинами

юкстагломерулярного

апарату ниркових артеріол.

Е. *Активація аденілат-

циклази тонкої кишки.

13. Хворий 23 років

скаржиться на головний

біль, зміну зовнішнього

вигляду (збільшення

розмірів ніг, кистей, рис

обличчя), огрубіння

голосу, погіршення

пам’яті. Захворювання

почалося приблизно 3

роки тому без видимих

причин. Об’єктивно:

збільшення надбрівних

дуг, носа, язика. Аналіз

сечі без особливих змін.

Причиною такого стану

може бути:

А. Дефіцит альдостерону.

В. Дефіцит глюкагону.

С. Дефіцит тироксину.

D. Гіперпродукція корти-

костероїдів.

Е. *Гіперпродукція сомато-

тропіну.

14. Продуктами гідролізу

та модифікації деяких

білків є біологічно активні

речовини – гормони. З

якого з наведених білків у

гіпофізі утворюються

Page 69: КРОК-1 БІОХІМІЯ Збірник тестових завданьlib.sumdu.edu.ua/library/docs/rio/2020/m4695.pdf · 2 Крок-1. Біохімія Н: збірник тестових

69

ліпотропін, кортикотропін,

меланотропін та ендорфі-

ни ?

А. Тиреоглобулін.

В. Нейроальбумін.

С. Нейростромін.

D. Нейроглобулін.

Е. *Проопіомеланокортин

(ПОМК).

15. Іони Са2+ - одні із

еволюційно найдавніших

вторинних месенджерів у

клітинах. Вони є акти-

ваторами глікогенолізу,

якщо взаємодіють із:

А. Кальцитоніном.

В. Фосфорилазою С.

С. Кальциферолом.

D. Кіназою легких ланцюгів

міозину.

Е. *Кальмодуліном.

16. Після крововиливу в

мозок із пошкодженням

ядер гіпоталамуса у хворої

67 років розвинувся

нецукровий діабет. Що

стало причиною поліурії в

цьому випадку ?

А. Гіпоглікемія.

В. Зменшення реабсорбції

іонів калію.

С. Прискорення клубочкової

фільтрації.

D. Гіперглікемія.

Е. *Зменшення реабсорбції

води.

17. Хлопчик 10 років

прийнятий до лікарні для

обстеження з приводу

маленького зросту. За два

останніх роки він виріс

усього на 3 см. Недостат-

ністю якого гормону

зумовлений такий стан ?

А. Тиреотропіну.

В. Кортикотропіну.

С. Гонадотропіну.

D. Паратгормону.

Е. *Соматотропіну.

18. Чоловік 50 років пе-

режив сильний стрес. У

крові різко збільшилася

концентрація адреналіну і

норадреналіну. Які

ферменти каталізують

процес інактивації

катехоламінів ?

А. Глікозидази.

В. Тирозинази.

С. Пептидази.

D. Карбоксилази.

Е. *Моноамінооксидази.

19. У хворих на тирео-

токсикоз спостерігаються

гіпертермія, тремор,

Page 70: КРОК-1 БІОХІМІЯ Збірник тестових завданьlib.sumdu.edu.ua/library/docs/rio/2020/m4695.pdf · 2 Крок-1. Біохімія Н: збірник тестових

70

схуднення, що пов’язане з

порушенням:

A. Реакцій бета-окиснення

жирних кислот.

B. Розпаду АТФ.

C. Реакцій окиснення триглі-

церидів.

D. Реакцій циклу лимонної

кислоти.

E. *Спряження окиснення і

фосфорилювання.

20. У хворого сеча у кіль-

кості 8 л на добу має

питому вагу 1,006. Зі

зниженням секреції якого

гормону це пов’язано ?

A. Соматотропіну.

B. Інсуліну.

C. Йодтиронінів.

D. Глюкокортикоїдів.

E. *Вазопресину.

21. Пацієнт скаржиться на

м’язову слабість та

потемніння шкіри всього

тіла. При обстеженні

виявлені такі зміни:

артеріальний тиск –

100/60 мм рт. ст.,

концентрація глюкози у

крові становить 3,0

ммоль/л. Яка хвороба у

пацієнта ?

A. Пелагра.

B. Мікседема.

C. Інсулома (із підвищеною

продукцією інсуліну).

D. Синдром Іценка-

Кушинга.

E. *Хвороба Аддісона.

22. В ендокринологічний

диспансер звернулася

жінка 40 років зі скаргами

на тремтіння рук,

серцебиття, постійну

гіпертермію (37 - 38 0С),

схуднення. При аналізі

крові виявлено підви-

щення рівня цукру,

жирних кислот та

амінокислот. Гіпер-

продукція яких гормонів

спричиняє ці симптоми ?

A. Соматотропінів.

B. Глюкокортикоїдів.

C. Кортикотропіну.

D. Інсуліну.

E. *Йодтиронінів.

23. У дитячу лікарню

потрапила дитина з

ознаками рахіту

(деформація кісток, пізнє

заростання тім’я та ін.)

При біохімічному аналізі

крові відмічені такі зміни:

A. Підвищення рівня Na+.

B. Зниження рівня К+.

Page 71: КРОК-1 БІОХІМІЯ Збірник тестових завданьlib.sumdu.edu.ua/library/docs/rio/2020/m4695.pdf · 2 Крок-1. Біохімія Н: збірник тестових

71

C. Підвищення рівня фосфа-

тів.

D. Зниження рівня Mg2+.

E. *Зниження рівня Са2+.

24. Чоловік 42 років

потрапив у кардіологічне

відділення з діагнозом

стенокардія. До комплексу

препаратів, призначених

хворому, входить інгібітор

ферменту фосфодиесте-

рази. Концентрація якої

речовини в серцевому

м’язі буде збільшуватися ?

А. АДФ.

В. ГМФ.

С. АМФ.

D. *цАМФ.

25. У хворої жінки з низь-

ким артеріальним тиском

після парентерального

введення гормону спосте-

рігається підвищення

артеріального тиску і

зростання концентрації

глюкози та ліпідів у крові.

Який гормон було

введено ?

A. Фолікулін.

B. Глюкагон.

C. Інсулін.

D. Прогестерон.

E. *Адреналін.

26. При огляді хворого

лікар запідозрив синдром

Іценка-Кушинга. Визна-

чення якої речовини в

крові хворого підтвердить

діагноз лікаря ?

A. Холестерину.

B. Токоферолу.

C. Ретинолу.

D. Адреналіну.

E. *Кортизолу.

27. Хворий 40 років

госпіталізований зі

скаргами на загальну

слабкість, судоми верхніх і

нижніх кінцівок, АТ -

160/100 мм–рт.ст.

Результати дослідження:

глюкоза крові -

6,5 ммоль/л, холестерин -

6 ммоль/л, кальцій -

2 ммоль/л, фосфор -

1 ммоль/л, натрій -

160 ммоль/л. Сечо-

виділення - 700 мл за –обу.

Яка патологія призвела до

такого стану ?

A. Рахіт.

B. Гіпоальдостеронізм.

C. Гіперпаратиреоїдизм.

D. Тиреотоксикоз.

E. *Гіперальдостеронізм.

Page 72: КРОК-1 БІОХІМІЯ Збірник тестових завданьlib.sumdu.edu.ua/library/docs/rio/2020/m4695.pdf · 2 Крок-1. Біохімія Н: збірник тестових

72

28. Тривале вживання

великих доз аспірину

спричиняє пригнічення

синтезу простагландинів у

результаті зниження

активності ферменту:

A. Фосфодиестерази.

B. Пероксидази.

C. 5-Ліпооксигенази.

D. Фосфоліпази А2.

E. *Циклооксигенази.

29. Арахідонова кислота як

незамінний компонент їжі

є попередником біологічно

активних речовин. Яка

сполука синтезується з

неї ?

A. Трийодтиронін.

B. Холін.

C. Норадреналін.

D. Етаноламін.

E. *Простагландин Е1.

30. У дитини спостерігає-

ться порушення форму-

вання емалі і дентину зубів

через знижений вміст іонів

кальцію в крові. Дефіцит

якого гормону може

спричинити такі зміни ?

A. Трийодотироніну.

B. Соматотропного гормону.

C. Тироксину.

D. Паратгормону.

E. *Тиреокальцитоніну.

31. Лікар призначив

аспірин хворому на

ревматизм як проти-

запальний засіб. Синтез

яких речовин, пов’язаних

із запаленням, блокує

аспірин ?

A. Дофаміну.

B. Гліцерину.

C. Треоніну.

D. Глюкагону.

E. *Простагландинів.

32. При надлишку секреції

гормону у хворого

развинувся екзофтальм,

виникли тахікардія,

нервовість і схуднення.

Про який гормон іде мова?

A. Естрадіол.

B. Адреналін.

C. Дезоксикортикостерон.

D. Тестостерон.

E. *Тироксин.

33. У хворого у віці 28

років унаслідок пухлини

спостерігається непропор-

ційно інтенсивний ріст

рук, ніг, підборіддя

(акромегалія). Надлишок

якого гормону спричинив

такі порушення ?

A. Тиротропіну.

B. Тироксину.

Page 73: КРОК-1 БІОХІМІЯ Збірник тестових завданьlib.sumdu.edu.ua/library/docs/rio/2020/m4695.pdf · 2 Крок-1. Біохімія Н: збірник тестових

73

C. Гонадотропіну.

D. Адренокортикотропіну.

E. *Соматотропіну.

34. Особливістю обміну

тирозину є залучення його

у процес синтезу гормонів.

Назвіть один із них, що

утворюється в мозковому

шарі наднирників:

A. Серотонін.

B. Глюкагон.

C. Тироксин.

D. Гістамін.

E. *Адреналін.

35. Тирозин використову-

ється як субстрат у процесі

синтезу тироксину. Наз-

віть хімічний елемент,

який бере участь у цьому

процесі:

A. Цинк.

B. Кальцій.

C. Залізо.

D. Мідь.

E. *Йод.

36. При емоційному стресі

в адипоцитах активується

гормоночутлива три-

гліцеридліпаза. Який

вторинний посередник

бере участь у цьому

процесі ?

A. Іони Са2+.

B. цГМФ.

C. АМФ.

D. Діацилгліцерол.

E. *цАМФ.

37. У хворого відмічається

схуднення, підвищення ос-

новного обміну при підви-

щеному апетиті і гарному

харчуванні. Надлишок

якого гормону спричиняє

такі зміни ?

A. Адреналіну.

B. Глюкагону.

C. АКТГ.

D. Інсуліну.

E. *Тироксину.

38. У хворого 50 років є

скарги на спрагу. Добовий

діурез 4 - 5 л. Рівень

глюкози в крові

4,6 ммоль/л, у сечі глюкоза

не виявлена. У даному

випадку доцільно переві-

рити вміст у крові:

A. Тироксину.

B. Естрогенів.

C. Альдостерону.

D. Кортизолу.

E. *Вазопресину.

39. На основі тестостерону

синтезовані препарати, які

Page 74: КРОК-1 БІОХІМІЯ Збірник тестових завданьlib.sumdu.edu.ua/library/docs/rio/2020/m4695.pdf · 2 Крок-1. Біохімія Н: збірник тестових

74

використовують при

захворюваннях, що

супроводжуються висна-

женням, при переломах і

т.д. Який ефект цих

препаратів ?

A. Амфіболічний.

B. Андрогенний.

C. Естрогенний.

D. Катаболічний.

E. *Анаболічний.

40. Нормальна будова і

функція емалі забезпе-

чуються динамічною

рівновагою процесів

демінералізації і ремінера-

лізації. Які гормони мають

найбільший вплив на

баланс цих процесів ?

A. Тиреоїдні гормони і сома-

тотропін.

B. Глюкокортикоїди і

мінералокортикоїди.

C. Інсулін і глюкагон.

D. Мінералокортикоїди і

паратгормон.

E. *Тиреокальцитонін і

паратгормон.

41. Хворий 45 років

скаржиться на спрагу,

споживання великої

кількості рідини (до 5 л),

виділення значної

кількості сечі (до 6 л на

добу). Концентрація

глюкози у крові становить

4,4 ммоль/л, рівень кето-

нових тіл не підвищений.

Сеча має густину 1,002;

цукор у сечі не виявлений.

Дефіцит якого гормону

може призводити до таких

змін ?

A. АКТГ.

B. Альдостерону.

C. Інсуліну.

D. Глюкагону.

E. *Вазопресину.

42. В експериментальних

дослідженнях було вста-

новлено, що стероїдні

гормони впливають на

протеосинтез. На який

етап цього процесу вони

впливають ?

А. Синтез ГТФ.

В. Синтез АТФ.

С. Синтез специфічних

тРНК.

D. Синтез специфічних

рРНК.

Е. *Синтез специфічних

мРНК.

43. У 4-місячної дитини

яскраво виражене явище

рахіту. Розладів травлення

Page 75: КРОК-1 БІОХІМІЯ Збірник тестових завданьlib.sumdu.edu.ua/library/docs/rio/2020/m4695.pdf · 2 Крок-1. Біохімія Н: збірник тестових

75

не відмічено. Дитина

багато буває на сонці.

Упродовж 2 місяців дитина

отримувала вітамін D3, але

ознаки рахіту не зникли.

Чим можна пояснити

розвиток рахіту у цієї

дитини ?

A. Порушенням синтезу

інсуліну.

B. Порушенням синтезу

кальцитоніну.

C. Порушенням синтезу

паратгормону.

D. Порушенням синтезу

тироксину.

E. *Порушенням синтезу

кальцитріолу.

44. Хворий напередодні

операції був у стані стресу.

Збільшення концентрації

якого гормону в крові

супроводжує цей стан ?

A. Глюкагон.

B. Інсулін.

C. Пролактин.

D. Прогестерон.

E. *Адреналін.

45. До лікаря звернувся

хворий із скаргами на

постійну спрагу. Виявлені

гіперглікемія, поліурія та

підвищений вміст 17-

кетостероїдів у сечі. Яке

захворювання ймовірне ?

A. Адісонова хвороба.

B. Інсулінозалежний діабет.

C. Мікседема.

D. Глікогеноз I типу.

E. *Стероїдний діабет.

46. До лікаря звернулися

батьки 5-річної дитини.

При обстеженні дитини

виявлено: відставання

розумового та фізичного

розвитку. Дитина мало-

рухлива. Загальний обмін

знижений. Яке захворю-

вання у дитини ?

A. Ендемічний зоб.

B. Синдром Леша –Ніхана.

C. Фенілкетонурія.

D. Гіперпаратиреоз.

E. *Кретинізм.

47. У хворого на цироз

печінки з’явилися ознаки

гіперальдостеронізму. Зни-

ження активності яких

ферментів призвело до

недостатньої інактивації

гормону ?

A. Лізосомальних гідролаз.

B. Цитоплазматичних

глутатіонпероксидаз.

C. Цитоплазматичних

дегідрогеназ.

Page 76: КРОК-1 БІОХІМІЯ Збірник тестових завданьlib.sumdu.edu.ua/library/docs/rio/2020/m4695.pdf · 2 Крок-1. Біохімія Н: збірник тестових

76

D. Мітохондріальних

дегідрогеназ.

E. *Мікросомальних

гідроксилаз.

48. Вторинним посередни-

ком у механізмі дії адрена-

ліну є:

A. цЦМФ.

B. цГМФ.

C. цУМФ.

D. цТМФ.

E. *цАМФ.

49. Чоловік 42 років

страждає на ревматоїдний

артрит. До комплексу

призначених йому

лікувальних препаратів

додали аспірин - інгібітор –

творення простагланди-

нів. З якої кислоти

утворюються проста-

гландини ?

A. Пропіонової.

B. Нейрамінової.

C. Ліноленової.

D. Лінолевої.

E. *Арахідонової.

50. При видаленні гіпер-

плазованої щитовидної

залози у 47-річної жінки

було пошкоджено пара-

щитовидну залозу. Через 1

місяць після операції у

пацієнтки з'явилися

озн’ки гіпопаратиреозу:

часті судоми, гіпер-

рефлекси, спазм гортані.

Що є найбільш вірогідною

причиною стану жінки ?

A. Гіперкаліємія.

B. Гіпонатріємія.

C. Гіперхлоргідрія.

D. Гіпофосфатемія.

E. *Гіпокальціємія.

51. У чоловіка 62 років

діагностовано аденому

передміхурової залози.

Йому був призначений

синестрол – синтетичний

естрогенний препарат. Що

обумовлює терапевтичний

ефект цього препарату ?

A. Гальмування синтезу

ЛПДНЩ.

B. Гальмування трансляції.

C. Порушення гліколізу.

D. Зниження транскрипції.

E. *Блокування рецепторів

тестостерону.

52. У чоловіка 40 років

виявлено гіпопаратиреоз.

Які результати лабора-

торних аналізів були

вирішальними при

встановленні діагнозу ?

Page 77: КРОК-1 БІОХІМІЯ Збірник тестових завданьlib.sumdu.edu.ua/library/docs/rio/2020/m4695.pdf · 2 Крок-1. Біохімія Н: збірник тестових

77

A. Підвищення вмісту в

крові сіалових кислот.

B. Гіпофосфатемія.

C. Підвищений рівень

оксипроліну в сечі.

D. Гіпокальціурія.

E. *Гіпокальціємія.

53. У хлопчика 15 років із

дефіцитом антидіуретич-

ного гормону спостерігає-

ться:

A. Аміноацидурія.

B. Кетонурія.

C. Глюкозурія.

D. Креатинурія.

E. *Гіпоізостенурія.

54. Зріст дорослої людини

дорівнює 100 см при про-

порційній статурі і

нормальному розумовому

розвитку. Недостатність

синтезу якого гормону має

місце ?

А. Гонадотропних гормонів.

В. Тироксину.

С. Вазопресину.

D. Мінералокортикоїдів.

Е. *Соматотропного

гормону.

55. При клінічному обсте-

женні жінки встановлено:

підвищення основного

обміну на 40 %, підвищене

потовиділення, тахікардія,

зниження маси тіла.

Функція якої з

ендокринних залоз

порушена ?

А. Статевих залоз, гіпофунк-

ція.

В. Кіркової речовини

наднирників, гіперфункція.

С. Мозкової речовини над-

нирників, гіперфункція.

D. Підшлункової залози,

гіперфункція.

Е. *Щитовидної залози,

гіперфункція.

56. Після інсульту з по-

шкодженням ядер гіпота-

ламусу у хворого виник

нецукровий діабет. Що

стало причиною посиле-

ного сечовиділення у

хворого ?

А. Зменшення реабсорбції

натрію.

В. Прискорення клубочко-

вої фільтрації.

С. Гіперглікемія.

D. Зменшення артеріального

тиску.

Е. *Зменшення реабсорбції

води.

Page 78: КРОК-1 БІОХІМІЯ Збірник тестових завданьlib.sumdu.edu.ua/library/docs/rio/2020/m4695.pdf · 2 Крок-1. Біохімія Н: збірник тестових

78

57. Жінка 44 років скар-

житься на загальну

слабість, біль у ділянці

серця, значне збільшення

маси тіла. Об’єктивно:

обличчя місяцеподібне,

гірсутизм, АТ – 165/100 мм

рт. ст., зріст – 164 см, вага

– 103 кг, переважне

накопичення жиру на шиї,

верхньому плечовому

поясі, животі. Що є

причиною ожиріння ?

А. Зниження продукції

тиреоїдних гормонів.

В. Зниження продукції

глюкагону.

С. Підвищення продукції

мінералокортикоїдів.

D. Підвищення продукції

інсуліну.

Е. *Підвищення продукції

глюкокортикоїдів.

58. У чоловіка 42 років,

який перебував у стані

стресу, в сечі значно

збільшився вміст 17-кето-

стероїдів, що свідчить про

підвищення секреції:

А. Адреналіну.

В. Норадреналіну.

С. Естрадіолу.

D. Альдостерону.

Е. *Кортизолу.

59. Унаслідок посиленого

потовиділення і зневод-

нення організму у пацієн-

та зросла осмолярність

сечі і зменшився діурез.

Зміни продукції якого

гормону забезпечують у

першу чергу компенсатор-

ну затримку води ?

А. Тироксину.

В. Альдостерону.

С. Інсуліну.

D. Кортикостерону.

Е. *Антидіуретичного.

60. У хворого із синдромом

Іценка-Кушинга в крові

підвищений вміст корти-

золу. З патологією якої

ендокринної залози це

пов’язано ?

А. Гіпофізу.

В. Підшлункової залози.

С. Мозкового шару

наднирників.

D. Щитовидної залози.

Е. *Кори наднирників.

61. У людини зменшений

діурез, гіпернатріємія, гі-

покаліємія. Гіперсекреція

якого гормону може бути

причиною таких змін ?

А. Адреналіну.

Page 79: КРОК-1 БІОХІМІЯ Збірник тестових завданьlib.sumdu.edu.ua/library/docs/rio/2020/m4695.pdf · 2 Крок-1. Біохімія Н: збірник тестових

79

В. Натрійуретичного

гормону передсердь.

С. Вазопресину.

D. Паратгормону.

Е. *Альдостерону.

62. У хворого із хронічним

запальним процесом

шкіри і підшкірної

клітковини виявлена

активація процесів пролі-

ферації. Недостатність

якого гормону може

призвести до цього ?

А. Альдостерону.

В. Інсуліну.

С. Тироксину.

D. СТГ.

Е. *Кортизону.

63. Чоловіку 35 років із

виразковою хворобою

зроблено резекцію антра-

льного відділу шлунка.

Секреція якого гастро-

інтестинального гормону

внаслідок операції буде

порушена найбільше ?

А. Нейротензину.

В. Гістаміну.

С. Холецистокініну.

D. Секретину.

Е. *Гастрину.

64. Хворого 40 років

госпіталізовано із скар-

гами на загальну слабість,

судоми верхніх і нижніх

кінцівок, АТ – 160/100 мм

рт.ст. Результати біохіміч-

ного аналізу крові:

глюкоза – 6,5 ммоль/л,

холестерин – 6 ммоль/л,

кальцій – 2 ммоль/л,

фосфор – 1 ммоль/л,

натрій – 160 ммоль/л.

Сечовиділення – 700 мл за

добу. Яка патологія буде

причиною такого стану ?

А. Гіпоальдостеронізм.

В. Рахіт.

С. Гіперпаратиреоїдизм.

D. Тиреотоксикоз.

Е. *Гіперальдостеронізм.

65. До лікаря звернувся

пацієнт високого зросту, з

довгими товстими паль-

цями рук, великою ниж-

ньою щелепою і відвис-

лою нижньою губою.

Підвищену секрецію якого

гормону якої залози можна

припустити ?

А. Гормонів наднирників із

групи глюкокортикоїдів.

В. Антидіуретичного

гормону.

Page 80: КРОК-1 БІОХІМІЯ Збірник тестових завданьlib.sumdu.edu.ua/library/docs/rio/2020/m4695.pdf · 2 Крок-1. Біохімія Н: збірник тестових

80

С. Гормонів щитовидної

залози.

D. Гонадотропного гормону.

Е. *Соматотропного

гормону.

66. Аспірин виявляє

протизапальну дію,

оскільки інгібує актив-

ність циклооксигенази.

Рівень яких біоологічно

активних речовин буде

при цьому знижуватися ?

А. Біогенних амінів.

В. Катехоламінів.

С. Йодтиронінів.

D. Лейкотрієнів.

Е. *Простагландинів.

67. У 4-місячної дитини

яскраво виражені ознаки

рахіту. Розлади травлення

не спостерігаються.

Дитина багато часу пере-

буває на сонці. Протягом 2

місяців дитина отри-

мувала вітамін D3, однак

ознаки рахіту не зменши-

лися. Порушенням синте-

зу якої речовини можна

пояснити розвиток рахіту

у цієї дитини ?

А. Кальцитоніну.

В. Інсуліну.

С. Тироксину.

D. Паратгормону.

Е. *Кальцитріолу.

68. У жінки 62 років

розвинулася катаракта на

фоні цукрового діабету.

Посилення якого процесу

при діабеті є причиною

помутніння кришталика ?

А. Протеолізу білків.

В. Глюконеогенезу.

С. Кетогенезу.

D. Ліполізу.

Е. *Глікозилювання білків.

69. У жінки з первинним

гіперпаратиреоїдизмом

періодично повторюються

напади ниркової коліки.

Ультразвукове обстеження

показало наявність

дрібних каменів у нирках.

Яка найбільш імовірна

причина утворення цих

каменів ?

А. Гіперкаліємія.

В. Гіперурикемія.

С. Гіперхолестеринемія.

D. Гіперфосфатемія.

Е. *Гіперкальціємія.

70. У дитини 2 років ви-

никли судоми унаслідок

зниження концентрації

іонів кальцію в плазмі

Page 81: КРОК-1 БІОХІМІЯ Збірник тестових завданьlib.sumdu.edu.ua/library/docs/rio/2020/m4695.pdf · 2 Крок-1. Біохімія Н: збірник тестових

81

крові. Це зумовлено

зниженням функції:

А. Кори наднирників.

В. Тимуса.

С. Шишкоподібної залози.

D. Гіпофіза.

Е. *Паращитовидних залоз.

71. У хворого видалено

дванадцятипалу кишку.

Це призвело до зменшення

секреції таких гастро-

інтестинальних гормонів:

А. Гістаміну.

В. Нейротензину.

С. Гастрину та гістаміну.

D. Гастрину.

Е. *Холецистокініну та

секретину.

72. Застосування оральних

контрацептивів, що

містять статеві гормони,

пригнічує секрецію гормо-

нів гіпофіза. Секреція

якого гормону пригні-

чується ?

А. Соматотропного.

В. Окситоцину.

С. Тиреотропного.

D. Вазопресину.

Е. *Фолікулостимулюючого.

73. У хворого з верхнім

типом ожиріння тривалий

час відзначалися

гіпертонія, гіперглікемія,

глюкозурія. Смерть

настала від крововиливу в

головний мозок. Під час

патоморфологічного

дослідження виявлені

базофільна аденома

гіпофіза, гіперплазія кори

наднирників. Який

найбільш імовірний

діагноз ?

А. Акромегалія.

В. Цукровий діабет.

С. Адипозогенітальна

дистрофія.

D. Гіпофізарний нанізм.

Е. *Хвороба Іценка-

Кушинга.

74. При моделюванні

запалення нижньої

кінцівки у тварини

підвищилася температура

тіла, збільшився вміст

антитіл та лейкоцитів у

крові. Які речовини

зумовили розвиток цих

запальних реакцій

організму ?

А. Соматомедини.

В. Глюкокортикоїди.

С. Інтерлейкіни.

D. Мінералокортикоїди.

Е. *Лейкотрієни.

Page 82: КРОК-1 БІОХІМІЯ Збірник тестових завданьlib.sumdu.edu.ua/library/docs/rio/2020/m4695.pdf · 2 Крок-1. Біохімія Н: збірник тестових

82

75. У жінки 35 років із

хронічним захворюванням

нирок розвинувся

остеопороз. Дефіцит якої

речовини є основною

причиною остеопорозу ?

А. 25(ОН)D3.

В. D2.

С. D3.

D. Холестерину.

Е. *1,25(ОН)2D3.

76. У людини внаслідок

втрати 1,5 л крові різко

зменшився діурез. Поси-

лена секреція якого гор-

мону спричинила зміни

діурезу ?

А. Натрійуретичного.

В. Кортикотропіну.

С. Кортизолу.

D. Паратгормону.

Е. *Вазопресину.

77. Недбалий студент

раптово зустрівся з деканом.

Концентрація якого гормону

найшвидше збільшиться в

крові студента ?

А. Тиреоліберину.

В. Кортизолу.

С. Соматотропіну.

D. Кортикотропіну.

Е. *Адреналіну.

78. Людина за призна-

ченням лікаря тривалий

час приймала препарат із

групи глюкокортикоїдних

гормонів. Секреція якого з

наведених гормонів буде

пригнічена внаслідок

цього ?

А. Мінералокортикоїди.

В. Соматотропний.

С. Статеві.

D. Тиреотропний.

Е. *Кортикотропний.

79. Підліток 12 років

протягом 3 місяців

втратив 7 кг маси тіла.

Вміст глюкози у крові

становить 20 ммоль/л.

Несподівано розвинулася

кома. Який вид цукрового

діабету найбільш вірогід-

ний у хлопчика ?

А. Гіпертиреоїдний.

В. Гіпофізарний.

С. Інсулінонезалежний

(II тип).

D. Стероїдний.

Е. *Інсулінозалежний

(I тип).

80. У нирках пацієнта

збільшена реабсорбція

іонів кальцію і зменшена –

фосфатних іонів. Впливом

Page 83: КРОК-1 БІОХІМІЯ Збірник тестових завданьlib.sumdu.edu.ua/library/docs/rio/2020/m4695.pdf · 2 Крок-1. Біохімія Н: збірник тестових

83

якого гормону це

зумовлено ?

А. Альдостероном.

В. Вазопресином.

С. Гормональною формою

вітаміну D3.

D. Тирокальцитоніном.

Е. *Паратгормоном.

81. Хворий із нейро-

дермітом упродовж

тривалого часу вживав

преднізолон. При обсте-

женні в нього виявили

підвищення рівня цукру в

крові. Вплив препарату на

яку ланку вуглеводного

обміну призводить до

виникнення цього

ускладнення ?

А. Пригнічення синтезу

глікогену.

В. Посилення всмоктування

глюкози в кишечнику.

С. Активація глікогено-

генезу.

D. Активація розщеплення

інсуліну.

Е. *Активація глюконео-

генезу.

82. У хворого з діагнозом

хвороба Іценка-Кушинга

(гіперпродукція гормонів

корою наднирників) в

крові визначено підвище-

ну концентрацію глюкози,

кетонових тіл, натрію.

Який біохімічний механізм

є провідним у виникненні

гіперглікемії ?

А. Гліколіз.

В. Глікогеноліз.

С. Аеробний гліколіз.

D. Глікогенез.

Е. *Глюконеогенез.

1.9. ВІТАМІНИ

1. При авітамінозі якого

вітаміну може суттєво

знижуватися активність

трансаміназ сироватки

крові ?

A. Вс (фолієва кислота).

B. В1 (тіамін).

C. В2 (рибофлавін).

D. В5 (нікотинамід).

E. *В6 (піридоксин).

2. У жінки, яка тривалий

час була на дієті з вико-

ристанням шліфованого

рису, виявлено поліневрит

(хвороба бері-бері). Відсут-

ність якого вітаміну в їжі

призводить до розвитку

цієї хвороби ?

A. Рибофлавіну.

Page 84: КРОК-1 БІОХІМІЯ Збірник тестових завданьlib.sumdu.edu.ua/library/docs/rio/2020/m4695.pdf · 2 Крок-1. Біохімія Н: збірник тестових

84

B. Аскорбінової кислоти.

C. Піридоксалю.

D. Фолієвої кислоти.

E. *Тіаміну.

3. У хворого після ектомії

жовчного міхура пригні-

чені процеси всмо-

ктування Са2+ через стінку

кишечника. Призначення

якого вітаміну буде

стимулювати цей процес ?

A. Вітамін К.

B. Вітамін РР.

C. Вітамін С.

D. Вітамін В12.

E. *Вітамін D3.

4. Гідроксипролін є важ-

ливою амінокислотою у

складі колагену. За участі

якого вітаміну відбуває-

ться утворення цієї

амінокислоти шляхом

гідроксилювання проліну?

А. В1.

В. D.

С. В6.

D. В2.

Е. *С.

5. Хворому встановлено

діагноз - хвороба бе–і-бері.

Активність якого фермен-

ту порушена у пацієнта ?

A. Фумарази.

B. Цитратсинтази.

C. Малатдегідрогенази.

D. Сукцинатдегідрогенази.

E. *Піруватдегідрогенази.

6. Лікар-дієтолог радить

хворому під час лікування

мегалобластичної анемії

включати в раціон

харчування напівсиру

печінку. Наявність якого

вітаміну в цьому продукті

стимулює процес

кровотворення ?

A. Вітамін Н.

B. Вітамін В1.

C. Вітамін В2.

D. Вітамін С.

E. *Вітамін В12.

7. У хворого на цингу

виявлено порушення

гідроксилювання проліну і

лізину у складі колагену.

Гальмування якого

біохімічного процесу

призводить до цього

порушення ?

A. Окиснювального фосфо-

рилювання.

B. Перекисного окиснення

ліпідів.

C. Тканинного дихання.

Page 85: КРОК-1 БІОХІМІЯ Збірник тестових завданьlib.sumdu.edu.ua/library/docs/rio/2020/m4695.pdf · 2 Крок-1. Біохімія Н: збірник тестових

85

D. Пероксидазного окис-

нення.

E. *Мікросомального окис-

нення.

8. При ентеробіозі призна-

чають акрихін – струк-

турний аналог вітаміну В2.

Порушення синтезу яких

ферментів спричиняє цей

препарат ?

А. Цитохромоксидаз.

В. Амінотрансфераз.

С. Пептидаз.

D. НАД-залежних дегідро-

геназ.

Е. *ФАД-залежних дегідро-

геназ.

9. Дівчинка 10 років часто

хворіє на гострі

респіраторні інфекції,

після яких спостері-

гаються множинні точкові

крововиливи в місцях

тертя одягу. Який

гіповітаміноз у дівчинки ?

А. В2.

В. В6.

С. В1.

D. А.

Е. *С.

10. При різних захворю-

ваннях рівень активних

форм кисню різко

підвищується, що

призводить до руйнуван-

ня клітинних мембран. З

метою запобігання цьому

застосовують анти-

оксиданти. Назвіть

найпотужніший природ-

ний антиоксидант:

А. Гліцерол.

В. Глюкоза.

С. Вітамін D.

D. Жирні кислоти.

Е. *α-Токоферол.

11. Вагітній із кількома

спонтанними абортами в

анамнезі призначено

терапію вітамінними

препаратами. Який

вітамін сприяє виношу-

ванню вагітності ?

А. Рутин.

В. Фолієва кислота.

С. Ціанокобаламін.

D. Піридоксальфосфат.

Е. *α-Токоферол.

12. У хворого з частими

кровотечами із внутрішніх

органів і слизових

оболонок виявлені пролін і

лізин у складі колагенових

волокон. Через відсутність

Page 86: КРОК-1 БІОХІМІЯ Збірник тестових завданьlib.sumdu.edu.ua/library/docs/rio/2020/m4695.pdf · 2 Крок-1. Біохімія Н: збірник тестових

86

якого вітаміну порушено їх

гідроксилювання ?

A. Вітаміну Е.

B. Вітаміну К.

C. Вітаміну А.

D. Тіаміну.

E. *Вітаміну С.

13. Вітамін А у комплексі

зі специфічними цито-

рецепторами проникає

через ядерні мембрани,

індукує процеси

транскрипції, що

стимулює ріст та

диференціювання клітин.

Ця біологічна функція

реалізується такою

формою вітаміну А:

A. Каротин.

B. Трансретиналь.

C. Цисретиналь.

D. Ретинол.

E. *Трансретиноєва кислота.

14. Хворому, що страждає

на тромбоемболічну

хворобу, призначено

штучний антикоагулянт -

пелентан. АУнтагоністом

якого вітаміну є його

сполука ?

A. Вітаміну С.

B. Вітаміну Е.

C. Вітаміну А.

D. Вітаміну D.

E. *Вітаміну К.

15. У хворого

спостерігається гемера-

лопія (куряча сліпота).

Яка з наведених нижче

речовин буде мати

лікувальну дію ?

A. Карнозин.

B. Кератин.

C. Креатин.

D. Карнітин.

E. *Каротин.

16. Хворий скаржиться на

загальну слабість і

кровоточивість ясен.

Недостатність якого

вітаміну може бути

причиною такого стану ?

A. Вітаміну D.

B. Вітаміну Е.

C. Вітаміну А.

D. Вітаміну Н.

E. *Вітаміну С.

17. У хворого 37 років на

фоні тривалого застосуван-

ня антибіотиків

спостерігається підвищена

кровоточивість при

невеликих пошкод-

женнях. У крові – зни-

ження активності

Page 87: КРОК-1 БІОХІМІЯ Збірник тестових завданьlib.sumdu.edu.ua/library/docs/rio/2020/m4695.pdf · 2 Крок-1. Біохімія Н: збірник тестових

87

факторів згортання крові

ІІ, VII, X, подовження часу

згортання крові. Недостат-

ністю якого вітаміну

обумовлені зазначені

зміни ?

A. Вітаміну Е.

B. Вітаміну А.

C. Вітаміну С.

D. Вітаміну D.

E. *Вітаміну К.

18. У хворого 36 років,

який страждає на

хронічний алкоголізм, у

крові має місце

накопичення пірувату, в

еритроцитах – зниження

активності транскетолази.

Назвіть коферментну

форму вітаміну, недостат-

ністю якого обумовлені

зазначені зміни ?

A. Тетрагідрофолат.

B. Карбоксибіотин.

C. Метилкобаламін.

D. Фосфопіридоксаль.

E. *Тіаміндифосфат.

19. Окуліст виявив у

хворого збільшення часу

адаптації ока до темряви.

Недостатність якого

вітаміну може бути

причиною такого

симптому ?

А. Е.

В. D.

С. С.

D. К.

Е. *А.

20. У хворого – дерматит,

діарея, деменція. Відсут-

ність якого вітаміну є

причиною цього стану ?

А. Фолієвої кислоти.

В. Аскорбінової кислоти.

С. Рутину.

D. Біотину.

Е. *Нікотинаміду.

21. У хворого діагносто-

вано мегалобластичну

анемію. Назвіть сполуку,

недостатня кількість якої

в організмі може

призводити до розвитку

цієї хвороби:

А. Мідь.

В. Гліцин.

С. Магній.

D. Холекальциферол.

Е. *Ціанокобаламін.

22. За клінічними показни-

ками хворому призначено

піридоксальфосфат. Для

корекції яких процесів

Page 88: КРОК-1 БІОХІМІЯ Збірник тестових завданьlib.sumdu.edu.ua/library/docs/rio/2020/m4695.pdf · 2 Крок-1. Біохімія Н: збірник тестових

88

рекомендований цей

препарат ?

А. Синтезу пуринових і

піримідинових основ.

В. Окисного декарбокси-

лювання кетокислот.

С. Дезамінування аміно-

кислот.

D. Синтезу білка.

Е. *Трансамінування і

декарбоксилювання

амінокислот.

23. У пацієнта після вжи-

вання сирих яєць з’явився

дерматит. Який розвинув-

ся авітаміноз ?

А. Фолієвої кислоти.

В. Інозиту.

С. Пантотенової кислоти.

D. Параамінобензойної

кислоти.

Е. *Біотину.

24. У хворого в крові збіль-

шена концентрація

пірувату. Значна кількість

його екскретується з

сечею. Який авітаміноз у

хворого ?

А. Авітаміноз В2.

В. Авітаміноз Е.

С. Авітаміноз В3.

D. Авітаміноз В6.

Е. *Авітаміноз В1.

25. У хворого 43 років

спостерігаються хронічний

атрофічний гастрит,

мегалобластична анемія,

збільшене виділення

метилмалонової кислоти із

сечею. Недостатністю

якого вітаміну обумовлене

виникнення зазначеного

стану ?

A. В1.

B. В2.

C. В3.

D. В5.

E. *В12.

26. Визначення активності

трансаміназ широко

застосовується з метою

діагностики пошкоджень

внутрішніх органів.

Кофактором цих фер-

ментів є активна форма

вітаміну:

A. РР.

B. В1.

C. В12.

D. В2.

E. *В6.

27. У дитини 2-х років

після тривалого лікування

антибіотиками розвинувся

дисбактеріоз: майже повна

відсутність кишкової

Page 89: КРОК-1 БІОХІМІЯ Збірник тестових завданьlib.sumdu.edu.ua/library/docs/rio/2020/m4695.pdf · 2 Крок-1. Біохімія Н: збірник тестових

89

палички. Недостатність

вітамінів якої групи може

виникнути у звязку з

цим ?

A. D.

B. А.

C. С.

D. Е.

E. *В.

28. Для запобігання після-

операційної кровотечі 6-

річній дитині рекомендо-

вано приймати вікасол,

який є синтетичним

аналогом вітаміну К.

Назвіть, які посттрансля-

ційні зміни факторів

згортання крові

активуються під впливом

вікасолу:

A. Глікозилювання.

B. Фосфорилювання серину.

C. Частковий протеоліз.

D. Полімеризація.

E. *Карбоксилювання глута-

мінової кислоти.

29. Більша частина

учасників навколосвітньої

подорожі Магеллана

загинула від авітамінозу,

що виявлявся загальною

слабістю, підшкірними

крововиливами, випадін-

ням зубів, кровотечею з

ясен. Назвіть цей

авітаміноз:

А. Пелагра.

В. Анемія Бірмера.

С. Рахіт.

D. Поліневрит (бері-бері).

Е. *Цинга.

30. У чоловіка, який

тривалий час не вживав

жирів, але отримував до-

статню кількість вуглево-

дів і білків, виявлено

дерматит, погане загою-

вання ран, погіршення

зору. Яка можлива

причина порушення

обміну речовин ?

А. Дефіцит олеїнової

кислоти.

В. Дефіцит пальмітинової

кислоти.

С. Дефіцит вітамінів РР, Н.

D. Низька калорійність

дієти.

Е. *Дефіцит лінолевої кис-

лоти, вітамінів А, Д, Е, К.

31. Хворі на алкоголізм

отримують основну масу

калорій із спиртними

напоями. У них може

виникнути характерна

недостатність тіаміну

Page 90: КРОК-1 БІОХІМІЯ Збірник тестових завданьlib.sumdu.edu.ua/library/docs/rio/2020/m4695.pdf · 2 Крок-1. Біохімія Н: збірник тестових

90

(синдром Верніке-

Корсакова), при якій

спостерігаються пору-

шення функцій нервової

системи, психози, втрата

пам’яті. Із зниженням

активності якого ферменту

це пов’язано ?

А. Алкогольдегідрогенази.

В. Гексокінази.

С. Трансамінази.

D. Альдолази.

Е. *Піруватдегідрогенази.

32. Лікування дитини, хво-

рої на рахіт, за допомогою

вітаміну D не дало

позитивного результату.

Яка найбільш імовірна

причина неефективності

лікування ?

А. Недостатність ліпідів у

їжі.

В. Порушення транспорту

вітаміну D3 білками крові.

С. Порушення включення

вітаміну D3 до ферменту.

D. Підвищене використання

вітаміну D3 мікрофлорою

кишечника.

Е. *Порушення гідрокси-

лювання вітаміну D3.

33. Після лікування хворо-

го антибіотиками внаслі-

док пригнічення мікро-

флори кишечника

можливий гіповітаміноз:

А. А.

В. С.

С. D.

D. Р.

Е. *В12.

34. Під час патронажу

лікар виявив у дитини

симетричну шорсткість

шкіри щік, пронос,

порушення нервової

діяльності. Дефіцит яких

харчових факторів є

причиною такого стану ?

А. Фенілаланіну, пангамової

кислоти.

В. Лізину, аскорбінової

кислоти.

С. Треоніну, пантотенової

кислоти.

D. Метіоніну, ліпоєвої

кислоти.

Е. *Нікотинової кислоти,

триптофану.

35. При огляді дитини у

віці 11 місяців педіатр

виявив викривлення

кісток нижніх кінцівок і

затримку мінералізації

кісток черепа. Недо-

статність якого вітаміну

Page 91: КРОК-1 БІОХІМІЯ Збірник тестових завданьlib.sumdu.edu.ua/library/docs/rio/2020/m4695.pdf · 2 Крок-1. Біохімія Н: збірник тестових

91

призводить до даної

патології ?

A. Рибофлавіну.

B. Тіаміну.

C. Пантотенової кислоти.

D. Біофлавоноїдів.

E. *Холекальциферолу.

36. У хворого в результаті

неповноцінного харчуван-

ня з’явилися діарея,

деменція і дерматит.

Недостатністю якого

вітаміну спричинений

такий стан ?

A. В12.

B. В1.

C. В2.

D. С.

E. *РР.

37. У пацієнта при повно-

цінному харчуванні розви-

нулася гіперхромна

(мегалобластична) анемія.

Напередодні він переніс

операцію резекції шлунка.

Яка причина анемії ?

A. Недостатність фолієвої

кислоти в їжі.

B. Недостатність вітаміну С

в їжі.

C. Недостатність вітаміну РР

в їжі.

D. Недостатність білка в їжі.

E. *Недостатність фактора

Касла.

38. Унаслідок дефіциту

вітаміну В1 порушується

окисне декарбоксилю-

вання -кетоглутарової

кислоти. Синтез якого з

наведених коферментів

порушується при цьому ?

A. Коензиму А.

B. Нікотинамідаденіндинук-

леотиду (НАД).

C. Флавінаденіндинуклео-

тиду (ФАД).

D. Ліпоєвої кислоти (ЛК).

E. *Тіамінпірофосфату

(ТПФ).

39. У 2-річної дитини киш-

ковий дисбактеріоз, на

фоні якого з’явився

геморагічний синдром.

Найбільш імовірною

причиною геморагій у цієї

дитини є:

А. Гіпокальціємія.

B. Активація тромбо-

пластину тканин.

C. Гіповітаміноз РР.

D. Дефіцит фібриногену.

E. *Нестача вітаміну К.

40. Гіповітаміноз С призво-

дить до зменшення

Page 92: КРОК-1 БІОХІМІЯ Збірник тестових завданьlib.sumdu.edu.ua/library/docs/rio/2020/m4695.pdf · 2 Крок-1. Біохімія Н: збірник тестових

92

утворення органічного

матриксу, порушення

синтезу колагену, оскільки

цей вітамін бере участь у

процесах:

A. Гідроксилювання

триптофану.

B. Карбоксилювання

проліну.

C. Карбоксилювання лізину.

D. Гідроксилювання

аргініну.

E. *Гідроксилювання

проліну.

41. Злоякісна гіперхромна

анемія – хвороба Бірмера –

виникає унаслідок дефі-

циту вітаміну В12. Який

біоелемент входить до

складу цього вітаміну ?

А. Залізо.

В. Молібден.

С. Цинк.

D. Магній.

Е. *Кобальт.

42. Інститут геронтології

людям літнього віку ра-

дить вживати комплекс

вітамінів, що містить

вітамін Е. Яку головну

функцію він виконує ?

А. Антискорбутну.

В. Антигеморагічну.

С. Антидерматитну.

D. Антиневритну.

Е. *Антиоксидантну.

43. Після курсу терапії

хворому з виразкою

дванадцятипалої кишки

лікар пропонує вживати

соки з капусти та картоплі.

Вміст якої речовини у цих

овочах сприяє профілак-

тиці та загоєнню виразок ?

А. Вітаміну К.

В. Пантотенової кислоти.

С. Вітаміну С.

D. Вітаміну В1.

Е. *Вітаміну U.

44. У хворого різко підви-

щилася кровоточивість

ясен. Які вітаміни слід

призначити цьому

пацієнту ?

A. Біотин, пантотенову

кислоту.

B. В1, В2.

C. А, Е.

D. РР, В12.

E. *С, К.

45. У хворого з

карциномою спостеріга-

ється авітаміноз PP, у

крові різко підвищена

концентрація серотоніну.

Page 93: КРОК-1 БІОХІМІЯ Збірник тестових завданьlib.sumdu.edu.ua/library/docs/rio/2020/m4695.pdf · 2 Крок-1. Біохімія Н: збірник тестових

93

Що є причиною такого

стану ?

A. Серотонін сприяє

зменшенню вмісту РР в

організмі.

B. Порушене використання

серотоніну в організмі.

C. Спостерігається алімен-

тарна нестача РР.

D. Спостерігається підви-

щене використання РР.

E. *Триптофан переважно

перетворюється в серотонін.

46. Хворому з гіпоплазією

твердих тканин зуба

стоматолог призначив

вітаміни А і D перорально.

На чому базується тактика

лікування ?

A. Забезпечують антиокси-

дантні властивості тканин

зуба.

B. Ці вітаміни сприяють

перетворенню проколагену в

колаген, що приводить до

ремінералізації.

C. Ці вітаміни активують

енергетичний обмін у

тканинах зуба.

D. Сприяють заміні

стронцієвого апатиту

гідроксиапатитом.

E. *Ці вітаміни регулюють

обмін гетерополісахаридів у

тканинах зуба і сприяють

відкладанню солей кальцію.

47. У дитини спосте-

рігається затримка

прорізування зубів, непра-

вильне їх розташування.

Скарги також на сухість у

роті, появу тріщин у

куточках рота з

подальшим нагноєнням. З

недостатністю якого

вітаміну це може бути

пов’язано ?

A. К.

B. D.

C. Е.

D. С.

E. *А.

48. У жінки 35 років із

хронічним захворюванням

нирок розвинувся остеопо-

роз. Назвіть, дефіцит якої з

нижченаведених речовин є

основною причиною цього

ускладнення:

A. Холестерин.

B. 25-OH-D3.

C. D3.

D. D2.

E. *1,25(OH)2 D3.

49. Після епілептиформно-

го нападу педіатром була

Page 94: КРОК-1 БІОХІМІЯ Збірник тестових завданьlib.sumdu.edu.ua/library/docs/rio/2020/m4695.pdf · 2 Крок-1. Біохімія Н: збірник тестових

94

оглянута грудна дитина,

яка отримувала штучну

їжу. У дитини виявлений

дерматит. При лаборатор-

ному обстеженні встанов-

лено зниження активності

аланін- і аспартааміно-

трансферази еритроцитів.

Недостатність якого

вітаміну можна

запідозрити ?

A. Кальциферолу.

B. Аскорбінової кислоти.

C. Кобаламіну.

D. Рибофлавіну.

E. *Піридоксину.

50. У 6-місячної дитини

спостерігалися часті та

сильні підшкірні

кровотечі. Призначення

синтетичного аналога

вітаміну K (вікасолу) дало

позитивний ефект. У γ-

карбоксилюванні глута-

мінової кислоти якого з

нижченаведених білків

згортальної системи крові

бере участь цей вітамін ?

A. Фактора Розенталя.

B. Фібриногену.

C. Фактора Хагемана.

D. Антигемофільного глобу-

ліну A.

E. *Протромбіну.

51. При обтураційній

жовтяниці часто спостері-

гається протромбінова

недостатність. Із дефіци-

том якого вітаміну це

пов’язано ?

A. Е.

B. В6.

C. А.

D. С.

E. *К.

52. У хворого спостерігаю-

ться дерматит, діарея,

деменція. Встановлено, що

основним продуктом

харчування хворого є

кукурудза. Дані

порушення пов’язані з

нестачею вітаміну:

A. В12.

B. В1.

C. В2.

D. В9.

E. *РР.

53. Людина захворіла на

пелагру. При опитуванні

стало відомо, що протягом

тривалого часу вона

харчувалася переважно

кукурудзою, вживала мало

м’яса. Що стало причиною

розвитку пелагри ?

Page 95: КРОК-1 БІОХІМІЯ Збірник тестових завданьlib.sumdu.edu.ua/library/docs/rio/2020/m4695.pdf · 2 Крок-1. Біохімія Н: збірник тестових

95

A. Дефіцит гістидину в

кукурудзі.

B. Дефіцит тирозину в

кукурудзі.

C. Дефіцит проліну в

кукурудзі.

D. Дефіцит аланіну в

кукурудзі.

E. *Дефіцит триптофану у

кукурудзі.

54. До лікаря звернувся

хворий із симетричним

дерматитом відкритих

ділянок шкіри. З розмови з

пацієнтом встановлено, що

він харчується в

основному крупами і

споживає мало м’яса,

молока та яєць. Дефіцит

якого з наведених

вітамінів спостерігається у

даного пацієнта ?

A. Токоферолу.

B. Кальциферолу.

C. Фолієвої кислоти.

D. Біотину.

E. *Нікотинаміду.

55. Після видалення 2/3

шлунка в крові пацієнта

зменшилася кількість

еритроцитів, збільшився їх

об’єм, знизився рівень

гемоглобіну. Дефіцит

якого вітаміну призводить

до таких змін картини

крові ?

А. РР.

В. С.

С. Р.

D. В6.

Е. *В12.

56. При якому гіпо-

вітамінозі спостерігаються

одночасне порушення

репродуктивної функції і

дистрофія скелетних

м’язів ?

А. В1.

В. А.

С. К.

D. D.

Е. *Е.

57. У моркві, гарбузі та ін-

ших червоних овочах міс-

тяться каротини. Недо-

статність якого вітаміну

компенсують ці рослинні

пігменти?

A. Кальциферолу.

B. Нафтохінону.

C. Рибофлавіну.

D. Токоферолу.

E. *Ретинолу.

58. У новонародженої

дитини з’явилися

Page 96: КРОК-1 БІОХІМІЯ Збірник тестових завданьlib.sumdu.edu.ua/library/docs/rio/2020/m4695.pdf · 2 Крок-1. Біохімія Н: збірник тестових

96

симптоми геморагічної

хвороби у зв’язку з

гіповітамінозом К. Розви-

ток захворювання зумов-

лений особливою біологіч-

ною роллю вітаміну К,

який:

A. Інгібує синтез гепарину.

B. Є кофактором

протромбіну.

C. Є специфічним

інгібітором антитромбінів.

D. Впливає на протео-

літичну активність тромбіну.

E. *Є кофактором -глута-

маткарбоксилази.

59. У новонародженого

спостерігалися судоми, які

зникали після призначен-

ня вітаміну B6. Цей ефект,

найбільш ймовірно,

викликаний тим, що

вітамін B6 бере участь в

утворенні:

A. Нікотинаміду.

B. Замінних амінокислот.

C. Гему.

D. Гістаміну.

E. *γ-Аміномасляної

кислоти (ГАМК).

60. У чоловіка 45 років

через 3 роки після операції

видалення частини

шлунка вміст еритроцитів

в крові становить 2,01012,

Hb – 85 г/л, кольоровий

показник – 1,27. Пору-

шення всмоктування

якого вітаміну викликало

зміни еритропоезу ?

А. С.

В. Р.

С. А.

D. В6.

Е. *В12.

61. За результатами ана-

лізу шлункового соку

хворого встановлено:

загальна кислотність –

28 ммоль/л, вільна соляна

кислота – 1,5 ммоль/л,

вміст гастромукопротеїну

знижений. Недостатність

якого вітаміну спостері-

гається в організмі ?

А. Пантотенової кислоти.

В. Нікотинаміду.

С. Біофлавоноїдів.

D. Фолієвої кислоти.

Е. *Ціанокобаламіну.

62. При злоякісній

карциномі кишечника

близько 60 % триптофану

окиснюється за серото-

ніновим шляхом. В якому

Page 97: КРОК-1 БІОХІМІЯ Збірник тестових завданьlib.sumdu.edu.ua/library/docs/rio/2020/m4695.pdf · 2 Крок-1. Біохімія Н: збірник тестових

97

вітаміні зростає потреба у

хворого ?

А. Фолієва кислота.

В. Пантотенова кислота.

С. Рибофлавін.

D. Піридоксин.

Е. *Нікотинова кислота.

63. У хворих на алкоголізм

часто спостерігається гіпо-

вітаміноз В1, що є

результатом нераціональ-

ного харчування. Симпто-

мами гіповітамінозу В1 є

розлади нервової системи,

психози, втрата пам’яті.

Чому до дефіциту В1

особливо чутливі клітини

нервової тканини ?

А. Порушується окиснення

жирних кислот.

В. Посилюється ліполіз

жирової тканини.

С. Підвищується інтенсив-

ність гліколізу.

D. Знижується інтенсив-

ність гліколізу.

Е. *Порушується аеробний

розпад глюкози.

64. У хворого виявлений

підвищений вміст пірувату

в крові. Нестача якого

вітаміну спостерігається у

хворого ?

А. Біотину.

В. РР.

С. Пантотенової кислоти.

D. В2.

Е. *В1.

65. У хворого 43 років

спостерігаються хронічний

атрофічний гастрит,

мегалобластична гіпер-

хромна анемія, Підвищене

виділення метилмалонової

кислоти із сечею.

Недостатністю якого

вітаміну зумовлене

виникнення зазначених

симптомів ?

А. В1.

В. В2.

С. В5.

D. В3.

Е. *В12.

66. У чоловіка 39 років

спостерігається підвище-

ний ризик розвитку

інфекційних процесів,

гіперкератоз, порушення

сутінкового зору. Який

вітамінний препарат

необхідно призначити ?

А. Піридоксину

гідрохлорид.

В. Токоферолу ацетат.

С. Рибофлавін.

Page 98: КРОК-1 БІОХІМІЯ Збірник тестових завданьlib.sumdu.edu.ua/library/docs/rio/2020/m4695.pdf · 2 Крок-1. Біохімія Н: збірник тестових

98

D. Ергокальциферол.

Е. *Ретинолу ацетат.

67. У дитини першого року

життя під час

профілактичного огляду

виявлено порушення

мінералізації кісток.

Нестача якого вітаміну

може бути причиною

цього ?

А. Рибофлавін.

В. Фолієва кислота.

С. Токоферол.

D. Кобаламін.

Е. *Кальциферол.

68. Який із перелічених

вітамінних препаратів має

виражену радіо-

протекторну дію, зумов-

лену антиоксидантною

активністю ?

А. Тіаміну гідрохлорид.

В. Ергокальциферол.

С. Фолієва кислота.

D. Рибофлавін.

Е. *Токоферолу ацетат.

1.10. ОБМІН

ПОРФІРИНІВ.

БІОХІМІЯ КРОВІ

1. У немовляти унаслідок

неправильного году-

вування виник виражений

пронос. Одним із його

наслідків є екскреція

великої кількості натрію

гідрокарбонату. Яка

форма порушення

кислотно-основного стану

спостерігається в цьому

випадку ?

А. Не порушено кислотно-

основний стан.

В. Метаболічний алкалоз.

С. Респіраторний ацидоз.

D. Респіраторний алкалоз.

Е. *Метаболічний ацидоз.

2. Мати звернулася до

лікаря: у дитини 5 років

під дією сонячних

променів на шкірі

з’являються еритеми,

везикулярний висип,

свербіж. Дослідження

крові виявили зменшення

вмісту заліза в сироватці

крові, збільшення

виділення із сечею

уропорфіриногену І.

Page 99: КРОК-1 БІОХІМІЯ Збірник тестових завданьlib.sumdu.edu.ua/library/docs/rio/2020/m4695.pdf · 2 Крок-1. Біохімія Н: збірник тестових

99

Найбільш імовірною

спадковою патологією є:

А. Копропорфірія.

В. Метгемоглобінемія.

С. Печінкова порфірія.

D. Інтермітуюча порфірія.

Е. *Еритропоетична

порфірія.

3. У пацієнта візуально

виявлено пухирі та поси-

лену пігментацію після

впливу УФ-променів. Сеча

після стояння набуває

червоного кольору. Вияв-

лення в сечі якого з

перелічених показників

дасть змогу діагностувати

хворобу Гюнтера ?

А. Ацетону.

В. Гемоглобіну.

С. Білірубіну.

D. Креатиніну.

Е. *Уропорфіриногену І.

4. У сироватці крові па-

цієнта виявлено

підвищення концентрації

оксипроліну, сіалових

кислот, С-реактивного

білка. Загострення якої

патології спостерігається ?

А. Ентероколіту.

В. Панкреатиту.

С. Гепатиту.

D. Бронхіту.

Е. *Ревматизму.

5. У процесі катаболізму

гемоглобіну вивіль-

няється залізо, яке в складі

спеціального транспорт-

ного білка надходить у

кістковий мозок і знову

використовується для

синтезу гемоглобіну. Цим

транспортним білком є:

А. Альбумін.

В. Транскобаламін.

С. Гаптоглобін.

D. Церулоплазмін.

Е. *Трансферин.

6. При дії окисників (Н2О2,

оксиди нітрогену та ін.),

гемоглобін утворює

сполуку, до складу якої

входить Fe3,+ та вона не

здатна переносити кисень.

Як називається ця

сполука ?

А. Карбгемоглобін.

В. Карбоксигемоглобін.

С. Глікозильований гемо-

глобін.

D. Оксигемоглобін.

Е. *Метгемоглобін.

7. У хворого діагностовано

α-таласемію. Які порушен-

Page 100: КРОК-1 БІОХІМІЯ Збірник тестових завданьlib.sumdu.edu.ua/library/docs/rio/2020/m4695.pdf · 2 Крок-1. Біохімія Н: збірник тестових

100

ня спостерігаються в

синтезі гемоглобіну при

цьому захворюванні ?

А. Пригнічення синтезу

δ- та β-ланцюгів.

В. Пригнічення синтезу

β-ланцюгів.

С. Пригнічення синтезу

γ-ланцюгів.

D. Пригнічення синтезу

δ-ланцюгів.

Е. *Пригнічення синтезу

α-ланцюгів.

8. У хворого швидко

розвиваються набряки.

Зниження вмісту яких

білків сироватки крові

призводить до їх появи ?

А. α1-Глобулінів.

В. Фібриногену.

С. α2-Глобулінів.

D. β-Глобулінів.

Е. *Альбумінів.

9. У крові хворого вияв-

лено значне збільшення

активності МВ-ізоформи

креатинфосфокінази та

ЛДГ1. Назвіть можливу

патологію:

А. Гепатит.

В. Холецистит.

С. Ревматизм.

D. Панкреатит.

Е. *Інфаркт міокарда.

10. На основі клінічних

даних хворому встанов-

лено попередній діагноз:

гострий панкреатит.

Назвіть біохімічний тест,

який підтверджує цей

діагноз:

А. Рівень креатиніну в крові.

В. Активність кислої фосфа-

тази крові.

С. Активність лужної

фосфатази крові.

D. Активність амінотрансфе-

раз крові.

Е. *Активність амілази

крові.

11. Пацієнт 33 років хворіє

10 років. Періодично звер-

тається до лікаря зі

скаргами на гострий біль у

животі, судоми, порушення

зору. У його родичів

спостерігаються подібні

симптоми. Сеча червоного

кольору. Госпіталізовано з

діагнозом: гостра переміж-

на порфірія. Причиною

захворювання може бути

порушення біосинтезу:

А. Інсуліну.

В. Колагену.

С. Жовчних кислот.

Page 101: КРОК-1 БІОХІМІЯ Збірник тестових завданьlib.sumdu.edu.ua/library/docs/rio/2020/m4695.pdf · 2 Крок-1. Біохімія Н: збірник тестових

101

D. Простагландинів.

Е. *Гему.

12. Людина у стані спокою

штучно примушує себе

дихати часто і глибоко

упродовж 3-4 хв. Чи

змінюється кислотно-

основний стан організму ?

А. Кислотно-основний стан

не змінюється.

В. Виникає дихальний

ацидоз.

С. Виникає метаболічний

алкалоз.

D. Виникає метаболічний

ацидоз.

Е. *Виникає дихальний

алкалоз.

13. Молекулярний аналіз

гемоглобіну пацієнта з

анемією виявив заміну

6Глу на 6Вал β-ланцюга.

Який молекулярний

механізм патології ?

А. Трансдукція генів.

В. Хромосомна мутація.

С. Геномна мутація.

D. Ампліфікація генів.

Е. *Генна мутація.

14. У хворої 38 років

ревматизм в активній фазі.

Визначення якого з пере-

лічених лабораторних

показників сироватки

крові має діагностичне

значення при цій

патології ?

А. Сечовини.

В. Сечової кислоти.

С. Трансферину.

D. Креатиніну.

Е. *С-реактивного білка.

15. У чоловіка 70 років

спостерігається підвище-

ний рівень згортання

крові, зумовлений

насамперед зниженням у

плазмі крові вмісту:

А. Вітаміну D.

В. Альбуміну.

С. Імуноглобуліну А.

D. Кальцію.

Е. *Антитромбіну.

16. Які компоненти

фракції залишкового азоту

переважають у крові при

продукційних азотеміях ?

А. Кетонові тіла, білки.

В. Ліпіди, вуглеводи.

С. Сечова кислота, холін.

D. Порфірини, білірубін.

Е. *Амінокислоти, сечовина.

17. У хворого вміст калію в

плазмі крові становить

Page 102: КРОК-1 БІОХІМІЯ Збірник тестових завданьlib.sumdu.edu.ua/library/docs/rio/2020/m4695.pdf · 2 Крок-1. Біохімія Н: збірник тестових

102

8 ммоль/л. При цьому

стані спостерігаються:

А. Підвищення артеріаль-

ного тиску.

В. Тахікардія.

С. Брадикардія.

D. Зниження артеріального

тиску.

Е. *Аритмії, зупинки серця.

18. У хворого відмічається

підвищена чутливість

шкіри до сонячних

променів. При стоянні сеча

набуває темно-червоного

кольору. Яка найбільш

імовірна причина такого

стану ?

А. Алкаптонурія.

В. Гемолітична жовтяниця.

С. Альбінізм.

D. Пелагра.

Е. *Порфірія.

19. У хворого знижені

рівень рН крові та вміст

гідрокарбонатних іонів

(зниження лужного резер-

ву крові), підвищений

вміст молочної та піро-

виноградної кислот у крові

та сечі. Який тип

порушення кислотно-

основного стану

спостерігається ?

А. Метаболічний алкалоз.

В. Респіраторний ацидоз.

С. Дихальний алкалоз.

D. Респіраторний алкалоз.

Е. *Метаболічний ацидоз.

20. При підвищенні кон-

центрації чадного газу в

повітрі може настати от-

руєння. При цьому пору-

шується транспортування

гемоглобіном кисню від

легень до тканин. Яке

похідне гемоглобіну при

цьому утворюється ?

А. Метгемоглобін.

В. Оксигемоглобін.

С. Гемохромоген.

D. Карбгемоглобін.

Е. *Карбоксигемоглобін.

21. У хворого, який

отримував антикоагулян-

ти непрямої дії, виявлено

зниження рівня протром-

біну з 0,15 до 0,05 г/л, що

бере участь у другій фазі

згортання крові –

утворенні тромбіну. Це

відбулося в результаті:

A. Зниженння кількості

глобулінів у крові.

B. Нестачі вітаміну В12.

C. Нестачі вітаміну С.

Page 103: КРОК-1 БІОХІМІЯ Збірник тестових завданьlib.sumdu.edu.ua/library/docs/rio/2020/m4695.pdf · 2 Крок-1. Біохімія Н: збірник тестових

103

D. Зниженння концентрації

Са2+.

E. *Нестачі вітаміну К.

22. Біохімічний аналіз

сироватки крові пацієнта з

хворобою Вільсона-Ко-

новалова виявив зниження

вмісту церулоплазміну.

Концентрація яких іонів

буде підвищена в

сироватці крові цього

хворого ?

A. Натрію.

B. Кальцію.

C. Фосфору.

D. Калію.

E. *Міді.

23. Молодий чоловік після

імплантації серцевого кла-

пана систематично

отримує непрямі анти-

коагулянти. Його стан

ускладнився кровотечею.

Зі зниженням концентрації

якої речовини у крові це

пов’язано ?

A. Церулоплазміну.

B. Гаптоглобіну.

C. Гепарину.

D. Креатину.

E. *Протромбіну.

24. У легенях вугільна кис-

лота Н2СО3 за допомогою

ферменту розкладається до

води та вуглекислого газу,

який виділяється з

повітрям. Який фермент

каталізує цю реакцію ?

A. Цитохромоксидаза.

B. Каталаза.

C. Пероксидаза.

D. Цитохром.

E. *Карбоангідраза.

25. Пролонгована дія ряду

антибіотиків і сульфаніл-

амідів обумовлена тим, що

вони циркулюють у крові

тривалий час у комплексі з

білком:

A. Гемопексином.

B. Трансферином.

C. Гемоглобіном.

D. Гаптоглобіном.

E. *Альбуміном.

26. Хворий 20 років

скаржиться на загальну

слабість, запаморочення,

швидку втомлюваність.

При обстеженні виявлено:

гемоглобін крові 80 г/л,

еритроцити зміненої

форми. Причиною може

бути:

A. Хвороба Аддісона.

Page 104: КРОК-1 БІОХІМІЯ Збірник тестових завданьlib.sumdu.edu.ua/library/docs/rio/2020/m4695.pdf · 2 Крок-1. Біохімія Н: збірник тестових

104

B. Паренхіматозна

жовтяниця.

C. Гостра переміжна

порфірія.

D. Обтураційна жовтяниця.

E. *Серпоподібноклітинна

анемія.

27. Пацієнт звернувся до

лікаря зі скаргами на

задишку, що виникала

після фізичного наванта-

ження. Клінічне обстежен-

ня виявило анемію та

наявність парапротеїну в

зоні -глобулінів. Який

показник у сечі необхідно

визначити для підтверд-

ження діагнозу мієломи ?

A. Антитрипсин.

B. Білірубін.

C. Гемоглобін.

D. Церулоплазмін.

E. *Білок Бенс-Джонса.

28. У результаті важкої

м’язової роботи у

робітника значно

зменшилася буферна

ємність крові. Надходжен-

ням якої кислої речовини

у кров можна пояснити це

явище ?

A. 3-Фосфогліцерату.

B. Пірувату.

C. 1,3-Бісфосфогліцерату.

D. α-Кетоглутарату.

E. *Лактату.

29. Жінка 62 років скар-

житься на біль у ділянці

груднини і хребта,

переломи ребер. Лікар

припустив мієломну

хворобу (плазмоцитому).

Який із перелічених нижче

лабораторних показників

буде мати найбільше діаг-

ностичне значення ?

A. Гіпопротеїнемія.

B. Гіперальбумінемія.

C. Протеїнурія.

D. Гіпоглобулінемія.

E. *Парапротеїнемія.

30. У чоловіка 58 років є

ознаки атеросклерозу.

Збільшення якого із

перелічених показників

біохімічного аналізу крові

найбільш характерне для

цього стану ?

A. Активності

сукцинатдегідрогенази.

B. Глікопротеїнів.

C. Рівня ЛВПЩ (α-ліпо-

протеїнів).

D. Активності аланінаміно-

трансферази.

Page 105: КРОК-1 БІОХІМІЯ Збірник тестових завданьlib.sumdu.edu.ua/library/docs/rio/2020/m4695.pdf · 2 Крок-1. Біохімія Н: збірник тестових

105

E. *Рівня ЛПНЩ (β-ліпо-

протеїнів).

31. У чоловіка 70 років

спостерігається підвище-

ний рівень згортання

крові, зумовлений у першу

чергу зниженням у плазмі

крові:

A. Вітаміну D.

B. Альбуміну.

C. Імуноглобуліну А.

D. Кальцію.

E. *Антитромбіну.

32. При тривалому голоду-

ванні у людей з’являють-

ся “голодні” набряки. Яка

причина їх появи ?

А. Зниження секреції вазо-

пресину.

В. Підвищення секреції

вазопресину.

С. Підвищення онкотичного

тиску крові.

D. Зниження осмотичного

тиску крові.

Е. *Зниження онкотичного

тиску крові.

33. У хворого на цукровий

діабет рН крові знизилося

до 7,3. Визначення

компонентів якої буферної

системи використовується

для діагностики порушень

кислотно-лужної

рівноваги ?

А. Білкової.

В. Фосфатної.

С. Оксигемоглобінової.

D. Гемоглобінової.

Е. *Бікарбонатної.

34. У приймальне відділен-

ня потрапила дитина

віком 1,5 року з ознаками

отруєння нітратами:

ціаноз, задишка, судоми.

Який механізм обумовлює

появу цих симптомів ?

А. Утворення карбокси-

гемоглобіну.

В. Утворення редукованого

гемоглобіну.

С. Утворення

карбгемоглобіну.

D. Утворення

оксигемоглобіну.

Е. *Утворення

метгемоглобіну.

35. У хворого відзна-

чається підвищена

чутливість шкіри до

сонячного світла. Його

сеча при тривалому

стоянні набуває темно-

червоного кольору. Яка

Page 106: КРОК-1 БІОХІМІЯ Збірник тестових завданьlib.sumdu.edu.ua/library/docs/rio/2020/m4695.pdf · 2 Крок-1. Біохімія Н: збірник тестових

106

найбільш імовірна

причина такого стану ?

А. Гемолітична жовтяниця.

В. Алкаптонурія.

С. Альбінізм.

D. Пелагра.

Е. *Порфірія.

36. У групі дітей, які їли

солодкий кавун, у двох

з’явилися ознаки

отруєння: значна слабість,

запаморочення, головний

біль, блювання, задишка,

тахікардія, синюшність

губ, вух, кінчиків пальців.

Лабораторний аналіз

кавуна показав високий

вміст нітратів. Який

основний механізм у

патогенезі отруєння тільки

у двох дітей ? А. Недостатність каталази.

В. Недостатність

супероксиддисмутази.

С. Недостатність глутатіон-

пероксидази.

D. Недостатність

цитохромоксидази.

Е. *Недостатність мет-Нb-

редуктази.

37. У хворого, який

страждає на спадкову

еритропоетичну порфірію,

відзначена світлочутли-

вість шкіри. Накопичен-

ням якої сполуки в

клітинах шкіри це

обумовлено ?

А. Гему.

В. Уропорфіриногену ІІ.

С. Копропорфіриногену ІІІ.

D. Протопорфірину.

Е. *Уропорфіриногену І.

38. У людини, яка

виконувала важку фізичну

роботу в умовах

підвищеної температури

навколишнього середо-

вища, змінилася кількість

білків плазми крові. Що

саме має місце у даному

випадку ?

А. Диспротеїнемія.

В. Абсолютна гіпо-

протеїнемія.

С. Абсолютна гіпер-

протеїнемія.

D. Парапротеїнемія.

Е. *Відносна гіпер-

протеїнемія.

39. У хворого струс

головного мозку, що

супроводжується повтор-

ним блюванням та

задишкою. В артеріальній

крові: рН - 7,62; рСО2 –

Page 107: КРОК-1 БІОХІМІЯ Збірник тестових завданьlib.sumdu.edu.ua/library/docs/rio/2020/m4695.pdf · 2 Крок-1. Біохімія Н: збірник тестових

107

40 мм рт. ст. Яке

порушення кислотно-

основного стану у

хворого ?

А. Кислотно-основний стан

не порушується.

В. Негазовий ацидоз.

С. Газовий алкалоз.

D. Газовий ацидоз.

Е. *Негазовий алкалоз.

40. Споживання пацієнтом

упродовж тривалого часу

забруднених овочів та

фруктів призвело до

отруєння нітратами. Яке

похідне гемоглобіну

утворилося у крові даного

хворого ?

А. HbCN.

В. HbNHCOOH.

С. HbO2.

D. HbCO.

Е. *Hb-OH.

41. При цукровому діабеті

внаслідок активації

процесів окиснення

жирних кислот виникає

кетоз. До яких порушень

кислотно-лужної рівно-

ваги може призвести

надмірне накопичення

кетонових тіл у крові ?

А. Дихальний ацидоз.

В. Зміни не відбудуться.

С. Метаболічний алкалоз.

D. Дихальний алкалоз.

E. *Метаболічний ацидоз.

42. У хворого виявлена

підвищена чутливість

шкіри до сонячного світла.

При стоянні сеча набуває

темно-червоного кольору.

Яка найбільш імовірна

причина такого стану ?

A. Алкаптонурія.

B. Гемолітична жовтяниця.

C. Альбінізм.

D. Пелагра.

E. *Порфірія.

1.11. БІОХІМІЯ

ТРАВНОЇ СИСТЕМИ

1 Глікоген, що надійшов із

їжею, гідролізувався у

шлунково-кишковому

тракті. Який кінцевий

продукт утворився в

результаті цього процесу ?

A. Фруктоза.

B. Лактат.

C. Лактоза.

D. Галактоза.

E. *Глюкоза.

Page 108: КРОК-1 БІОХІМІЯ Збірник тестових завданьlib.sumdu.edu.ua/library/docs/rio/2020/m4695.pdf · 2 Крок-1. Біохімія Н: збірник тестових

108

2. У хворого в порції

шлункового соку виявлено

лактат. Назвіть причину

його появи:

A. Недостатність реніну.

B. Надлишок HCl.

C. Недостатність пепсину.

D. Недостатність

гастриксину.

E. *Недостатність HCl.

3. У хворого 30 років із

гострим запаленням під-

шлункової залози –

панкреатиом – виявлено

порушення порожнинного

травлення білків. Це може

бути пов’язано із

недостатнім синтезом та

виділенням залозою:

A. Протромбіну.

B. Пепсину.

C. Ліпази.

D. Дипептидаз.

E. Амілази.

E. *Трипсину.

4. У клініку доставлена

дитина, у якої спостері-

гаються блювання, втрата

свідомості, судоми. У крові

виявлено дуже високий

вміст цитруліну.

Попередній діагноз –

спадкова цитрулінемія.

Яку дієту можна

рекомендувати такому

хворому ?

A. З низьким вмістом ліпідів

і вуглеводів.

B. З високим вмістом

вуглеводів і білків.

C. З високим вмістом ліпідів

і вуглеводів.

D. З високим вмістом білків

і ліпідів.

E. *З низьким вмістом

білків.

5. У клініку потрапив

хворий з такими характер-

ними симптомами:

слабість, гіпотонія м’язів,

слабі паралічі, втрата

апетиту, послаблена

перистальтика, тахікардія.

Яке порушення електро-

літного обміну можна

передбачити у цього

хворого ?

A. Гіпернатріємія.

B. Гіперкаліємія.

C. Гіпонатріємія.

D. Гіперфосфатемія.

E. *Гіпокаліємія.

6. У хворого 35 років, який

часто вживає алкоголь, на

фоні лікування сечогін-

ними засобами виникли

Page 109: КРОК-1 БІОХІМІЯ Збірник тестових завданьlib.sumdu.edu.ua/library/docs/rio/2020/m4695.pdf · 2 Крок-1. Біохімія Н: збірник тестових

109

сильна м'язова і серцева

слабкість, блювання,

діарея, депресія. АТ -

100/60 мм. рт. ст.

Причиною такого стану є

посилене виділення із

сечею:

A. Фосфатів.

B. Натрію.

C. Хлору.

D. Кальцію.

E. *Калію.

7. У чоловіка, який трива-

лий час не вживав із їжею

жирів, але отримував

достатню кількість

вуглеводів і білків,

виявлено дерматит, погане

загоювання ран, по-

гіршення зору. Яка

можлива причина пору-

шення обміну речовин ?

A. Нестача олеїнової

кислоти.

B. Нестача пальмітинової

кислоти.

C. Нестача вітамінів РР, Н.

D. Низька калорійність

дієти.

E. *Нестача лінолевої

кислоти, вітамінів А, D, Е,

К.

8. Скарги та об’єктивні

дані дозволяють припу-

стити наявність у хворого

запального процесу в

жовчному міхурі, пору-

шення колоїдних власти-

востей жовчі, ймовірність

утворення жовчних

каменів. Що може

головним чином спричи-

нити їх утворення ?

A. Фосфати.

B. Урати.

C. Оксалати.

D. Хлориди.

E. *Холестерин.

9. У пацієнта віком 40

років непереносимість

молочних продуктів.

Недостатністю якого

ферменту травлення

можна пояснити це

явище ?

A. Амілази.

B. Лактатдегідрогенази.

C. Мальтази.

D. Ліпази.

E. *Лактази.

10. Недостатня секреція

якого ферменту зумовлює

неповне перетравлювання

жирів у шлунково-

кишковому тракті та

Page 110: КРОК-1 БІОХІМІЯ Збірник тестових завданьlib.sumdu.edu.ua/library/docs/rio/2020/m4695.pdf · 2 Крок-1. Біохімія Н: збірник тестових

110

появу великої кількості

нейтральних жирів у кало-

вих масах ?

A. Пепсину.

B. Фосфоліпази.

C. Ентерокінази.

D. Амілази.

E. *Панкреатичної ліпази.

11. У хворого із хронічним

гепатитом виявлено

значне зниження синтезу і

секреції жовчних кислот.

Який процес у кишечнику

порушений у цього

хворого ?

A. Всмоктування

амінокислот.

B. Перетравлення білків.

C. Перетравлення

вуглеводів.

D. Всмоктування гліцерину.

E. *Емульгування жирів.

12. До лікарні прийнятий

чоловік 40 років з

діагнозом хронічний

гастрит. Порушення пере-

травлення в шлунку яких

нутрієнтів є характерною

ознакою цієї патології ?

A. Тригліцеридів.

B. Фосфоліпідів.

C. Крохмалю.

D. Лактози.

E. *Білків.

13. У слині міститься

-амілаза, здатна розщеп-

лювати поживні речовини.

На які субстрати може

діяти цей фермент ?

A. Хромопротеїни

B. Ліпіди.

C. Прості білки.

D. Нуклеопротеїни.

E. *Вуглеводи.

14. Після вживання жирної

їжі у хворого з’являються

нудота і печія, має місце

стеаторея. Причиною

такого стану може бути:

A. Порушення синтезу

фосфоліпази.

B. Підвищення виділення

ліпази.

C. Порушення синтезу

трипсину.

D. Недостатність амілази.

E. *Недостатність жовчних

кислот.

15. Хворий скаржиться на

зниження ваги, біль у

ділянці шлунка після

вживання їжі, при аналізі

шлункового соку загальна

кислотність 20 ОД.

Перетравлення яких

Page 111: КРОК-1 БІОХІМІЯ Збірник тестових завданьlib.sumdu.edu.ua/library/docs/rio/2020/m4695.pdf · 2 Крок-1. Біохімія Н: збірник тестових

111

компонентів їжі порушено

в першу чергу ?

A. Крохмалю.

B. Фосфоліпідів.

C. Нейтральних жирів.

D. Олігосахаридів.

E. *Білків.

16. Перетравлення білків у

шлунку є початковою

стадією розщеплення

білків у травному каналі

людини. Назвіть

ферменти, які

розщеплюють білки у

шлунку:

A. Карбоксипептидаза та

амінопептидаза.

B. Трипсин та катепсини.

C. Хімотрипсин та лізоцим.

D. Ентеропептидаза та

еластаза.

E. *Пепсин та гастриксин.

17. В організмі людини

хімотрипсиноген секре-

тується підшлунковою

залозою і в порожнині

кишечника перетво-

рюється на активний

хімотрипсин шляхом

обмеженого протеолізу під

дією:

A. Карбоксипептидази.

B. Ентерокінази.

C. Пепсину.

D. Амінопептидази.

E. *Трипсину.

18. Хворого з явищами

енцефалопатії госпіта-

лізували в неврологічний

стаціонар. Виявили

кореляцію між наро-

станням енцефалопатії і

збільшенням концентрації

речовин, що надходять з

кишечника у кров'яне

русло’ Яка сполука, що

утворюється в кишеч-

нику, може спричинити

ендотоксимію ?

A. Орнітин.

B. Бутират.

C. Ацетоацетат.

D. Біотин.

E. *Індол.

19. Активація якого проце-

су в пухлинних клітинах є

найбільш вірогідною

причиною утворення в

шлунку молочної

кислоти ?

A. Глюконеогенезу.

B. Пентозофосфатного

шляху.

C. β-Окиснення вищих

жирних кислот.

Page 112: КРОК-1 БІОХІМІЯ Збірник тестових завданьlib.sumdu.edu.ua/library/docs/rio/2020/m4695.pdf · 2 Крок-1. Біохімія Н: збірник тестових

112

D. Аеробного окиснення

глюкози.

E. *Анаеробного окиснення

глюкози.

20. У хворого в сечі під-

вищена амілазна

активність і виявлена

наявність трипсину, в

крові підвищена амілазна

активність. Про патологію

якого органу це свідчить ?

A. Кишечника.

B. Печінки.

C. Шлунка.

D. Нирок.

E. *Підшлункової залози.

21. На основі клінічних

даних хворому встановили

попередній діагноз –

гострий панкреатит.

Назвіть біохімічний тест,

який підтверджує цей

діагноз:

A. Рівень креатиніну в крові.

B. Активність кислої

фосфатази крові.

C. Активність лужної

фосфатази крові.

D. Активність амінотрансфе-

раз крові.

E. *Активність амілази

крові.

22. При запальних

процесах у жовчному

міхурі порушуються

колоїдні властивості

жовчі, що може призвести

до утворення жовчних

каменів. Кристалізація

якої речовини є основною

причиною їх утворення ?

А. Уратів.

В. Фосфатів.

С. Оксалатів.

D. Хлоридів.

Е. *Холестеролу.

23. Піддослідному собаці

через зонд у порожнину

шлунка ввели 150 мл

м’ясного бульйону. Вміст

якої з наведених речовин

швидко збільшиться в

крові тварини ?

А. Нейротензину.

В. Інсуліну.

С. Соматостатину.

D. Вазоінтестинального

поліпептиду.

Е. *Гастрину.

24. Хворий відзначає часті

проноси, особливо після

вживання жирної їжі,

схуднення. Лабораторні

дослідження показали

наявність стеатореї, кал

Page 113: КРОК-1 БІОХІМІЯ Збірник тестових завданьlib.sumdu.edu.ua/library/docs/rio/2020/m4695.pdf · 2 Крок-1. Біохімія Н: збірник тестових

113

гіпохолічний. Що може

бути причиною такого

стану ?

А. Недостатність

панкреатичної фосфоліпази.

В. Запалення слизової

оболонки тонкої кишки.

С. Незбалансована дієта.

D. Недостатність

панкреатичної ліпази.

Е. *Обтурація жовчних

шляхів.

25. У хворого, який

тривалий час страждає на

хронічний ентероколіт,

після вживання молока

виникли метеоризм,

діарея, коліки. З нестачею

якого ферменту в

кишечнику це пов’язано ?

А. Сахараза.

В. Глікогенсинтетаза.

С. Мальтаза.

D. Амілаза.

Е. *Лактаза.

26. Які ферменти беруть

участь у перетравленні

білків у шлунку:

А. Хімотрипсин і лізоцим.

В. Трипсин і катепсини.

С. Карбоксипептидаза та

амінопептидаза.

D. Ентеропептидаза та

еластаза.

Е. *Пепсин і гастриксин.

1.12. БІОХІМІЯ ПЕЧІНКИ

ТА НИРОК

1. У новонародженого

фізіологічна жовтяниця.

Рівень вільного білірубіну

в крові значно перевищує

норму. Дефіцитом якого

ферменту це зумовлено ?

А. Гемоксигенази.

В. Трансамінази.

С. Ксантиноксидази.

D. Аденозиндезамінази.

Е. *УДФ-глюкуронілтранс-

ферази.

2. Пацієнт звернувся до

клініки зі скаргами на

загальну слабість, ниючий

біль у животі, поганий

апетит. Підозра на

жовтяницю. У сироватці

крові виявлено 77,3

мкмоль/л загального

білірубіну і 70,76 мкмоль/л

кон’югованого білірубіну.

Який найбільш імовірний

вид жовтяниці ?

А. Гемолітична жовтяниця.

В. Гострий гепатит.

Page 114: КРОК-1 БІОХІМІЯ Збірник тестових завданьlib.sumdu.edu.ua/library/docs/rio/2020/m4695.pdf · 2 Крок-1. Біохімія Н: збірник тестових

114

С. Цироз печінки.

D. Печінкова жовтяниця.

Е. *Механічна жовтяниця.

3. У пацієнта – цироз

печінки. Дослідження якої

з перелічених речовин, що

екскретуються із сечею,

може характеризувати

стан антитоксичної

функції печінки ?

А. Амінокислот.

В. Амонієвих солей.

С. Креатиніну.

D. Сечової кислоти.

Е. *Гіпурової кислоти.

4. Знешкодження ксено-

біотиків (лікарських

засобів, епоксидів,

ареноксидів, альдегідів,

нітропохідних тощо) та

ендогенних метаболітів

(естрадіолу, простагланди-

нів, лейкотрієнів) реалізує-

ться в печінці шляхом їх

кон’югації з:

А. Фосфоаденозином.

В. Аспарагіновою кислотою.

С. Гліцином.

D. S-Аденозилметіоніном.

Е. *Глутатіоном.

5. У людини порушений

процес синтезу сечовини.

Про патологію якого

органу це свідчить ?

A. Сечового міхура.

B. Нирок.

C. Мозку.

D. М'язів.

E.’*Печінки.

6. Кал хворого містить

велику кількість

нерозщепленого жиру і має

сірувато-білий колір.

Назвіть причину цього

явища:

A. Подразнення епітелію

кишечника.

B. Недостатня активація

пепсину соляною кислотою.

C. Гіповітаміноз.

D. Ентерит.

E. *Обтурація жовчної

протоки.

7. Хвора госпіталізована з

вираженою жовтяницею,

скаргами на відчуття

тяжкості в епігастральній

ділянці і правому

підребер’ї, шкірний

свербіж, втомлюваність,

дратівливість. При лабора-

торному обстеженні

виявлено: гіпербілірубіне-

мію з переважним

підвищенням кон’югова-

Page 115: КРОК-1 БІОХІМІЯ Збірник тестових завданьlib.sumdu.edu.ua/library/docs/rio/2020/m4695.pdf · 2 Крок-1. Біохімія Н: збірник тестових

115

ного білірубіну, тимолова

проба в нормі. Про яку

патологію свідчать ці

симптоми ?

A. Цироз печінки.

B. Гострий гепатит.

C. Гемолітичну жовтяницю.

D. Жовтяницю Криглера-

Найяра.

E. *Механічну жовтяницю.

8. У хворого з жовтяницею

виявлено: підвищення у

плазмі крові вмісту

загального білірубіну за

рахунок непрямого

(неконюгованого), в калі і

сечі – високий вміст

стеркобіліну, рівень

прямого (кон'югованого’

білірубіну в плазмі крові в

межах норми. Про який

вид жовтяниці можна

стверджувати ?

A. Хворобу Жильбера.

B. Паренхіматозну

(печінкову).

C. Механічну.

D. Жовтяницю

новонароджених.

E. *Гемолітичну.

9. У хворого після

переливання крові

спостерігається жовту-

ватість шкіри та слизових

оболонок, у крові

підвищений рівень

загального та непрямого

білірубіну, в сечі

підвищений рівень

уробіліну, в калі -

стеркобі–іну. Про який вид

жовтяниці це свідчить ?

A. Жовтяниця

новонароджених.

B. Спадкова жовтяниця.

C. Обтураційна жовтяниця.

D. Паренхіматозна

жовтяниця.

E. *Гемолітична жовтяниця.

10. У хворого встановлено

зниження синтезу вазопре-

сину, що призводить до

поліурії і, як наслідок, до

вираженої дегідратації

організму. Який ймовір-

ний механізм поліурії ?

A. Підвищення гідростатич-

ного тиску.

B. Порушення канальцевої

реабсорбції іонів Nа.

C. Зниження канальцевої

реабсорбції білка.

D. Порушення реабсорбції

глюкози.

E. *Зниження канальцевої

реабсорбції води.

Page 116: КРОК-1 БІОХІМІЯ Збірник тестових завданьlib.sumdu.edu.ua/library/docs/rio/2020/m4695.pdf · 2 Крок-1. Біохімія Н: збірник тестових

116

11. У хлопчика 9 років, що

перебуває на стаціонар-

ному лікуванні, виявлено

ураження нирок та

підвищений артеріальний

тиск. З підвищенням

кількості якого біологічно

активного пептиду

повязаний цей стан ?

A. Інсуліну.

B. Антидіуретичного

гормону.

C. Глюкагону.

D. Калідину.

E. * Ангіотензину ІІ.

12. У хворого в крові та

сечі виявлено високий

вміст індикану –

показника активації

процесів гниття білків у

кишечнику. Яка аміно-

кислота є джерелом

індикану ?

A. Гістидин.

B. Тирозин.

C. Пролін.

D. Фенілаланін.

E. *Триптофан.

13. У пацієнта спосте-

рігається жовте забарв-

лення шкіри, сеча темна,

кал темно-жовтого ко-

льору. Підвищення кон-

центрації якої речовини

буде спостерігатися в

сироватці крові ?

А. Кон’югованого

білірубіну.

В. Білівердину.

С. Мезобілірубіну.

D. Вердоглобіну.

Е. *Вільного білірубіну.

14. Для визначення анти-

токсичної функції печінки

хворому призначено бензо-

ат натрію, який у печінці

перетворюється на

гіпурову кислоту. Яка

сполука використовується

для цього ?

A. УДФ–глюкуронова

кислота.

B. Цистеїн.

C. Метіонін.

D. ФАФС.

E. *Гліцин.

15. У хворого виявлена

кетонурія. При якому

захворюванні в сечі

з’являються кетонові

тіла ?

A. Інфаркт нирки.

B. Гострий гломелуро-

нефрит.

C. Сечокам’яна хвороба.

D. Туберкульоз нирки.

Page 117: КРОК-1 БІОХІМІЯ Збірник тестових завданьlib.sumdu.edu.ua/library/docs/rio/2020/m4695.pdf · 2 Крок-1. Біохімія Н: збірник тестових

117

E. *Цукровий діабет.

16. При дослідженні

залишкового азоту крові

визначили, що азот

сечовини значно зниже-

ний. Про патологію якого

органа це свідчить ?

A. Шлунка.

B. Мозку.

C. Серця.

D. Кишечника.

E. *Печінки.

17. У чоловіка 32 років з

ураженням печінки при

проведенні проби Квіка на

детоксикаційну здатність

спостерігали низький

рівень у сечі: A. Амінокислот.

B. Оксипроліну.

C. Бензоату натрію.

D. Креатиніну.

E. *Гіпурової кислоти.

18. При обстеженні хворого

виявили застій жовчі в

печінці та жовчні камені в

жовчному міхурі. Який

основний компонент жовч-

них каменів ?

A. Мінеральні солі.

B. Тригліцериди.

C. Білірубінат кальцію.

D. Білок.

E. *Холестерин.

19. При аналізі крові у

хворого залишковий азот

становив 48 ммоль/л,

сечовина 15,3 ммоль/л.

Про захворювання якого

органа свідчать резуль-

тати аналізу ?

A. Селезінки.

B. Печінки.

C. Шлунка.

D. Кишечника.

E. *Нирок.

20. При аналізі крові у

хворого концентрація

альбумінів становить

28 г/л, підвищена

активність ЛДГ5.

Захворюванню якого

органа відповідає цей

аналіз ?

A. Селезінки.

B. Нирок.

C. Серця.

D. Легенів.

E. *Печінки.

21. В 70-ті роки XX ст.

вчені встановили, що

причиною жовтяниці

новонароджених є

порушення зв’язування

Page 118: КРОК-1 БІОХІМІЯ Збірник тестових завданьlib.sumdu.edu.ua/library/docs/rio/2020/m4695.pdf · 2 Крок-1. Біохімія Н: збірник тестових

118

білірубіну в гепатоцитах.

Яка речовина використо-

вується для утворення

кон’югату ?

A. Піровиноградна кислота.

B. Сечова кислота.

C. Сірчана кислота.

D. Молочна кислота.

E. * Глюкуронова кислота.

22. У хворого на цироз

печінки з’явилися

набряки. Яка можлива

причина їх виникнення ?

A. Зниження вмісту глюкози

в крові.

B. Зменшення вмісту в крові

гаптоглобіну.

C. Збільшення вмісту в крові

трансферину.

D. Збільшення вмісту в крові

гамма-глобулінів.

E. *Зменшення вмісту

альбумінів у крові.

23. У пацієнтки з

постійною гіпоглікемією

аналіз крові після

введення адреналіну

суттєво не змінився. Лікар

відмітив можливість пору-

шення в печінці. Про зміну

якої функції печінки може

йти мова ?

A. Екскреторної.

B. Холестеринутворюючої.

C. Кетогенної.

D. Гліколітичної.

E. *Глікогендепонуючої.

24. У працівника хім-

чистки виявлена жирова

дистрофія печінки.

Порушення синтезу якої

речовини в печінці може

призвести до даної

патології ?

A. Холевої кислоти.

B. Тристеарину.

C. Сечовини.

D. Фосфатидної кислоти.

E. *Фосфатидилхоліну.

25. У юнака 18 років з

ураженням паренхіми

печінки в сироватці крові

найвірогідніше буде

виявлено підвищений

рівень активності:

A. α-Амілази.

B. ЛДГ1.

C. Креатинкінази.

D. Кислої фосфатази.

E.*Аланінамінотрансферази.

26. У пацієнта 18 років при

лабораторному дослід-

женні виявлена наявність

глюкози в сечі при

нормальній концентрації її

Page 119: КРОК-1 БІОХІМІЯ Збірник тестових завданьlib.sumdu.edu.ua/library/docs/rio/2020/m4695.pdf · 2 Крок-1. Біохімія Н: збірник тестових

119

у плазмі крові. Найбільш

імовірною причиною цього

є порушення процесу:

А. Клубочкової фільтрації.

В. Канальцевої секреції.

С. Секреції глюкокортико-

їдів.

D. Секреції інсуліну.

Е. *Канальцевої

реабсорбції.

27. У чоловіка 38 років із

жовтяничною шкірою

спостерігаються анемія,

збільшення селезінки, гі-

пербілірубінемія (білірубін

непрямий), уробілінурія,

гіперхолічний кал. Для

якого стану найбільш

характерні ці зміни ?

А. Синдрому Жильбера.

В. Підпечінкової жовтяниці.

С. Паренхіматозної

жовтяниці.

D. Синдрому печінкової

недостатності.

Е. *Надпечінкової жовтя-

ниці.

28. Пацієнтку віком 52

років упродовж останніх

кількох днів турбують

напади болю у правому

підребер’ї після вживання

жирної їжі. Візуально

визначається пожовтіння

склер і шкіри, ахолічний

кал, сеча темного кольору.

Наявність якої речовини в

сечі пацієнтки обумовила

темний колір сечі при

обтураційній жовтяниці ?

А. Стеркобіліну.

В. Уробіліну.

С. Глюкози.

D. Кетонових тіл.

Е. *Білірубінглюкуронідів.

29. У хворого внаслідок

тривалого тютюнопаління

розвинувся рак легенів.

Яка канцерогенна

речовина міститься в

тютюновому димі і

належить до ПАВ

(поліненасичених арома-

тичних вуглеводнів) ?

А. β-Нафтиламін.

В. Диметиламіноазобензол.

С. Діетилнітрозамін.

D. Ортоаміноазотолуол.

Е. *Бензпірен.

30. Хворому три роки тому

було встановлено діагноз -

гломерулонефрит. За

останні 6 місяців

з’явилися набряки. Що

лежить в основі їх

розвитку ?

Page 120: КРОК-1 БІОХІМІЯ Збірник тестових завданьlib.sumdu.edu.ua/library/docs/rio/2020/m4695.pdf · 2 Крок-1. Біохімія Н: збірник тестових

120

А. Порушення білковосинте-

зуючої функції печінки.

В. Гіперпродукція вазопре-

сину.

С. Гіперальдостеронізм.

D. Гіперосмолярність плаз-

ми.

Е. *Протеїнурія.

31. У чоловіка 58 років

клінічна картина гострого

панкреатиту. Підвищення

в сечі якої речовини буде

підтверджувати діагноз?

А. Сечовини.

В. Сечової кислоти.

С. Альбуміну.

D. Залишкового азоту.

Е. *Амілази.

32. Хворому 65 років з

ознаками загального ожи-

ріння, ризиком жирової

дистрофії печінки реко-

мендована дієта, збагачена

ліпотропними речови-

нами. Вміст якої речовини

у раціоні має найбільше

значення у даному

випадку ?

А. Вітаміну С.

В. Глюкози.

С. Гліцину.

D. Холестерину.

Е. *Метіоніну.

33. У жінки 46 років, яка

страждає на жовчно-

кам’яну хворобу, розви-

нулася жовтяниця. При

цьому сеча стала темно-

жовтого кольору, а кал

знебарвлений. Концен-

трація якої речовини в

сироватці крові підви-

щиться найбільше ?

А. Вільного білірубіну.

В. Уробіліногену.

С. Білівердину.

D. Мезобілірубіну.

Е. *Кон’югованого біліру-

біну.

34. У юнака 16 років

діагностований спадковий

дефіцит УДФ-глюкуроніл-

трансферази. Лабораторно

визначається гіпер-

білірубінемія, зумовлена

переважно підвищенням у

крові концентрації:

А. Уробіліногену.

В. Прямого білірубіну.

С. Білівердину.

D. Стеркобіліногену.

Е. *Непрямого білірубіну.

35. У печінці хворого по-

рушена детоксикація при-

родних метаболітів і

Page 121: КРОК-1 БІОХІМІЯ Збірник тестових завданьlib.sumdu.edu.ua/library/docs/rio/2020/m4695.pdf · 2 Крок-1. Біохімія Н: збірник тестових

121

ксенобіотиків. Назвіть

цитохром, активність

якого може бути

зниженою:

А. Цитохром с1.

В. Цитохромоксидаза.

С. Гемоглобін.

D. Цитохром b.

Е. *Цитохром Р450.

36. У хворого встановлено

підвищення в плазмі крові

вмісту кон’югованого

(прямого) білірубіну на

фоні одночасного підви-

щення рівня некон’юго-

ваного (непрямого) біліру-

біну і різкого зниження в

калі і сечі вмісту

стеркобіліногену. Про

який вид жовтяниці

йдеться ?

А. Гемолітичну.

В. Паренхіматозну

(печінкову).

С. Хворобу Жильбера.

D. Жовтяницю немовлят.

Е. *Обтураційну.

1.13. БІОХІМІЯ МЯЗІВ

ТА СПОЛУЧНОЇ

ТКАНИНИ

1. Відомо, що синовіальна

рідина зменшує тертя

суглобових поверхонь. При

ревматизмі чи артриті її

в’язкість знижується

унаслідок деполімеризації

(руйнування):

А. Альбуміну.

В. Глікогену.

С. Колагену.

D. Гепарину.

Е. *Гіалуронової кислоти.

2. Активність яких

ферментів необхідно

визначити з діагностичною

і прогностичною метою,

якщо у хворого патологія

серцевого м’яза ?

A. Малатдегідрогенази,

піруватдегідрогенази.

B. Аргінази, пептидази,

фосфатази.

C. Лізоциму, цитратсинтази,

альдолази.

D. Нейрамінідази,

гексокінази, піруваткінази.

E. *Креатинкінази, АлАТ,

АсАТ.

Page 122: КРОК-1 БІОХІМІЯ Збірник тестових завданьlib.sumdu.edu.ua/library/docs/rio/2020/m4695.pdf · 2 Крок-1. Біохімія Н: збірник тестових

122

3. До косметолога

звернувся пацієнт із

проханням позбавити його

татуювання на плечі. Яка

речовина, що міститься в

сполучній тканині, обме-

жує поширення барвника і

робить можливим такий

вид “живопису” ?

A. Еластин.

B. -Глобулін.

C. Фібронектин.

D. Гепарин.

E. *Гіалуронова кислота.

4. Хвора 36 років страждає

на колагеноз. Збільшення

вмісту якого метаболіту

найбільш вірогідно буде

встановлено у сечі ?

A. Уробіліногену.

B. Iндикану.

C. Креатиніну.

D. Сечовини.

E. *Оксипроліну.

5. Хворому з прогресую-

чою м'язовою дистрофією

було проведено біохімічне

дослідження сечі. Виявлен-

ня якої речовини у великій

кількості в сечі може

підтвердити захворювання

м'язів у даного хворого ?

A. Креатиніну.

B. Порфіринів.

C. Сечовини.

D. Гіпурової кислоти.

E. *Креатину.

6. Назвіть фермент,

визначення активності

якого в крові є найбільш

інформативним у перші

години після виникнення

інфаркту міокарда ?

A. Глутаматдегідрогеназа.

B. Аспартатаміно-

трансфераза.

C. Аланінамінотрансфераза.

D. Лактатдегідрогеназа.

E. *Креатинфосфокіназа.

7. При парадонтозі відбу-

вається деструкція

білкових та полісахарид-

них компонентів сполучної

тканини. Який із

наведених білків входить

до складу сполучної

тканини ?

A. Антитрипсин.

B. Альбумін.

C. Трансферин.

D. Церулоплазмін.

E. *Колаген.

8. Підвищена ламкість су-

дин, руйнування емалі і

дентину зубів при цинзі

Page 123: КРОК-1 БІОХІМІЯ Збірник тестових завданьlib.sumdu.edu.ua/library/docs/rio/2020/m4695.pdf · 2 Крок-1. Біохімія Н: збірник тестових

123

обумовлені порушенням

дозрівання колагену. Який

етап модифікації прокола-

гену порушений при цьому

авітамінозі ?

A. Відщеплення N-кінцевого

пептиду.

B. Утворення поліпептидних

ланцюгів.

C. Глікозилювання

гідроксилізинових залишків.

D. Видалення з проколагену

С-кінцевого пептиду.

E. *Гідроксилювання

проліну.

9. Для роботи серцевого

м’яза необхідна енергія.

Який субстрат є основним

джерелом енергії в

працюючому м’язі ?

A. α-Кетоглутарова кислота.

B. Амінокислоти.

C. Молочна кислота.

D. Піровиноградна кислота.

E. *Жирні кислоти.

10. До клініки прийнята

дитина віком 1 рік із

ознаками ураження м'язів.

Після обстеження виявле-

ний дефіцит карнітину у

м'язах. Біохімічною

основою цієї патології є

порушення процесу:

A. Синтезу актину та

міозину.

B. Регуляції рівня Ca2+ в

мітохондріях.

C. Субстратного

фосфорилювання.

D. Утилізації молочної

кислоти.

E. *Транспорту вищих

жирних кислот до

мітохондрій.

11. До фібрилярних

елементів сполучної

тканини належать

колаген, еластин та

ретикулін. Назвіть аміно-

кислоту, яка входить

тільки до складу колагену

і визначення якої в

біологічних рідинах

використовується для

діагностики захворювань

сполучної тканини:

A. Фенілаланін.

B. Пролін.

C. Гліцин.

D. Лізин.

E. *Гідроксипролін.

12. У сироватці крові паці-

єнта встановлено підви-

щення активності

гіалуронідази. Визначення

якого біохімічного

Page 124: КРОК-1 БІОХІМІЯ Збірник тестових завданьlib.sumdu.edu.ua/library/docs/rio/2020/m4695.pdf · 2 Крок-1. Біохімія Н: збірник тестових

124

показника сироватки

крові дозволить підтвер-

дити припущення про

патологію сполучної

тканини ?

A. Галактози.

B. Білірубіну.

C. Сечової кислоти.

D. Глюкози.

E. *Сіалових кислот.

13. Після загоювання рани

на її місці утворився

рубець. Яка речовина є

основним компонентом

цього різновиду сполучної

тканини ?

A. Кератансульфат.

B. Еластин.

C. Гіалуронова кислота.

D. Хондроїтину сульфат.

E. *Колаген.

14. У хворого спостеріга-

ється атонія м’язів.

Назвіть фермент м’язової

тканини, активність якого

може бути знижена при

цьому:

A. Каталаза.

B. Амілаза.

C. Транскетолаза.

D. Глутамінтрансфераза.

E. *Креатинфосфокіназа.

15. У чоловіка 53 років

діагностована хвороба

Педжета. В добовій сечі

різко підвищений рівень

оксипроліну, що свідчить

передусім про посилення

розпаду:

A. Фібриногену.

B. Кератину.

C. Альбуміну.

D. Еластину.

E. *Колагену.

16. Чоловік 42 років

прийнятий у кардіологічне

відділення з діагнозом

стенокардія. До комплексу

препаратів, призначених

хворому, входить інгібітор

ферменту фосфодиесте-

рази. Концентрація якої

речовини в серцевому

м'язі буде збільшуватися ?

A. АТФ.

B. ГМФ.

C. АМФ.

D. АДФ.

E. * цАМФ.

17. Хвора 46 років

тривалий час страждає від

прогресуючої м’язової

дистрофії (хвороба

Дюшена). Зміни актив-

ності якого ферменту крові

Page 125: КРОК-1 БІОХІМІЯ Збірник тестових завданьlib.sumdu.edu.ua/library/docs/rio/2020/m4695.pdf · 2 Крок-1. Біохімія Н: збірник тестових

125

є діагностичним тестом у

цьому разі ?

А. Лактатдегідрогенази.

В. Аденілаткінази.

С. Піруватдегідрогенази.

D. Глутаматдегідрогенази.

Е. *Креатинфосфокінази.

18. До лікаря звернувся

пацієнт зі скаргами на біль

у суглобах. Візуально

помітні набряки і

почервоніння в ділянці

суглобів. Активність якого

ферменту необхідно

дослідити для зясування

діагнозу ?

А. Уреази.

В. Кислої фосфатази.

С. Лужної фосфатази.

D. Креатинкінази.

Е. *Гіалуронідази.

19. У жінки 63 років

виявлені ознаки ревма-

тоїдного артриту. Підви-

щення рівня якого

показника крові буде

найбільш вирішальним

для підтвердження

діагнозу ?

А. Кислої фосфатази.

В. Ліпопротеїдів.

С. R-глікозидази.

D. Загального холестерину.

Е. *Сумарних глікозаміно-

гліканів.

20. У відділення травмато-

логії привезли хворого із

пошкодженою м’язовою

тканиною. Який біохіміч-

ний показник сечі при

цьому буде збільшений ?

А. Сечова кислота.

В. Загальні ліпіди.

С. Мінеральні солі.

D. Глюкоза.

Е. *Креатинін.

21. Чоловік 40 років пробіг

10 км за 60 хв. Які зміни

енергетичного обміну

відбудуться у його м’язах ?

А. Посилиться глюко-

неогенез.

В. Посилиться гліколіз.

С. Посилиться протеоліз.

D. Посилиться глікогеноліз.

Е. *Збільшиться швидкість

окиснення жирних кислот.

22. У людей після

тривалого фізичного

навантаження виникає

інтенсивний біль у м'язах.

Які зміни у м'язах є

найбільш вірогідною

причиною цього ?

А. Накопичення креатиніну.

Page 126: КРОК-1 БІОХІМІЯ Збірник тестових завданьlib.sumdu.edu.ua/library/docs/rio/2020/m4695.pdf · 2 Крок-1. Біохімія Н: збірник тестових

126

В. Підвищена збудливість.

С. Підвищення вмісту АДФ.

D. Посилений розпад білків.

Е. *Накопичення молочної

кислоти.

23. У юнака 18 років

діагностовано м’язову дис-

трофію. Підвищення якої

речовини в сироватці

крові найбільш імовірне

при цій патології ?

А. Аланіну.

В. Міоглобіну.

С. Міозину.

D. Лактату.

Е. *Креатину.

Page 127: КРОК-1 БІОХІМІЯ Збірник тестових завданьlib.sumdu.edu.ua/library/docs/rio/2020/m4695.pdf · 2 Крок-1. Біохімія Н: збірник тестових

127

ЧАСТИНА ІІ

2.1. КРОК-2007

1. У хворого із запаленням

легень спостерігається

підвищення температури

тіла. Яка біологічно

активна речовина відіграє

провідну роль у

виникненні цього прояву ?

А. Серотонін.

В. Гістамін.

С. Лейкотрієни.

D. Брадикінін.

Е. * Інтерлейкін-1.

2. У групи альпіністів на

висоті 3000 метрів було

зроблено аналіз крові.

Виявлено зниження НСО3 -

до 15 ммоль/л (норма 22 -

26 ммоль/л). Який

механізм зниження НСО3-

крові ?

А. Посилення ацидогенезу.

В. Зниження реабсорбції

бікарбонатів у нирках.

С. Гіповентиляція.

D. Зниження амоніогенезу.

Е. *Гіпервентиляція.

3. У хворого виявлено

гіперкаліємію та гіпо-

натріємію. Знижена

секреція якого гормону

може спричинити такі

зміни ?

А. Паратгормон.

В. Кортизол.

С. Натрійуретичний.

D. Вазопресин.

Е. *Альдостерон.

4. У дитини 2 років

виникли судоми внаслідок

зниження концентрації

іонів кальцію в плазмі

крові. Функція якого

ендокринного органа

знижена ?

А. Кора наднирників.

В. Шишкоподібна залоза.

С. Гіпофіз.

D. Тимус.

Е. *Паращитовидні залози.

5. У культурі клітин,

отриманих від хворого з

лізосомною патологією,

виявлено накопичення

значної кількості ліпідів у

лізосомах. При якому з

перелічених захворювань

має місце це порушення ?

А. Галактоземія.

В. Фенілкетонурія.

С. Подагра.

Page 128: КРОК-1 БІОХІМІЯ Збірник тестових завданьlib.sumdu.edu.ua/library/docs/rio/2020/m4695.pdf · 2 Крок-1. Біохімія Н: збірник тестових

128

D. Хвороба Вільсона-

Коновалова.

Е. *Хвороба Тея-Сакса.

6. Хвора 48 років

доставлена в клініку зі

скаргами на слабість,

дратівливість, порушення

сну. Об'єктивно: шкіра та

склери жовтого кольору. У

крові: підвищення рівня

загального білірубіну з

переважанням прямого.

Кал – ахолічний. Сеча –

темного кольору. Яка

жовтяниця має місце у

хворої ?

А. Паренхіматозна.

В. Гемолітична.

С. Синдром Криглера-

Найяра.

D. Синдром Жильбера.

Е. *Механічна.

7. При утилізації

арахідонової кислоти за

циклооксигеназним

шляхом утворюються

біологічно активні

речовини. Назвіть їх:

А. Біогенні аміни.

В. Інсуліноподібні фактори

росту.

С. Соматомедини.

D. Тироксин.

Е. *Простагландини.

8. До лікаря звернулися

батьки хлопчика 10 років,

у якого відзначалося

збільшення волосяного

покриву на тілі, ріст

бороди й вусів, низький

голос. Збільшення секреції

якого гормону можна

припустити ?

А. Кортизол.

В. Естроген.

С. Прогестерон.

D. Соматотропін.

Е. *Тестостерон.

9. У хворого через 12 год.

після гострого нападу

загруднинного болю вста-

новлено різке підвищення

активності АсАТ у

сироватці крові. Назвіть

патологію, для якої

характерне це підвищення:

А. Колагеноз.

В. Нецукровий діабет.

С. Вірусний гепатит.

D. Цукровий діабет.

Е. *Інфаркт міокарда.

10. У людини порушені

процеси всмоктування

продуктів гідролізу жирів.

Причиною цього може

Page 129: КРОК-1 БІОХІМІЯ Збірник тестових завданьlib.sumdu.edu.ua/library/docs/rio/2020/m4695.pdf · 2 Крок-1. Біохімія Н: збірник тестових

129

бути дефіцит у порожнині

тонкої кишки таких

компонентів:

А. Жовчні пігменти.

В. Іони натрію.

С. Ліполітичні ферменти.

D. Жиророзчинні вітаміни.

Е. *Жовчні кислоти.

11. У районах Південної

Африки у людей поширена

серпоподібноклітинна ане-

мія, при якій еритроцити

мають форму серпа

внаслідок заміни в

молекулі гемоглобіну

амінокислоти глутаміну на

валін. Чим викликана ця

хвороба ?

А. Кросинговер.

В. Трансдукція.

С. Порушення механізмів

реалізації генетичної

інформації.

D. Геномні мутації.

Е. *Генна мутація.

12. У хлопчика 4 років

після перенесеного

тяжкого вірусного

гепатиту мають місце

блювання, втрата

свідомості, судоми. У крові

– гіперамоніємія. Пору-

шення якого біохімічного

процесу зумовило

патологічний стан

хворого ?

А. Порушення знешкоджен-

ня біогенних амінів.

В. Активація декарбокси-

лювання амінокислот.

С. Посилення гниття білків у

кишечнику.

D. Пригнічення ферментів

трансамінування.

Е. *Порушення знешкод-

ження аміаку в печінці.

13. Аміак є дуже отруйною

речовиною, особливо для

нервової системи. Яка

речовина бере особливо

активну участь у

знешкодженні аміаку в

тканинах мозку ?

А. Гістидин.

В. Лізин.

С. Пролін.

D. Аланін.

Е. *Глутамінова кислота.

14. До клініки доставлена

дитина 4 років з ознаками

тривалого білкового

голодування: затримка

росту, анемія, набряки,

розумова відсталість.

Причиною розвитку

Page 130: КРОК-1 БІОХІМІЯ Збірник тестових завданьlib.sumdu.edu.ua/library/docs/rio/2020/m4695.pdf · 2 Крок-1. Біохімія Н: збірник тестових

130

набряків у цієї дитини є

зниження синтезу:

А. Глобулінів.

В. Гемоглобіну.

С. Глікопротеїнів.

D. Ліпопротеїнів.

Е. *Альбумінів.

15. У новонародженої

дитини з пілоростенозом

часте блювання, що

супроводжується апатією,

слабістю, підвищенням

тонусу мязів, інколи

судомами. Яка форма

порушення кислотно-

основного стану розви-

нулася у дитини ?

А. Газовий ацидоз.

В. Видільний ацидоз.

С. Метаболічний ацидоз.

D. Газовий алкалоз.

Е. *Негазовий алкалоз.

16. Хворий 42 років

скаржиться на сильне

серцебиття, пітливість,

нудоту, порушення зору,

тремор рук, підвищення

артеріального тиску. З

анамнезу: 2 роки тому було

встановлено діагноз

феохромоцитома. Гіпер-

продукція яких гормонів

зумовлює цю патологію ?

А. АКТГ.

В. Тиреоїдні гормони.

С. Альдостерон.

D. Глюкокортикоїди.

Е. *Катехоламіни.

17. Хворий перебуває на

обліку в ендокринологіч-

ному диспансері з приводу

гіпертиреозу. До схуднен-

ня, тахікардії, тремтіння

пальців рук додалися

симптоми гіпоксії –

головний біль, втомлю-

ваність, мерехтіння

«мушок» перед очима.

Який механізм дії

тиреоїдних гормонів

лежить в основі розвитку

гіпоксії ?

А. Гальмування синтезу

дихальних ферментів.

В. Посилення синтезу

дихальних ферментів.

С. Конкурентне гальмування

дихальних ферментів.

D. Специфічне зв’язування

активних центрів дихальних

ферментів.

Е. *Розєднання окиснення

та фосфорилювання.

18. При декарбоксилюван-

ні глутамату в ЦНС

утворюється медіатор

Page 131: КРОК-1 БІОХІМІЯ Збірник тестових завданьlib.sumdu.edu.ua/library/docs/rio/2020/m4695.pdf · 2 Крок-1. Біохімія Н: збірник тестових

131

гальмування. Назвіть

його:

А. Серотонін.

В. Аспарагін.

С. Глутатіон.

D. Гістамін.

Е. *ГАМК.

19. У людини осмотичний

тиск плазми крові

становить 350 мосмоль/л

(норма 300 мосмоль/л). Це

приведе до посиленої

секреції такого гормону:

А. Альдостерон.

В. Натрійуретичний.

С. Адренокортикотропін.

D. Кортизон.

Е. *Вазопресин.

20. При дослідженні

сироватки крові хворого

виявлене підвищення

рівня аланінаміно-

трансферази (АЛТ) та

аспартатамінотрансферази

(АСТ). Які зміни на

клітинному рівні можуть

призвести до подібної

ситуації ?

А. Порушення ферментних

систем клітин.

В. Руйнування генетичного

апарату клітин.

С. Порушення міжклітинних

взаємодій.

D. Порушення функції

енергозабезпечення клітин.

Е. *Руйнування клітин.

21. Госпіталізовано

хворого з діагнозом

карциноїд кишечника.

Аналіз виявив підвищену

продукцію серотоніну.

Відомо, що ця речовина

утворюється з аміно-

кислоти триптофану. Який

біохімічний механізм

лежить в основі даного

процесу ?

А. Утворення парних

сполук.

В. Дезамінування.

С. Трансамінування.

D. Мікросомальне окисне-

ння.

Е. *Декарбоксилювання.

2.2. КРОК-2008

1. У пацієнта цироз

печінки. Дослідження якої

з перелічених речовин, що

екскретуються із сечею,

може характеризувати

стан антитоксичної

функції печінки ?

Page 132: КРОК-1 БІОХІМІЯ Збірник тестових завданьlib.sumdu.edu.ua/library/docs/rio/2020/m4695.pdf · 2 Крок-1. Біохімія Н: збірник тестових

132

А. Креатинін.

В. Амінокислоти.

С. Сечова кислота.

D. Амонійні солі.

Е. *Гіпурова кислота.

2. При обстеженні дитини

лікар виявив ознаки

рахіту. Нестача якої

сполуки в організмі

дитини сприяє розвитку

цього захворювання ?

А. Ретинол.

В. Нафтохінон.

С. Біотин.

D. Токоферол.

Е. *1,25-дигідроксихоле-

кальциферол.

3. Онкологічному хворому

призначили препарат

метотрексат, до якого з

часом клітини-мішені

пухлини втратили

чутливість. Експресія гену

якого ферменту при цьому

змінюється ?

А. Тимідиназа.

В. Фолатоксидаза.

С. Фолатдекарбоксилаза.

D. Дезаміназа.

Е. *Дигідрофолатредуктаза.

4. Під час голодування

мязові білки

розпадаються до вільних

амінокислот. В який

процес найімовірніше

будуть залучатися ці

сполуки ?

А. Глікогеноліз.

В. Синтез вищих жирних

кислот.

С. Глюконеогенез у мязах.

D. Декарбоксилювання.

Е. *Глюконеогенез у

печінці.

5. Фармакологічні ефекти

антидепресантів пов’язані

з блокуванням (інгібуван-

ням) ними ферменту, який

каталізує розпад таких

біогенних амінів, як

норадреналін і серотонін у

мітохондріях нейронів

головного мозку. Який

фермент бере участь у

цьому процесі ?

А. Трансаміназа.

В. Пептидаза.

С. Декарбоксилаза.

D. Ліаза.

Е. *Моноамінооксидаза.

6. Чоловік 70 років хворіє

на атеросклероз судин

нижніх кінцівок та

ішемічну хворобу серця.

Під час обстеження

Page 133: КРОК-1 БІОХІМІЯ Збірник тестових завданьlib.sumdu.edu.ua/library/docs/rio/2020/m4695.pdf · 2 Крок-1. Біохімія Н: збірник тестових

133

виявлено порушення

ліпідного складу крові.

Надлишок яких

ліпопротеїдів є головною

ланкою в патогенезі

атеросклерозу ?

А. Проміжної щільності.

В. Дуже низької щільності.

С. Високої щільності.

D. Хіломікрони.

Е. *Низької щільності.

7. Плазмові фактори

згортання крові зазнають

посттрансляційної моди-

фікації за участі вітаміну

К. Як кофактор він бере

участь у системі

-карбоксилювання білко-

вих факторів коагуляції

крові. Яка амінокислота

карбоксилюється в цих

білках ?

А. Аргінін.

В. Валін.

С. Фенілаланін.

D. Серин.

Е. *Глутамінова.

8. На прийом до лікаря

звернувся хворий із

симетричним дерматитом

відкритих ділянок шкіри.

З бесіди з пацієнтом

встановлено, що він

харчується в основному

крупами і вживає мало

мяса, молока та яєць.

Дефіцит якого вітаміну є

провідним у цього

пацієнта ?

А. Кальциферол.

В. Токоферол.

С. Фолієва кислота.

D. Біотин.

Е. *Нікотинамід.

9. У пацієнта, що

звернувся до лікаря,

спостерігається жовте

забарвлення шкіри, сеча

темна, кал темно-жовтого

кольору. Підвищення

концентрації якої

речовини буде спостеріга-

тися в сироватці крові ?

А. Вердоглобін.

В. Конюгований білірубін.

С. Мезобілірубін.

D. Білівердин.

Е. *Вільний білірубін.

10. Унаслідок пост-

трансляційних змін деяких

білків, що беруть участь у

зсіданні крові, зокрема

протромбіну, вони

набувають здатності

зв’язувати кальцій. У

Page 134: КРОК-1 БІОХІМІЯ Збірник тестових завданьlib.sumdu.edu.ua/library/docs/rio/2020/m4695.pdf · 2 Крок-1. Біохімія Н: збірник тестових

134

цьому процесі бере участь

вітамін:

А. В1.

В. А.

С. С.

D. В2.

Е. *К.

11. Чоловік 65 років, який

страждає на подагру,

скаржиться на біль у

ділянці нирок. При

ультразвуковому обсте-

женні встановлена

наявність ниркових

каменів. Підвищення

концентрації якої

речовини є найбільш

вірогідною причиною

утворення каменів у

даному випадку ?

А. Сечовина.

В. Білірубін.

С. Цистин.

D. Холестерин.

Е. *Сечова кислота.

12. У трирічної дитини з

підвищеною температурою

тіла після прийому

аспірину спостерігається

посилений гемоліз

еритроцитів. Спадковий

дефіцит якого ферменту

спричинив у дитини

гемолітичну анемію ?

А. Глікогенфосфорилаза.

В. Гліцерофосфатдегідро-

геназа.

С. Глюкозо-6-фосфатаза.

D. -Глутамілтрансфераза.

Е. *Глюкозо-6-фосфат-

дегідрогеназа.

13. Було доведено, що

молекула незрілої і-РНК

(про-іРНК) містить більше

триплетів, ніж знайдено

амінокислот у синтезо-

ваному білку. Це

пояснюється тим, що

трансляції у нормі передує:

А. Ініціація.

В. Мутація.

С. Репарація.

D. Реплікація.

Е. *Процесинг.

14. У хворого, який

знаходиться на лікуванні з

приводу вірусного

гепатиту В, зявилися

ознаки печінкової

недостатності. Які зміни

крові, що свідчать про

порушення білкового

обміну, спостерігатиму-

ться у цьому випадку ?

Page 135: КРОК-1 БІОХІМІЯ Збірник тестових завданьlib.sumdu.edu.ua/library/docs/rio/2020/m4695.pdf · 2 Крок-1. Біохімія Н: збірник тестових

135

А. Білковий склад крові не

змінений.

В. Абсолютна

гіперглобулінемія.

С. Абсолютна

гіперальбумінемія.

D. Абсолютна

гіперфібриногенемія.

Е. *Абсолютна

гіпоальбумінемія.

15. У лікарню звернувся

хворий зі скаргами на

швидку втомлюваність і

виражену мязову слабість.

При обстеженні виявлено

аутоімунне захворювання,

внаслідок якого пору-

шується функціональний

стан рецепторів у нервово-

мязових синапсах. Дія

якого медіатора буде

заблокована ?

А. Дофамін.

В. Гліцин.

С. Серотонін.

D. Норадреналін.

Е. *Ацетилхолін.

16. Після видалення

частини шлунка у хворого

порушилося всмоктування

вітаміну В12, він

виводиться з калом.

Розвинулася анемія. Який

фактор необхідний для

всмоктування цього

вітаміну ?

А. Пепсин.

В. Гастрин.

С. Фолієва кислота.

D. Соляна кислота.

Е. *Гастромукопротеїн.

17. Встановлено, що певні

сполуки, наприклад,

токсини грибів та деякі

антибіотики, можуть

пригнічувати активність

РНК-полімерази. Пору-

шення якого процесу

відбувається в клітині у

випадку інгібування

даного ферменту ?

А. Репарація.

В. Трансляція.

С. Реплікація.

D. Процесинг.

Е. *Транскрипція.

18. У хворої 38 років

ревматизм в активній фазі.

Визначення якого

лабораторного показника

сироватки крові має

діагностичне значення при

даній патології ?

А. Креатинін.

В. Трансферин.

С. Сечовина.

Page 136: КРОК-1 БІОХІМІЯ Збірник тестових завданьlib.sumdu.edu.ua/library/docs/rio/2020/m4695.pdf · 2 Крок-1. Біохімія Н: збірник тестових

136

D. Сечова кислота.

Е. *С-реактивний білок.

2.3. КРОК-2009

1. У хворого на хронічний

гепатит виявлено значне

зниження синтезу і

секреції жовчних кислот.

Який процес найбільшою

мірою буде порушений у

кишечнику цього

хворого ?

А. Всмоктування гліцерину.

В. Травлення білків.

С. Травлення вуглеводів.

D. Всмоктування

амінокислот.

Е. *Емульгування жирів.

2. Під час огляду дитини 11

місяців педіатр виявив

викривлення кісток

нижніх кінцівок і затримку

мінералізації кісток

черепа. Нестача якого

вітаміну призводить до

даної патології ?

А. Біофлавоноїди.

В. Пантотенова кислота.

С. Тіамін.

D. Рибофлавін.

Е. *Холекальциферол.

3. У хворих із

непрохідністю жовчовід-

відних шляхів пригнічує-

ться згортання крові,

виникають кровотечі, що є

наслідком недостатнього

засвоєння вітаміну :

А. Е.

В. D.

С. А.

D. С.

Е. *К.

4. У дівчинки діагносто-

ваний псевдогермафроди-

тизм. Надмірна секреція

яких гормонів наднир-

ників зумовила дану

патологію ?

А. Мінералокортикоїди.

В. Катехоламіни.

С. Естрогени.

D. Глюкокортикоїди.

Е. *Андрогени.

5. Хворий 49 років, водій

за професією, скаржиться

на нестерпний стискаючий

біль за грудниною, що

«віддає» у ділянці шиї.

Біль виник 2 год тому.

Обєктивно: стан тяжкий,

тони серця послаблені.

Лабораторне обстеження

показало високу

Page 137: КРОК-1 БІОХІМІЯ Збірник тестових завданьlib.sumdu.edu.ua/library/docs/rio/2020/m4695.pdf · 2 Крок-1. Біохімія Н: збірник тестових

137

активність креатинкінази

та ЛДГ1. Для якого

захворювання характерні

ці симптоми ?

А. Жовчнокамяна хвороба.

В. Цукровий діабет.

С. Стенокардія.

D. Гострий панкреатит.

Е. *Гострий інфаркт

міокарда.

6. Вивчається робота

оперону бактерії.

Відбулося звільнення гена-

оператора від білка

репресора. Безпосередньо

після цього в клітині

почнеться:

А. Реплікація.

В. Репресія.

С. Трансляція.

D. Процесинг.

Е. *Транскрипція.

7. Немовля відмовляється

від годування груддю,

збудливе, дихання

неритмічне, сеча має

специфічний запах

«кленового сиропу».

Уроджений дефект якого

ферменту викликав дану

патологію ?

А. УДФ-глюкуроніл-

трансфераза.

В. Гліцеролкіназа.

С. Аспартатамінотранс-

фераза.

D. Глюкозо-6-фосфат-

дегідрогеназа.

Е. *Дегідрогеназа розгалу-

жених -кетокислот.

8. У немовляти спосте-

рігається диспепсія після

годування молоком. При

заміні молока розчином

глюкози симптоми

диспепсії зникають.

Недостатня активність

якого ферменту

спостерігається у

новонародженого ?

А. Мальтаза.

В. Сахараза.

С. Амілаза.

D. Ізомальтаза.

Е. *Лактаза.

9. У хворого зявилося

пожовтіння шкіри, склер

та слизових оболонок. У

плазмі крові підвищений

рівень загального

білірубіну, в калі – рівень

стеркобіліну, в сечі –

уробіліну. Який вид

жовтяниці у хворого ?

А. Паренхіматозна.

В. Синдром Жильбера.

Page 138: КРОК-1 БІОХІМІЯ Збірник тестових завданьlib.sumdu.edu.ua/library/docs/rio/2020/m4695.pdf · 2 Крок-1. Біохімія Н: збірник тестових

138

С. Обтураційна.

D. Холестатична.

Е. *Гемолітична.

10. У хворого після

вживання недоброякісної

їжі розвинувся

багаторазовий пронос. На

наступний день у нього

знизився артеріальний

тиск, зявилися тахікардія,

екстрасистолія. рН крові

становить 7,18. Ці

порушення є наслідком

розвитку:

А. Газового алкалозу.

В. Газового ацидозу.

С. Метаболічного алкалозу.

D. Негазового алкалозу.

Е. *Негазового ацидозу.

11. Дитина 9 місяців

харчується штучними

сумішами, які не

збалансовані за вмістом

вітаміну В6. У дитини

спостерігається пелагро-

подібний дерматит,

судоми, анемія. Розвиток

судом може бути

пов'язаний із порушенням

утворення:

А. ДОФА.

В. Серотоніну.

С. Гістаміну.

D. Дофаміну.

Е. *ГАМК.

12. Хворому внутрішньо-

венно ввели гіпертонічний

розчин глюкози. Це

підсилить рух води:

А. Із міжклітинної рідини до

капілярів.

В. Із капілярів до

міжклітинної рідини.

С. Змін руху води не буде.

D. Із міжклітинної рідини до

клітин.

Е. *Із клітин до

міжклітинної рідини.

13. Для прискорення

загоєння рани слизової

оболонки в ротовій

порожнині хворому

призначено препарат,

який являє собою

термостабільний білок, що

міститься в сльозах, слині,

грудному молоці матері, а

також його можна виявити

у свіжому курячому яйці.

Відомо, що він являє собою

фактор природної

резистентності організму і

має назву:

А. Іманін.

В. Інтерферон.

С. Комплемент.

Page 139: КРОК-1 БІОХІМІЯ Збірник тестових завданьlib.sumdu.edu.ua/library/docs/rio/2020/m4695.pdf · 2 Крок-1. Біохімія Н: збірник тестових

139

D. Інтерлейкін.

Е. *Лізоцим.

14. У хворого виявлено

зниження вмісту іонів

магнію, які потрібні для

прикріплення рибосом до

гранулярної ендоплазма-

тичної сітки. Відомо, що це

призводить до порушення

біосинтезу білка. Який

процес буде порушений ?

А. Реплікація.

В. Транскрипція.

С. Термінація.

D. Активація амінокислот.

Е. *Трансляція.

15. У юнака 16 років після

перенесеного захворюван-

ня знижена функція

синтезу білків у печінці

внаслідок нестачі вітаміну

К. Це може призвести до

порушення:

А. Осмотичного тиску крові.

В. Утворення

антикоагулянтів.

С. Утворення

еритропоетинів.

D. Швидкості осадження

еритроцитів .

Е. *Згортання крові.

16. До лікарні потрапила

дитина 6 років. Під час

обстеження було виявлено,

що дитина не може

фіксувати погляд, не

слідкує за іграшками, на

очному дні відзначається

симптом «вишневої

кістки». Лабораторні

аналізи показали, що у

мозку, печінці та селезінці

– підвищений рівень

гангліозиду глікометиду.

Яке спадкове захворю-

вання у дитини ?

А. Хвороба Німана-Піка.

В. Синдром

Шерешевського-Тернера.

С. Хвороба Мак-Ардля.

D. Хвороба Вільсона-

Коновалова.

Е. *Хвороба Тея-Сакса.

17. Дитина 3 років із

симптомами стоматиту,

гінгівіту, дерматиту

відкритих ділянок шкіри

була госпіталізована. При

обстеженні встановлено

спадкове порушення

транспорту нейтральних

амінокислот у кишечнику.

Нестача якого вітаміну

зумовила дані симптоми ?

А. Вітамін А.

Page 140: КРОК-1 БІОХІМІЯ Збірник тестових завданьlib.sumdu.edu.ua/library/docs/rio/2020/m4695.pdf · 2 Крок-1. Біохімія Н: збірник тестових

140

В. Кобаламін.

С. Пантотенова кислота.

D. Біотин.

Е. *Ніацин.

18. У жінки 30 років

виникли набряки обличчя.

При обстеженні виявлені

протеїнурія (5,87 г/л),

гіпопротеїнемія, диспротеї-

немія, гіперліпідемія. Для

якого стану характерне

таке поєднання

симптомів ?

А. Гломерулонефрит.

В. Гостра ниркова

недостатність.

С. Хронічний пієлонефрит.

D. Хронічна ниркова

недостатність.

Е. *Нефротичний синдром.

19. У хворого знижений

синтез вазопресину, що

призводить до поліурії і, як

наслідок, до вираженої

дегідратації організму. У

чому полягає механізм

розвитку поліурії ?

А. Збільшення швидкості

клубочкової фільтрації.

В. Зниження канальцевої

реабсорбції.

С. Зниження реабсорбції

глюкози.

D. Зниження канальцевої

реабсорбції іонів натрію.

Е. *Зниження канальцевої

реабсорбції води.

20. До лікаря звернулася

жінка 32 років зі скаргами

на відсутність лактації

після народження дитини.

Дефіцитом якого гормону

можна пояснити дане

порушення ?

А. Соматотропін.

В. Тиреокальцитонін.

С. Вазопресин.

D. Глюкагон.

Е. *Пролактин.

21. У пацієнта діагносту-

вали гостру променеву

хворобу. Лабораторно

встановлено різке зниже-

ння вмісту серотоніну в

тромбоцитах. Порушення

метаболізму якої речовини

є можливою причиною

зниження тромбоцитар-

ного серотоніну:

А. Гістидин.

В. Фенілаланін.

С. Тирозин.

D. Серин.

Е. *5-Окситриптофан.

Page 141: КРОК-1 БІОХІМІЯ Збірник тестових завданьlib.sumdu.edu.ua/library/docs/rio/2020/m4695.pdf · 2 Крок-1. Біохімія Н: збірник тестових

141

22. У дитини спостерігає-

ться затримка фізичного

та розумового розвитку,

глибокі порушення з боку

сполучної тканини

внутрішніх органів, у сечі

виявлено кератан-

сульфати. Обмін яких

речовин порушений ?

А. Гіалуронова кислота.

В. Еластин.

С. Фібронектин.

D. Колаген.

Е. *Глікозаміноглікани.

23. До лікарні була

доставлена дитина 2 років

із уповільненим розумовим

і фізичним розвитком, що

страждає на часті

блювання після прийому

їжі. У сечі визначена

фенілпіровиноградна

кислота. Обмін яких

речовин порушений у

дитини ?

А. Обмін ліпідів.

В. Фосфорно-кальцієвий

обмін.

С. Вуглеводний обмін.

D. Водно-сольовий обмін.

Е. *Обмін амінокислот.

24. У 12-річного хлопчика

в сечі виявлений високий

вміст усіх амінокислот

аліфатичного ряду. При

цьому відмічена найбільш

висока екскреція цистину

та цистеїну. Крім того,

УЗД нирок показало

наявність каменів у них.

Яка патологія найбільш

імовірна ?

А. Цистит.

В. Фенілкетонурія.

С. Алкаптонурія.

D. Хвороба Хартнупа.

Е. *Цистинурія.

2.4. КРОК-2010

1. Хворий 48 років

звернувся до лікаря зі

скаргами на сильний біль,

припухлість, почервоніння

шкіри над суглобами,

підвищення температури

до 38oC. У крові виявлено

високий вміст уратів.

Вірогідною причиною

такого стану може бути

порушення обміну:

A. Піримідинів.

B. Вуглеводів.

C. Колагену.

D. Холестерину.

E. *Пуринів.

Page 142: КРОК-1 БІОХІМІЯ Збірник тестових завданьlib.sumdu.edu.ua/library/docs/rio/2020/m4695.pdf · 2 Крок-1. Біохімія Н: збірник тестових

142

2. Хлопчик 5 місяців

госпіталізований з приводу

судом. Хворіє з

народження. Об'єктивно:

волосся жорстке, нігті

тонкі та ламкі, шкірні

покриви бліді та сухі. В

біохімічному аналізі крові:

кальцій - 0,5 ммоль/л

(норма - 0,75-2,5 ммоль/л),

фосфор - 1,9 ммоль/л

(норма - 0,646-1,292

ммоль/л). Із чим пов'язані

ці зміни ?

A. Гіперпаратиреоз.

B. Гіпотиреоз.

C. Гіперальдостеронізм.

D. Гіпоальдостеронізм.

E. *Гіпопаратиреоз.

3. Для лікування

жовтяниць призначають

барбітурати, що індукують

синтез УДФ-глюкуроніл-

трансферази. Лікувальний

ефект при цьому

обумовлений утворенням:

A. Непрямого (некон'юго-

ваного) білірубіну.

B. Протопорфірину.

C. Гему.

D. Білівердину.

E. *Прямого (кон'юго-

ваного) білірубіну.

4. Під час емоційного

стресу в адипоцитах

активується гормон-

чутлива тригліцерид-

ліпаза. Який вторинний

посередник бере участь у

цьому процесі ?

A. Іони Ca2+.

B. Диацилгліцерол.

C. АМФ.

D. цГМФ.

E. *цАМФ.

5. У хворого на жовтяницю

у крові підвищений вміст

прямого білірубіну та

жовчних кислот; у сечі

відсутній стеркобіліноген.

При якій жовтяниці

можлива наявність цих

ознак ?

A. Паренхіматозна.

B. Печінкова.

C. Гемолітична.

D. Надпечінкова.

E. *Механічна.

6. У хворого 42 років

виявили пухлину

аденогіпофіза. Об'єктивно:

вага 117 кг, обличчя

місяцеподібне, на шкірі

живота синьо-багряні

смуги розтягу. Остеопороз,

дистрофія м'язів. АТ-

Page 143: КРОК-1 БІОХІМІЯ Збірник тестових завданьlib.sumdu.edu.ua/library/docs/rio/2020/m4695.pdf · 2 Крок-1. Біохімія Н: збірник тестових

143

210/140 мм рт.ст. У хворого

найбільш вірогідно:

A. Цукровий діабет.

B. Хвороба Кона.

C. Гіпертонічна хвороба.

D. Синдром Іценка-

Кушинга.

E. *Хвороба Іценка-

Кушинга.

7. Пацієнта турбують

поліурія (7 л на добу) і

полідипсія. При

обстеженні не виявлено

ніяких розладів

вуглеводного обміну.

Дисфункція якої

ендокринної залози може

бути причиною даних

порушень ?

A. Острівці підшлункової

залози.

B. Мозкова речовина

наднирників.

C. Аденогіпофіз.

D. Кора наднирників.

E. *Нейрогіпофіз.

8. До стоматолога звернув-

ся чоловік 35 років зі

скаргами на зменшення

щільності зубної тканини,

підвищену крихкість її при

прийомі твердої їжі.

Нестача якого мінераль-

ного елемента має місце у

даного пацієнта ?

A. Залізо.

B. Натрій.

C. Калій.

D. Магній.

E. *Кальцій.

9. Хворий 46 років

звернувся до лікаря зі

скаргою на біль у суглобах,

який посилюється

напередодні зміни погоди.

У крові виявлено

підвищення концентрації

сечової кислоти.

Посилений розпад якої

речовини є причиною

захворювання ?

A. ЦМФ.

B. ТМФ.

C. УМФ.

D. УТФ.

E. *АМФ.

10. Під час тестування на

гіперчутливість пацієнту

під шкіру ввели алерген,

після чого спостерігалися

почервоніння, набряк, біль

унаслідок дії гістаміну. В

результаті якого перетво-

рення амінокислоти

гістидину утворюється цей

біогенний амін ?

Page 144: КРОК-1 БІОХІМІЯ Збірник тестових завданьlib.sumdu.edu.ua/library/docs/rio/2020/m4695.pdf · 2 Крок-1. Біохімія Н: збірник тестових

144

A. Дезамінування.

B. Метилювання.

C. Ізомеризації.

D. Фосфорилювання.

E. *Декарбоксилювання.

11. У новонародженої

дитини спостерігалися

судоми, що проходили

після введення вітаміну B6.

Цей ефект, найбільш

імовірно, викликаний тим,

що вітамін B6 входить до

складу ферменту:

A. Глікогенфосфорилази.

B. Глікогенсинтази.

C. Піруватдегідрогеназиа.

D. Аргінази.

E.*Глутаматдекарбоксилази.

12. Провідними симпто-

мами первинного

гіперпаратиреозу є

остеопороз та ураження

нирок із розвитком

сечокам'яної хвороби. Яка

речовина складає основу

каменів при цьому

захворюванні ?

A. Цистин.

B. Холестерин.

C. Сечова кислота.

D. Білірубін.

E. *Фосфат кальцію.

13. Із сироватки крові

людини виділили п'ять

ізоферментних форм

лактатдегідрогенази і

вивчили їх властивості.

Яка властивість доводить,

що виділені ізоферментні

форми одного й того

самого ферменту ?

A. Однакові фізико-хімічні

властивості.

B. Однакова молекулярна

маса.

C. Тканинна локалізація.

D. Однакова

електрофоретична

рухливість.

E. *Каталізують однакову

реакцію.

14. Чоловіку 46 років, що

хворіє на дифузний

токсичний зоб, була

проведена операція

резекції щитоподібної

залози. Після операції

відмічаються відсутність

апетиту, диспепсія,

підвищена нервово-

м'язова збудливість. Маса

тіла не збільшилася.

Температура тіла у нормі.

Чим із нижче переліченого

обумовлений стан

хворого ?

Page 145: КРОК-1 БІОХІМІЯ Збірник тестових завданьlib.sumdu.edu.ua/library/docs/rio/2020/m4695.pdf · 2 Крок-1. Біохімія Н: збірник тестових

145

A. Підвищенням продукції

кальцитоніну.

B. Підвищенням продукції

тиреоліберину.

C. Підвищенням продукції

тироксину.

D. Зниженням продукції

тироксину.

E. *Зниженням продукції

паратгормону.

15. У новонародженої

дитини спостерігаються

зниження інтенсивності

смоктання, часте

блювання, гіпотонія. У

сечі та крові значно

підвищена концентрація

цитруліну. Який

метаболічний процес

порушений ?

A. Цикл Корі.

B. Глюконеогенез.

C. ЦТК.

D. Гліколіз.

E. *Орнітиновий цикл.

16. У підлітка 12 років,

який упродовж 3 місяців

різко схуд, вміст глюкози у

крові 50 ммоль/л. У нього

розвинулася кома. Який

головний механізм її

розвитку ?

A. Лактацидемічний.

B. Гіпоксичний.

C. Гіпоглікемічний.

D. Кетонемічний.

E. *Гіперосмолярний.

17. У немовляти спостері-

гається жовте забарвлення

шкіри та слизових

оболонок. Вірогідною

причиною такого стану

може бути тимчасова

нестача ферменту:

A. Уридинтрансферази.

B. Білівердинредуктази.

C. Гемоксигенази.

D. Гемсинтазиа.

E. *УДФ-глюкуроніл-

трансферази.

18. У крові 12-річного

хлопчика виявлено

зниження концентрації

сечової кислоти і

накопичення ксантину та

гіпоксантину. Генетичний

дефект якого ферменту

має місце у дитини ?

A. Аргіназа.

B. Орнітинкарбамоїл-

трансфераза.

C. Уреаза.

D. Гліцеролкіназа.

E. *Ксантиноксидаза.

19. У людей, адаптованих

Page 146: КРОК-1 БІОХІМІЯ Збірник тестових завданьlib.sumdu.edu.ua/library/docs/rio/2020/m4695.pdf · 2 Крок-1. Біохімія Н: збірник тестових

146

до дії високої зовнішньої

температури, посилене

потовиділення не

супроводжується втратою

з потом великої кількості

хлориду натрію. Дія якого

гормону на потові залози

спричиняє цей результат ?

A. Тироксин.

B. Натрійуретичний.

C. Вазопресин.

D. Кортизол.

E. *Альдостерон.

20. Встановлено ураження

вірусом ВІЛ Т-лімфоцитів.

При цьому фермент вірусу

- зворотна траскриптаза

(РНК-залежна ДНК-

полімераза) - каталізує

синтез:

A. і-РНК на матриці

вірусного білка.

B. ДНК на вірусній р-РНК.

C. Вірусної і-РНК на

матриці ДНК.

D. Вірусної ДНК на матриці

ДНК.

E. *ДНК на матриці вірусної

і-РНК.

21. У жінки 64 років

спостерігаються порушен-

ня сутінкового зору

(гемералопія). Препарат

якого вітаміну їй слід

рекомендувати в першу

чергу:

A. B2.

B. E.

C. C.

D. B6.

E. *A.

22. До клініки потрапила

дитина 1 року з ознаками

ураження м'язів кінцівок

та тулуба. Після

обстеження виявлений

дефіцит карнітину у

м'язах. Біохімічною

основою цієї патології є

порушення процесу:

A. Окисного фосфорилю-

вання.

B. Субстратного

фосфорилювання.

C. Регуляції рівня Ca2+ в

мітохондріях.

D. Утилізації молочної

кислоти.

E. *Транспорту жирних

кислот у мітохондрії.

23. При запальних

процесах в організмі

починається синтез білків

"гострої фази". Що є

стимулятором їх синтезу ?

A. Інтерферони.

Page 147: КРОК-1 БІОХІМІЯ Збірник тестових завданьlib.sumdu.edu.ua/library/docs/rio/2020/m4695.pdf · 2 Крок-1. Біохімія Н: збірник тестових

147

B. Біогенні аміни.

C. Ангіотензини.

D. Імуноглобуліни.

E. *Інтерлейкін-1.

24. Після прийому великої

кількості білкової їжі у

пацієнта встановлено

збільшення вмісту

протеолітичних ферментів

у соку підшлункової

залози. Вміст якого з

перелічених ферментів

збільшується при цьому ?

A. Ренін.

B. Гастриксин.

C. Ентерокіназа.

D. Пепсин.

E. *Трипсин.

25. Секреція яких гормонів

гіпофіза гальмується після

прийому оральних

контрацептивів, що

містять статеві гормони ?

A. Тиреотропні.

B. Соматотропний.

C. Окситоцин.

D. Вазопресин.

E. *Гонадотропні.

26. У клініку потрапив

чоловік 49 років із

суттєвим збільшенням

часу згортання крові,

шлунково-кишковими

кровотечами, підшкірними

крововиливами. Нестачею

якого вітаміну можна

пояснити такі симптоми ?

A. H.

B. E.

C. PP.

D. B1.

E. *K.

27. У хворого на ентерит,

що супроводжувався

значною діареєю,

спостерігається зменше-

ння кількості води в

позаклітинному просторі,

збільшення її всередині

клітин та зниження

осмолярності крові. Як

називають таке пору-

шення водно-електроліт-

ного обміну ?

A. Гіпоосмолярна гіпер-

гідратація.

B. Гіперосмолярна гіпо-

гідратація.

C. Гіперосмолярна гіпер-

гідратація.

D. Осмолярна

гіпогідратація.

E. *Гіпоосмолярна гіпо-

гідратація.

28. У 4-річної дитини зі

Page 148: КРОК-1 БІОХІМІЯ Збірник тестових завданьlib.sumdu.edu.ua/library/docs/rio/2020/m4695.pdf · 2 Крок-1. Біохімія Н: збірник тестових

148

спадковим ураженням

нирок спостерігаються

ознаки рахіту,

концентрація вітаміну D в

крові знаходиться в межах

норми. Що із переліченого

є найвірогіднішою причи-

ною розвитку рахіту ?

A. Гіпофункція паращито-

подібних залоз.

B. Порушення синтезу

кальцитріолу.

C. Недостатнє надходження

кальцію з їжею.

D. Гіперфункція паращито-

подібних залоз.

E. *Підвищена екскреція

кальцію із організму.

29. Під час обстеження

хворого виявлена

характерна клініка

колагенозу. Збільшення

вмісту в сечі якого

компонента характерне

для цієї патології ?

A. Аргінін.

B. Глюкоза.

C. Солі амонію.

D. Мінеральні солі.

E. *Гідроксипролін.

30. Після тривалого

фізичного навантаження

під час заняття з фізичної

культури у студентів

розвинулася м'язова

крепатура. Причиною її

виникнення стало

накопичення у скелетних

м'язах молочної кислоти.

Вона утворилася після

активації в організмi

студентів:

A. Пентозофосфатного

циклу.

B. Ліполізу.

C. Глюконеогенезу.

D. Глікогенезу.

E. *Гліколізу.

31. Після відновлення

кровообігу в ушкодженій

тканині припиняється

накопичення лактату та

зменшується швидкість

споживання глюкози.

Активацією якого процесу

зумовлені ці метаболічні

зміни ?

A. Біосинтез глікогену.

B. Глюконеогенез.

C. Анаеробний гліколіз.

D. Ліполіз.

E. *Аеробний гліколіз.

32. При дослідженні

перетворення харчового

барвника було встановле-

но, що знешкодження

Page 149: КРОК-1 БІОХІМІЯ Збірник тестових завданьlib.sumdu.edu.ua/library/docs/rio/2020/m4695.pdf · 2 Крок-1. Біохімія Н: збірник тестових

149

цього ксенобіотика

відбувається тільки в одну

фазу - мікросомального

окиснення. Назвіть

компонент цієї фази:

A. Цитохромоксидаза.

B. Цитохром с.

C. Цитохром в.

D. Цитохром а.

E. *Цитохром P-450.

2.5. КРОК-2011

1.Тварині внутрішньо-

венно ввели концентро-

ваний розчин хлориду

натрію, що зумовило

зниження реабсорбції іонів

натрію у канальцях нирок.

Унаслідок яких змін

секреції гормонів це

відбувається ?

А. Зменшення натрій-

уретичного фактора.

В. Збільшення альдостерону.

С. Збільшення вазопресину.

D. Зменшення вазопресину.

Е.*Зменшення

альдостерону.

2. Інозитолтрифосфати в

тканинах організму

утворюються в результаті

гідролізу фосфатидил-

інозитолдифосфатів і

відіграють роль вторинних

посередників (месендже-

рів) у механізмі дії

гормонів. Їхній вплив у

клітині спрямований на:

А. Гальмування фосфо-

диестерази.

В. Активацію аденілат-

циклази.

С. Гальмування протеїн-

кінази С.

D. Активацію протеїн-

кінази А.

Е. *Вивільнення іонів

кальцію з клітинних депо.

3. Після перенесеного

сепсису у хворої 27 років

зявився бронзовий колір

шкіри, характерний для

Аддісонової хвороби.

Механізм гіперпігментації

полягає в підвищенні

секреції такого гормону:

А. Гонадотропний.

В. Соматотропний.

С. Тиреотропний.

D. В-ліпотропний.

Е. *Меланоцито-

стимулюючий.

4. У клітині в гранулярній

ЕПС відбувається етап

трансляції. Амінокислоти

Page 150: КРОК-1 БІОХІМІЯ Збірник тестових завданьlib.sumdu.edu.ua/library/docs/rio/2020/m4695.pdf · 2 Крок-1. Біохімія Н: збірник тестових

150

зєднуються пептидними

звязками в певній

послідовності –

відбувається біосинтез

поліпептиду. Послідов-

ність амінокислот у

поліпептиді буде відповіда-

ти послідовності:

А. Антикодонів р-РНК.

В. Антикодонів т-РНК.

С. Нуклеотидів р-РНК.

D. Нуклеотидів т-РНК.

Е. *Кодонів і-РНК.

5. Хворому, що страждає

на хронічний гепатит, для

оцінки знешкоджувальної

функції печінки було

проведене навантаження

бензоатом натрію. За

виділенням якої речовини

з сечею оцінюють

знешкоджувальну

функцію печінки ?

А. Щавелева кислота.

В. Валеріанова кислота.

С. Фенілоцтова кислота.

D. Лимонна кислота.

Е. *Гіпурова кислота.

6. У хворого в крові

збільшена концентрація

пірувату. Значна його

кількість екскретується з

сечею. Дефіцит якого

вітаміну має місце у

хворого ?

А. В3.

В. В2.

С. Е.

D. В6.

Е. *В1.

7. До лікаря звернулася

мати з приводу поганого

самопочуття дитини –

відсутність апетиту,

поганий сон, дратівли-

вість. При біохімічному

дослідженні в крові

виявлено відсутність

ферменту глюкоцеребро-

зидази. Для якої патології

це характерно ?

А. Хвороба Гірке.

В. Хвороба Німана-Піка.

С. Хвороба Помпе.

D. Хвороба Тея-Сакса.

Е. *Хвороба Гоше.

8. Робітниця хімічного

підприємства внаслідок

порушення правил

безпечної роботи зазнала

токсичної дії азотистої

кислоти та нітритів, які

викликають дезамінуван-

ня цитозину в молекулі

ДНК. Який фермент

Page 151: КРОК-1 БІОХІМІЯ Збірник тестових завданьlib.sumdu.edu.ua/library/docs/rio/2020/m4695.pdf · 2 Крок-1. Біохімія Н: збірник тестових

151

ініціює ланцюг репарацій-

них процесів ?

А. ДНК-залежна РНК-

полімераза.

В. Оротидилмонофосфат-

декарбоксилаза.

С. Тимідилатсинтаза.

D. Цитидинтрифосфат-

синтетаза.

Е. *Урацил-ДНК-

глікозидаза.

9. Недостатність в

організмі лінолевої та

ліноленової кислот

призводить до ушкоджень

шкіри, випадіння волосся,

сповільненого загоювання

ран, тромбоцитопенії,

зниження опірності до

інфекційних захворювань.

Порушення синтезу яких

речовин найімовірніше

зумовлює названі

симптоми ?

А. Інтерлейкіни.

В. Інтерферони.

С. Кортикостероїди.

D. Катехоламіни.

Е. *Ейкозаноїди.

10. У хворого спостерігаю-

ться геморагії. В крові

знижена концентрація

протромбіну. Недостат-

ність якого вітаміну

призвела до порушення

синтезу цього фактора

згортання крові ?

А. Е.

В. С.

С. D.

D. А.

Е. *К.

11. Людина упродовж

тривалого часу вживала

їжу, збіднену на метіонін,

унаслідок чого у неї

спостерігалися розлади

функції нервової та

ендокринної систем. Це

може бути наслідком

порушення синтезу:

А. Тироніну.

В. Глюкагону.

С. Жирних кислот.

D. Пірувату.

Е. *Адреналіну.

12. У хворого 40 років

ознаки гірської хвороби:

запаморочення, задишка,

тахікардія, рН крові – 7,5,

рСО2 – 30 мм рт.ст., зсув

буферних основ

+4 ммоль/л. Яке пору-

шення кислотно-основного

стану має місце ?

А. Негазовий алкалоз.

Page 152: КРОК-1 БІОХІМІЯ Збірник тестових завданьlib.sumdu.edu.ua/library/docs/rio/2020/m4695.pdf · 2 Крок-1. Біохімія Н: збірник тестових

152

В. Газовий ацидоз.

С. Негазовий ацидоз.

D. Видільний ацидоз.

Е. *Газовий алкалоз.

13. У хворих на

тиреотоксикоз спосте-

рігаються гіпертермія,

зменшення маси тіла, що

повязане з порушенням:

А. Синтезу жирів.

В. Розпаду АТФ.

С. -окиснення жирних

кислот.

D. Циклу лимонної кислоти.

Е. *Спряження окиснення і

фосфорилювання.

14. Унаслідок значного

зниження концентрації

кальцію в плазмі крові у

дитини 2 років виникли

тетанічні скорочення

дихальних і глоткових

мязів. Зниження секреції

якого гормону може бути

причиною цього ?

А. Соматотропін.

В. Кортизол.

С. Тиреокальцитонін.

D. Альдостерон.

Е. *Паратгормон.

15. До реанімаційного

відділення в тяжкому

стані, без свідомості

доставлений пацієнт.

Діагностовано передозу-

вання барбітуратів, що

призвели до феномену

тканинної гіпоксії. На

якому рівні відбулося

блокування електронного

транспорту ?

А. Цитохром b-цитохром с1.

В. Убіхінон.

С. Цитохромоксидаза.

D. АТФ-синтаза.

Е. *НАДН-коензим-Q-

редуктаза.

16. При лабораторному

дослідженні у хворого

виявили стеаторею.

Назвіть фермент,

недостатність дії якого

призвела до виникнення

цього симптому:

А. Хімотрипсин.

В. Пепсин.

С. Амілаза.

D. Лактаза.

Е. *Ліпаза.

17. У хворого, який

проходить курс

лікувального голодування,

нормальний рівень

глюкози у крові

підтримується за рахунок

Page 153: КРОК-1 БІОХІМІЯ Збірник тестових завданьlib.sumdu.edu.ua/library/docs/rio/2020/m4695.pdf · 2 Крок-1. Біохімія Н: збірник тестових

153

глюконеогенезу. З якої

амінокислоти при цьому у

печінці людини найбільш

активно синтезується

глюкоза ?

А. Глутамінова кислота.

В. Валін.

С. Лейцин.

D. Лізин.

Е. *Аланін.

18. Причиною жовтяниці

новонароджених є

порушення звязування

білірубіну в гепатоцитах.

Яка речовина використо-

вується для утворення

конюгату ?

А. Молочна кислота.

В. Піровиноградна кислота.

С. Сечова кислота.

D. Сірчана кислота.

Е. *Глюкуронова кислота.

19. До пологового

відділення госпіталізували

жінку зі слабкою

пологовою діяльністю.

Який засіб необхідно

використати для

стимуляції скорочень

матки ?

А. Соматостатин.

В. Кортиколіберин.

С. Вазопресин.

D. Гонадоліберин.

Е. *Окситоцин.

20. Після видалення у

пацієнта 2/3 шлунка у

крові зменшився вміст

гемоглобіну, кількість

еритроцитів, збільшилися

розміри цих клітин крові.

Дефіцит якого вітаміну

призводить до таких змін у

крові ?

А. РР.

В. С.

С. Р.

D. В6.

Е. *В12.

21. Хворий на туберкульоз

у складі комбінованої

терапії одержує ізоніазид.

Через деякий час пацієнт

почав скаржитися на

мязову слабість, зниження

чутливості шкіри,

порушення зору,

координації рухів. Який

вітамінний препарат

доцільно використати для

усунення даних явищ ?

А. Вітамін D.

В. Вітамін С.

С. Вітамін А.

D. Вітамін В12.

Е. *Вітамін В6.

Page 154: КРОК-1 БІОХІМІЯ Збірник тестових завданьlib.sumdu.edu.ua/library/docs/rio/2020/m4695.pdf · 2 Крок-1. Біохімія Н: збірник тестових

154

22. Дитина 1 року відстає у

розумовому розвитку від

своїх однолітків. Уранці

відзначаються блювання,

судоми, непритомність. У

крові – гіпоглікемія натще.

З дефектом якого

ферменту це повязано ?

А. Аргінази.

В. Фосфорилази.

С. Сахарази.

D. Лактази.

Е. *Глікогенсинтази.

23. До лікарні доставлений

хворий з отруєнням

інсектицидом – ротеноном.

Яка ділянка міто-

хондріального ланцюга

перенесення електронів

блокується цією

речовиною ?

А. Сукцинат-коензим Q-

редуктаза.

В. Цитохром С-оксидаза.

С. Коензим Q- цитохром С-

редуктаза.

D. АТФ-синтетаза.

Е. *НАДН- коензим Q-

редуктаза.

2.6. КРОК-2012

1. У дорослої людини за

добу виділяється 20 л сечі

із низькою густиною.

Найбільш імовірною

причиною цього є дефіцит

в організмі:

А. Альдостерону.

В. Реніну.

С. Паратгормону.

D.Натрійуретичного

фактора.

Е. *Вазопресину.

2. У хворого нормально

забарвлений кал, у складі

якого знаходиться велика

кількість вільних жирних

кислот. Причиною цього є

порушення такого

процесу:

А. Гідролізу жирів.

В. Секреції ліпаз.

С. Жовчовиділення.

D. Жовчоутворення.

Е. *Усмоктування жирів.

3. У пацієнта з підвищеним

артеріальним тиском,

тремором, тахікардією

було діагностовано

доброякісну пухлину

мозкової речовини

наднирників. Гіпер-

Page 155: КРОК-1 БІОХІМІЯ Збірник тестових завданьlib.sumdu.edu.ua/library/docs/rio/2020/m4695.pdf · 2 Крок-1. Біохімія Н: збірник тестових

155

секреція якого гормону

викликає таку симптома-

тику ?

А. Інсулін.

В. Соматотропін.

С. Тироксин.

D. Глюкагон.

Е. *Адреналін.

4. При хворобі Вільсона-

Коновалова порушується

транспорт міді, що

призводить до накопи-

чення цього металу в

клітинах мозку та печінки.

З порушенням синтезу

якого білка це пов’язано ?

А. Сидерофілін.

В. Металотіонеїн.

С. Гаптоглобін.

D. Транскобаламін.

Е. *Церулоплазмін.

5. У хворого хлопчика 12

років вміст холестерину в

сироватці крові

25 ммоль/л. В анамнезі –

спадкова сімейна гіпер-

холестеринемія, причиною

якої є порушення синтезу

білків рецепторів до:

А. Ліпопротеїнів високої

щільності.

В. Ліпопротеїнів дуже

низької щільності.

С. Хіломікронів.

D. Ліпопротеїнів проміжної

щільності.

Е. *Ліпопротеїнів низької

щільності.

6. У тварини через 2 тижні

після експериментального

звуження ниркової артерії

підвищився артеріальний

тиск. Зі збільшенням дії на

судини якого фактора

гуморальної регуляції це

пов’язано ?

А. Альдостерону.

В. Дофаміну.

С. Вазопресину.

D. Кортизолу.

Е. *Ангіотензину ІІ.

7. При нестачі біотину

спостерігається порушен-

ня синтезу вищих жирних

кислот. Утворення якого із

зазначених метаболітів

може бути порушено при

цьому ?

А. Серотоніну.

В. Пірувату.

С. Сукциніл-КоА.

D. Аланіну.

Е. *Малоніл-КоА.

8. В експерименті на кролі

встановлено, що об’єм

Page 156: КРОК-1 БІОХІМІЯ Збірник тестових завданьlib.sumdu.edu.ua/library/docs/rio/2020/m4695.pdf · 2 Крок-1. Біохімія Н: збірник тестових

156

кисню, який споживається

головним мозком за 1 хв,

дорівнює об’єму СО2, який

виділяється клітинами

мозку в кров. Це свідчить,

що у клітинах головного

мозку має місце:

А. Гіпокапнія.

В. Окиснення білків.

С. Гіпоксія.

D. Окиснення жирів.

Е. *Окиснення вуглеводів.

9. У хворого діагностовано

алкаптонурію. Назвіть

фермент, дефект якого є

причиною цієї патології:

А. Піруватдегідрогеназа.

В. Фенілаланінгідроксилаза.

С. Глутаматдегідрогеназа.

D. ДОФА-декарбоксилаза.

Е. *Оксидаза

гомогентизинової кислоти.

10. При активації

запального процесу,

деяких аутоімунних та

інфекційних захворюван-

нях у плазмі крові різко

зростає рівень білків

гострої фази. Який із

наведених нижче білків

здатний утворювати гель

при охолодженні

сироватки ?

А. С-реактивний білок.

В. Гаптоглобін.

С. α2-Макроглобулін.

D. Церулоплазмін.

Е. *Кріоглобулін.

11. На прийом до

терапевта прийшов

чоловік 37 років зі

скаргами на періодичні

інтенсивні больові напади

у суглобах великого

пальця стопи та їх

припухлість. У сечі: кисла

реакція і рожеве

забарвлення. З наявністю

яких речовин можуть бути

пов’язані такі зміни ?

А. Хлоридів.

В. Фосфат кальцію.

С. Сульфат магнію.

D. Амонієві солі.

Е. *Солей сечової кислоти.

12. При токсичному

пошкодженні клітин

печінки з порушенням її

функцій у хворого

з’явилися набряки. Які

зміни складу плазми крові

є основною причиною

розвитку набряків ?

А. Зменшення вмісту

фібриногену.

Page 157: КРОК-1 БІОХІМІЯ Збірник тестових завданьlib.sumdu.edu.ua/library/docs/rio/2020/m4695.pdf · 2 Крок-1. Біохімія Н: збірник тестових

157

В. Зменшення вмісту

глобулінів.

С. Збільшення вмісту

глобулінів.

D. Збільшення вмісту

альбумінів.

Е. *Зниження вмісту

альбумінів.

13. Хворий після

перенесеного паротиту

схуднув, постійно відчуває

спрагу, п’є багато води,

відмічає часте сечо-

виділення, підвищений

апетит, шкірний свербіж,

слабість, фурункульоз. У

крові: глюкоза –

16 ммоль/л, кетонові тіла –

100 мкмоль/л; глюкозурія.

Яке захворювання розви-

нулось у пацієнта ?

А. Цукровий діабет

недостатнього харчування.

В. Нецукровий діабет.

С. Інсулінонезалежний

цукровий діабет.

D. Стероїдний діабет.

Е. *Інсулінозалежний

цукровий діабет.

14. Недостатність в

організмі мікроелемента

селену проявляється

кардіоміопатією. Імовір-

ною причиною такого

стану є зниження

активності такого селен-

вмісного ферменту:

А. Каталаза.

В. Сукцинатдегідрогеназа.

С. Цитохромоксидаза.

D. Лактатдегідрогеназа.

Е. *Глутатіонпероксидаза.

15. Батьки хлопчика 3

років звернули увагу на

потемніння кольору його

сечі при відстоюванні.

Об’єктивно: температура у

нормі, шкірні покриви

чисті, печінка не

збільшена. Назвіть

імовірну причину даного

стану:

А. Подагра.

В. Синдром Іценка-

Кушинга.

С. Гемоліз.

D. Фенілкетонурія.

Е. *Алкаптонурія.

16. До лікарні потрапив 9-

річний хлопчик розумово і

фізично відсталий. При

біохімічному дослідженні

крові: підвищена кількість

фенілаланіну. Блокування

якого ферменту може

Page 158: КРОК-1 БІОХІМІЯ Збірник тестових завданьlib.sumdu.edu.ua/library/docs/rio/2020/m4695.pdf · 2 Крок-1. Біохімія Н: збірник тестових

158

призвести до такого

стану ?

А. Глутамінтрансаміназа.

В. Аспартатаміно-

трансфераза.

С. Оксидаза

гомогентизинової кислоти.

D. Глутаматдекарбоксилаза.

Е. *Фенілаланін-4-

монооксигеназа.

17. У людини збільшений

вміст іонів кальцію в

плазмі крові, зменшений у

кістках. Надмірна секреція

якого гормону може

спричинити такі зміни ?

А. Трийодтиронін.

В. Альдостерон.

С. Тироксин.

D. Тиреокальцитонін.

Е. *Паратгормон.

18. У хворого з дихальною

недостатністю рН крові

7,35. Визначення рСО2

показало наявність

гіперкапнії. При

дослідженні рН сечі

відзначається підвищення

її кислотності. Яка форма

порушення кислотно-

основного стану в даному

випадку ?

А. Алкалоз газовий,

некомпенсований.

В. Ацидоз метаболічний,

некомпенсований.

С. Алкалоз газовий,

компенсований.

D. Ацидоз метаболічний,

компенсований.

Е. *Ацидоз газовий,

компенсований.

19. У людей, які постійно

проживають у гірській

місцевості, адаптація до

«кисневого голодування»

здійснюється шляхом

полегшеної віддачі кисню

гемоглобіном унаслідок:

А. Зниження температури

крові.

В. Зростання парціального

тиску СО2.

С. Зниженого утворення 2,3-

дифосфогліцерату в

еритроцитах.

D. Підвищення рН крові.

Е. *Підвищеного утворення

2,3-дифосфогліцерату в

еритроцитах.

20. Хвора 38 років

доставлена до реанімацій-

ного відділення у

несвідомому стані.

Рефлекси відсутні. Цукор

Page 159: КРОК-1 БІОХІМІЯ Збірник тестових завданьlib.sumdu.edu.ua/library/docs/rio/2020/m4695.pdf · 2 Крок-1. Біохімія Н: збірник тестових

159

крові – 2,1 ммоль/л. В

анамнезі – цукровий діабет

із 18 років. Яка кома має

місце у хворої ?

А. Лактацидемічна.

В. Кетоацидотична.

С. Гіперосмолярна.

D. Гіперглікемічна.

Е. *Гіпоглікемічна.

21. Метильні групи (-СН3)

використовуються в

організмі для синтезу

таких важливих сполук, як

креатин, холін, адреналін,

інші. Джерелом цих груп є

одна з незамінних

амінокислот, а саме:

А. Лейцин.

В. Триптофан.

С. Ізолейцин.

D. Валін.

Е. *Метіонін.

22. При недостатності

кровообігу у період

інтенсивної м’язової

роботи у м’язах в

результаті анаеробного

гліколізу накопичується

молочна кислота. Як вона

утилізується ?

А. Видаляється через нирки

із сечею.

В. Використовується у

м’язах для синтезу

амінокислот.

С. Використовується

тканинами для синтезу

кетонових тіл.

D. Використовується у

тканинах для синтезу

жирних кислот.

Е. *Включається в

глюконеогенез у печінці.

23. В експерименті

показано, що при саркомі

Ієнсена споживання

глюкози з привідної до

пухлини артерії значно

збільшується, має місце

також приріст вмісту

молочної кислоти у

відвідній вені. Про що

свідчить дане явище ?

А. Зменшення анаеробного

гліколізу.

В. Посилення окисних

процесів.

С. Посилення окиснення

білків.

D. Зменшення окисних

процесів.

Е. *Посилення анаеробного

гліколізу.

24. У хлопчика 3 років із

вираженим геморагічним

Page 160: КРОК-1 БІОХІМІЯ Збірник тестових завданьlib.sumdu.edu.ua/library/docs/rio/2020/m4695.pdf · 2 Крок-1. Біохімія Н: збірник тестових

160

синдромом відсутній

антигемофільний глобулін

А (фактор VIII) у плазмі

крові. Яка фаза гемостазу

первинно порушена у

цього хворого ?

А. Зовнішній механізм

активації протромбінази.

В. Ретракція кров’яного

згустка.

С. Перетворення

протромбіну в тромбін.

D. Перетворення

фібриногену у фібрин.

Е. *Внутрішній механізм

активації протромбінази.

25. Для лікування деяких

інфекційних захворювань,

викликаних бактеріями,

застосовуються сульфаніл-

амідні препарати, що

блокують синтез фактора

росту бактерій. Назвіть

механізм їх дії:

А. Є алостеричними

ферментами.

В. Є алостеричними

інгібіторами ферментів.

С. Беруть участь в окисно-

відновних процесах.

D. Інгібують всмоктування

фолієвої кислоти.

Е. *Є антивітамінами пара-

амінобензойної кислоти.

26. Чоловік 60 років

скаржиться на біль у

суглобах. У сироватці

крові пацієнта виявлено

підвищення концентрації

С-реактивного білка та

оксипроліну. Для якого

захворювання характерні

ці симптоми ?

А. Гепатит.

В. Цукровий діабет.

С. Подагра.

D. Жовтяниця.

Е. *Ревматизм.

27. У хворого, прооперова-

ного з приводу «гострого

живота», сеча коричневого

кольору, кількість

індикану в сечі вище

93 ммоль/добу. Про що це

свідчить ?

А. Порушення фільтраційної

здатності нирок.

В. Збільшення швидкості

окисного дезамінування

ароматичних амінокислот.

С. Зниження активності

ферментів орнітинового

циклу.

D. Зниження інтенсивності

знешкодження аміаку.

Е. *Збільшення

інтенсивності гниття білків

у кишечнику.

Page 161: КРОК-1 БІОХІМІЯ Збірник тестових завданьlib.sumdu.edu.ua/library/docs/rio/2020/m4695.pdf · 2 Крок-1. Біохімія Н: збірник тестових

161

2.7. КРОК-2013

1. Хворий 47-ми рокiв з

дiагнозом вогнищевий

туберкульоз верхньої долi

правої легенi, в складi

комбiнованої терапiї

одержує iзонiазид. Через

деякий час пацiєнт почав

пред’являти скарги на

м’язову слабкiсть, знижен-

ня шкiрної чутливостi,

порушення зору, коорди-

нацiї рухiв. Який

вiтамiнний препарат

доцiльно використати для

усунення даних явищ ?

A. Вiтамiн C.

B. Вiтамiн A.

C. Вiтамiн D.

D. Вiтамiн B12.

E. *Вiтамiн B6.

2. Похiднi птерину (амiно-

птерин i метотрексат) - є

конкурентними iнгiбiтора-

ми дигiдрофолатредуктази,

внаслiдок чого вони

пригнiчують регенерацiю

тетрагiдрофолiєвої кисло-

ти з дигiдрофолату. Цi

лiкарськi засоби призво-

дять до гальмування

мiжмолекулярного транс-

порту одновуглецевих

груп. Бiосинтез якого

полiмеру при цьому

пригнiчується?

A. Глiкозамiноглiкани.

B. Бiлок.

C. Гомополiсахариди.

D. Ганглiозиди.

E. *ДНК.

3. Бiохiмiчний аналiз

сироватки кровi пацiєнта з

хворобою Вiльсона-

Коновалова виявив

зниження вмiсту

церулоплазмiну. У цього

пацiєнта в сироватцi кровi

буде пiдвищена концен-

трацiя таких iонiв:

A. Натрiй.

B. Кальцiй.

C. Фосфор.

D. Калiй.

E. *Мiдь.

4. В процесi метаболiзму в

органiзмi людини

виникають активнi форми

кисню, у тому числi

супероксидний анiон-

радикал О-2. Цей анiон

iнактивується за

допомогою ферменту:

A. Глутатiонредуктаза.

B. Каталаза.

C. Пероксидаза.

Page 162: КРОК-1 БІОХІМІЯ Збірник тестових завданьlib.sumdu.edu.ua/library/docs/rio/2020/m4695.pdf · 2 Крок-1. Біохімія Н: збірник тестових

162

D. Глутатiонпероксидаза.

E. *Супероксиддисмутаза.

5. Жiнцi 54-х рокiв

поставили попереднiй

дiагноз: iнфаркт мiокарда.

Характерною ознакою

даного захворювання є

суттєве пiдвищення в

кровi активностi такого

ферменту:

A. Аргiназа.

B. Каталаза.

C. Г-6-ФДГ.

D. Альфа-амiлаза.

E. *Креатинфосфокiназа.

6. Електрофоретичне до-

слiдження сироватки кровi

хворого на пневмонiю

показало збiльшення однiєї

з бiлкових фракцiй.

Вкажiть її:

A. β-глобулiни.

B. Альбумiни.

C. α1-глобулiни.

D. α2-глобулiни.

E. *γ-глобулiни.

7. На основi лабораторного

аналiзу у хворого

пiдтверджено дiагноз -

подагра. Для встановлення

дiагнозу було проведено

визначення вмiсту:

A. Амiаку в сечi.

B. Креатинiну в сечi.

C. Залишкового азоту в

кровi.

D. Сечовини в кровi та сечi.

E.*Сечової кислоти в кровi

та сечi.

8. Для лiкування деяких

iнфекцiйних захворювань,

викликаних бактерiями,

застосовуються сульфанiл-

амiднi препарати, що

блокують синтез фактора

росту бактерiй. Назвiть

механiзм їх дiї:

A. Є алостеричними

ферментами.

B. Iнгiбують всмоктування

фолiєвої кислоти.

C. Є алостеричними

iнгiбiторами ферментiв.

D. Беруть участь в окисно-

вiдновних процесах.

E. *Є антивiтамiнами пара-

амiнобензойної кислоти.

9. При глiкогенозi - хворобi

Гiрке - порушується

перетворення глюкозо-6-

фосфату на глюкозу, що

призводить до

накопичення глiкогену в

печiнцi та нирках. Дефiцит

Page 163: КРОК-1 БІОХІМІЯ Збірник тестових завданьlib.sumdu.edu.ua/library/docs/rio/2020/m4695.pdf · 2 Крок-1. Біохімія Н: збірник тестових

163

якого ферменту є

причиною захворювання ?

A. Альдолаза.

B. Глiкогенсинтетаза.

C. Фосфорилаза.

D. Гексокiназа.

E. *Глюкозо-6-фосфатаза.

10. У хворого 20-ти рокiв з

жовтяницею встановлено:

пiдвищення у плазмi кровi

вмiсту загального

бiлiрубiну за рахунок

непрямого (вiльного), в

калi та сечi - високий вмiст

стеркобiлiну, рiвень

прямого (зв’язаного)

бiлiрубiну в плазмi кровi в

межах норми. Про який

вид жовтяницi можна

думати?

A. Хвороба Жильбера.

B. Паренхiматозна

(печiнкова).

C. Механiчна.

D.Жовтяниця немовлят.

E. *Гемолiтична.

11. Для утворення

транспортної форми

амiнокислот для синтезу

бiлка необхiдно:

A. Ревертаза.

B. ГТФ.

C. м-РНК.

D. Рибосома.

E. *Амiноацил-тРНК-

синтетаза.

12. Одна з форм вродженої

патологiї супроводжується

гальмуванням перетво-

рення фенiлаланiну в

тирозин. Бiохiмiчною

ознакою хвороби є

накопичення в органiзмi

деяких органiчних кислот,

зокрема:

A. Глутамiнова.

B. Лимонна.

C. Пiровиноградна.

D. Молочна.

E. *Фенiлпiровиноградна.

13. У хворого 28-ми рокiв

тривале блювання

призвело до зневоднення

органiзму. Пiдвищена

секрецiя якого гормону,

перш за все, сприятиме

збереженню води в

органiзмi ?

A. Альдостерон.

B. Кальцитонiн.

C. Тироксин.

D. Соматостатин.

E. *Вазопресин.

14. Катiоннi глiкопротеїни

є основними компо-

Page 164: КРОК-1 БІОХІМІЯ Збірник тестових завданьlib.sumdu.edu.ua/library/docs/rio/2020/m4695.pdf · 2 Крок-1. Біохімія Н: збірник тестових

164

нентами слини привушних

залоз. Якi амiнокислоти

обумовлюють їх позитив-

ний заряд ?

A. Цистеїн, глiцин, пролiн.

B. Аспартат, глутамат,

глiцин.

C. Аспартат, аргiнiн,

глутамат.

D. Глутамат, валiн, лейцин.

E. *Лiзин, аргiнiн, гiстидин.

15. До бiорегуляторiв

клiтинних функцiй

лiпiдної природи належать

тромбоксани. Джерелом

для синтезу цих сполук є:

A. Пальмiтоолеїнова

кислота.

B. Стеаринова кислота.

C. Пальмiтинова кислота.

D. Фосфатидна кислота.

E. *Арахiдонова кислота.

16. Синтез i-РНК прохо-

дить на матрицi ДНК з

урахуванням принципу

комплементарностi. Якщо

триплети у ДНК наступнi -

АТГ-ЦГТ, то вiдповiднi

кодони i-РНК будуть:

A. ТАГ-УГУ.

B. АУГ-ЦГУ.

C. АТГ-ЦГТ.

D. УАГ-ЦГУ.

E. *УАЦ-ГЦА.

17. При декарбоксилюван-

нi глутамату утворюється

нейромедiатор гама-

амiномасляна кислота

(ГАМК). При розпадi

ГАМК перетворюється у

метаболiт циклу лимонної

кислоти, яким є:

A. Оксалоацетат.

B. Лимонна кислота.

C. Малат.

D. Фумарат.

E. *Сукцинат.

18. Жiнка 26-ти рокiв

поступила у пологове

вiддiлення в термiнi

вагiтностi 40 тижнiв.

Шийка матки розкрита,

але скорочення матки

вiдсутнє. Лiкар дав засiб

гормональної природи для

посилення пологової

дiяльностi. Назвiть засiб:

A. АКТГ.

B. Гiдрокортизон.

C. Естрон.

D. Тестостерон.

E. *Окситоцин.

19. У клiтинi в грану-

лярнiй ЕПС вiдбувається

етап трансляцiї, при якому

Page 165: КРОК-1 БІОХІМІЯ Збірник тестових завданьlib.sumdu.edu.ua/library/docs/rio/2020/m4695.pdf · 2 Крок-1. Біохімія Н: збірник тестових

165

спостерiгається просуван-

ня i-РНК щодо рибосоми.

Амiнокислоти з’єднуються

пептидними зв’язками в

певнiй послiдовностi -

вiдбувається бiосинтез

полiпептиду. Послiдов-

нiсть амiнокислот у

полiпептидi буде вiдповi-

дати послiдовностi:

A. Антикодонiв р-РНК.

B. Нуклеотидiв т-РНК.

C. Антикодонiв т-РНК.

D. Нуклеотидiв р-РНК.

E. *Кодонiв i-РНК.

20. У результатi побутової

травми у пацiєнта виникла

значна крововтрата, що

супроводжувалося знижен-

ням артерiального тиску.

Дiя яких гормонiв

забезпечує швидке

вiдновлення кров’яного

тиску, викликаного

крововтратою?

A. Альдостерон.

B. Кортизол.

C. Статевi.

D. Окситоцин.

E. *Адреналiн, вазопресин.

21. Лiкарi-iнфекцiонiсти

широко застосовують

антибiотики, якi iнгiбують

синтез нуклеїнових

кислот. Який етап

бiосинтезу гальмує

рифампiцин?

A. Сплайсинг у прокарiотах

i еукарiотах.

B. Транскрипцiя в

прокарiотах i еукарiотах.

C. Реплiкацiя в прокарiотах.

D. Термiнацiя транскрипцiї в

прокарiотах i еукарiотах

E. *Iнiцiацiя транскрипцiї в

прокарiотах.

22. Вiдомо,що введення в

органiзм людини

лiкарського препарату

дикумаролу викликає

рiзке зниження в кровi

вмiсту протромбiну i ряду

iнших бiлкових факторiв

згортання кровi. Антивiта-

мiном якого вiтамiну є

дикумарол ?

A. Вiтамiн H.

B. Вiтамiн C.

C. Вiтамiн E.

D. Вiтамiн P.

E. *Вiтамiн K.

23. У хлопчика 11-ти рокiв

вмiст холестерину в

сироватцi кровi до 25

ммоль/л. В анамнезi -

спадкова сiмейна гiпер-

Page 166: КРОК-1 БІОХІМІЯ Збірник тестових завданьlib.sumdu.edu.ua/library/docs/rio/2020/m4695.pdf · 2 Крок-1. Біохімія Н: збірник тестових

166

холестеринемiя, причиною

якої є порушення синтезу

бiлкiв-рецепторiв до:

A. Лiпопротеїнiв промiжної

щiльностi.

B. Лiпопротеїнiв високої

щiльностi.

C. Хiломiкронiв.

D. Лiпопротеїнiв дуже

низької щiльностi.

E. *Лiпопротеїнiв низької

щiльностi.

2.8. КРОК-2014

1. У сироватцi кровi

пацiєнта встановлено

пiдвищення активностi

гiалуронiдази. Визначення

якого бiохiмiчного показ-

ника сироватки кровi

дозволить пiдтвердити

припущення про патоло-

гiю сполучної тканини ?

A. Галактоза.

B. Бiлiрубiн.

C. Сечова кислота.

D. Глюкоза.

E. *Сiаловi кислоти.

2. Пiсля загоєння рани на її

мiсцi утворився рубець.

Яка речовина є основним

компонентом цього

рiзновиду сполучної

тканини ?

A. Кератансульфат.

B. Еластин.

C. Гiалуронова кислота.

D. Хондроiтин-сульфат.

E. *Колаген.

3. У чоловiка 35-ти рокiв

феохромоцитома. В кровi

виявляється пiдвищений

рiвень адреналiну та

норадреналiну, концентра-

цiя вiльних жирних кислот

зросла в 11 разiв.

Активацiя якого ферменту

пiд впливом адреналiну

пiдвищує лiполiз?

A. Холестеролестераза.

B. Лiпопротеїдлiпаза.

C. Фосфолiпаза А2.

D. Фосфолiпаза С.

E. *ТАГ-лiпаза.

4. При обтурацiйнiй жовтя-

ницi часто спостерiгається

протромбiнова недостат-

нiсть. З дефiцитом в

органiзмi якого вiтамiну це

пов’язано?

A. Е.

B. B6.

C. A.

D. C.

Е. *К.

Page 167: КРОК-1 БІОХІМІЯ Збірник тестових завданьlib.sumdu.edu.ua/library/docs/rio/2020/m4695.pdf · 2 Крок-1. Біохімія Н: збірник тестових

167

5. При дослiдженнi кровi

хворого виявлено значне

збiльшення активностi

МВ-форм КФК (креатин-

фосфокiнази) та ЛДГ-1.

Яку патологiю можна

припустити?

A. Холецистит.

B. Гепатит.

C. Ревматизм.

D. Панкреатит.

E. *Iнфаркт мiокарда.

6. У чоловiка 53-х рокiв

дiагностовано сечокам’яну

хворобу з утворенням

уратiв. Цьому пацiєнту

призначено аллопуринол,

який є конкурентним

iнгiбiтором ферменту:

A. Уридилтрансфераза.

B. Уреаза.

C. Уратоксидаза.

D. Дигiдроурацил-

дегiдрогеназа.

E. *Ксантиноксидаза.

7. Крива дисоцiацiї

оксигемоглобiну змiщена

вправо. Якi змiни в

органiзмi людини можуть

бути причиною цього?

A. Гiпертермiя.

B. Гiпоксемiя.

C. Алкалоз.

D. Гiпокапнiя.

E. *Збiльшення концентрацiї

2,3-дифосфоглiцерату в

еритроцитах.

8. Цианiстий калiй є

отрутою, вiд якої смерть

органiзму наступає

миттєво. На якi ферменти

в мiтохондрiях дiє

цианістий калiй?

A. Цитохром Р-450.

B. Флавiновi ферменти.

C. Цитохром b5.

D. НАД+-залежнi

дегiдрогенази.

E. *Цитохромоксидаза (аа3).

9. При рiзноманiтних

захворюваннях рiвень

активних форм кисню

рiзко зростає, що

призводить до руйнування

клiтинних мембран. Для

запобiгання цьому

використовують антиокси-

данти. Найпотужнiшим

природним антиоксидан-

том є:

A. Глiцерол.

B. Глюкоза.

C. Вiтамiн D.

D. Жирнi кислоти.

E. *Альфа-токоферол.

Page 168: КРОК-1 БІОХІМІЯ Збірник тестових завданьlib.sumdu.edu.ua/library/docs/rio/2020/m4695.pdf · 2 Крок-1. Біохімія Н: збірник тестових

168

10. Чоловiк 53-х рокiв

доставлений у стацiонар у

непритомному станi.

Об’єктивно: шкiра суха,

дихання часте поверхневе,

запах ацетону вiдсутнiй,

Ps- 126/хв., АТ- 70/40 мм

рт.ст. Вмiст глюкози у

кровi 48 ммоль/л, реакцiя

сечi на ацетон негативна.

Для якого iз перелiчених

станiв найбiльш

характернi симптоми у

хворого ?

A. Колапс.

B. Гiперкетонемiчна кома.

C. Лактацидемiчна кома.

D. Токсична кома.

E. *Гiперосмолярна кома.

11. Глiкоген, що надiйшов

з їжею, гiдролiзувався у

шлунково-кишковому

трактi. Який кiнцевий

продукт утворився в

результатi цього процесу?

A. Фруктоза.

B. Лактат.

C. Лактоза.

D. Галактоза.

E. *Глюкоза.

12. Для пiдвищення

результатiв спортсмену

рекомендували застосову-

вати препарат, який

мiстить у собi карнiтин.

Який процес в

найбiльшому ступенi

активується карнiтином?

A. Синтез стероїдних

гормонiв.

B. Синтез кетонових тiл.

C. Синтез лiпiдiв.

D. Тканинне дихання.

E. *Транспорт жирних

кислот у мiтохондрiї.

13. Хворому з прогресую-

чою м’язовою дистрофiєю

було проведено бiохiмiчне

дослiдження сечi. Поява

якої речовини у великiй

кiлькостi в сечi може

пiдтвердити захворювання

м’язiв у даного хворого?

A. Креатинiн.

B. Порфiрини.

C. Сечовина.

D. Гiпурова кислота.

E. *Креатин.

14. При обстеженнi

чоловiка 45-ти рокiв, що

тривалий час перебував на

рослиннiй дiєтi, виявлено

негативний азотистий

баланс. Яка особливiсть

рацiону стала причиною

цього явища?

Page 169: КРОК-1 БІОХІМІЯ Збірник тестових завданьlib.sumdu.edu.ua/library/docs/rio/2020/m4695.pdf · 2 Крок-1. Біохімія Н: збірник тестових

169

A. Недостатня кiлькiсть

жирiв i бiлкiв.

B. Надмiрна кiлькiсть води.

C. Надмiрна кiлькiсть

вуглеводiв.

D. Недостатня кiлькiсть

жирiв.

E. *Недостатня кiлькiсть

бiлкiв.

15. У жiнки обмежений

кровотiк у нирках,

пiдвищений артерiальний

тиск. Гiперсекрецiя якого

гормону зумовила пiдви-

щення тиску?

A. Вазопресин.

B. Адреналiн.

C. Норадреналiн.

D. Еритропоетин.

E. *Ренiн.

16. У чоловiка 41-го року

вiдзначаються перiодичнi

напади серцебиття

(пароксизми), сильне

потовидiлення, напади

головного болю. При

обстеженнi виявлена

гiпертензiя, гiперглiкемiя,

пiдвищення основного

обмiну, тахiкардiя. При

якiй патологiї наднирникiв

спостерiгається подiбна

картина ?

A. Первинний

альдостеронiзм.

B. Гiпофункцiя мозкового

шару.

C. Гiперфункцiя кори

наднирникiв.

D. Гiпофункцiя кори

наднирникiв.

E. *Гiперфункцiя мозкового

шару.

17. Пацiєнт 16-ти рокiв, що

страждає на хворобу

Iценко-Кушинга, консу-

льтований з приводу

надмiрної ваги тiла. При

опитуваннi з’ясувалося,

що енергетична цiннiсть

спожитої їжi складає 1700 -

1900 ккал/добу. Яка

провiдна причина

ожирiння у даному

випадку?

A. Гiподинамiя.

B. Нестача iнсулiну.

C. Надлишок iнсулiну.

D. Нестача

глюкокортикоїдiв.

E. *Надлишок

глюкокортикоїдiв.

18. Лiмфоцит уражений

ретровiрусом ВIЛ (СНIД).

В цьому випадку

Page 170: КРОК-1 БІОХІМІЯ Збірник тестових завданьlib.sumdu.edu.ua/library/docs/rio/2020/m4695.pdf · 2 Крок-1. Біохімія Н: збірник тестових

170

напрямок потоку

iнформацiї в клiтинi буде:

A. Полiпептид → РНК →

ДНК →i-РНК.

B. ДНК → i-РНК →

полiпептид →ДНК.

C. ДНК → полiпептид → i-

РНК.

D. i-РНК → полiпептид →

ДНК.

E. *РНК → ДНК → i-РНК →

полiпептид.

19. У пацiєнта у результатi

тривалого блювання

вiдбувається значна

втрата шлункового соку,

що є причиною порушення

кислотно-лужного стану в

органiзмi. Яка з

перерахованих форм

порушення кислотно-

лужного стану має мiсце?

A. Метаболiчний ацидоз.

B. Газовий ацидоз.

C. Негазовий ацидоз.

D. Газовий алкалоз.

E. *Негазовий алкалоз.

20. Препарат "Гептрал",

який використовують при

хворобах печiнки, мiстить

S-аденозилметiонiн. Ця

активна амiнокислота бере

участь у синтезi:

A. Гему.

B.Жовчних кислот.

C. Триацилглiцеролiв.

D. Холестерину.

E. *Фосфолiпiдiв.

21. Хвора 39-ти рокiв, з

цукровим дiабетом в

анамнезi, госпiталiзована

до клiнiки у

прекоматозному станi

кетоацидотичного типу.

Збiльшення вмiсту якого

метаболiту до цього

призвело ?

A. Аспартат.

B. Цитрат.

C. Альфа-кетоглутарат.

D. Малонат.

E. *Ацетоацетат.

22. Дихальний коефiцiєнт у

хворого складає 0,7. Це

свiдчить, що у клiтинах

людини переважає:

A. Змiшане окислення жирiв

та бiлкiв.

B. Окислення вуглеводiв.

C. Окислення бiлкiв.

D. Змiшане окислення жирiв

та вуглеводiв.

E. *Окислення жирiв.

23. У хворого 49-ти рокiв

на гострий панкреатит

Page 171: КРОК-1 БІОХІМІЯ Збірник тестових завданьlib.sumdu.edu.ua/library/docs/rio/2020/m4695.pdf · 2 Крок-1. Біохімія Н: збірник тестових

171

виникала загроза некрозу

пiдшлункової залози, що

супроводжувалось надход-

женням у кров i тканини

активних панкреатичних

протеїназ i розщепленням

тканинних бiлкiв. Якi

захиснi фактори органiзму

можуть iнгiбувати цi

процеси?

A. Гемоплексин,

гаптоглобiн.

B. Iмуноглобулiни.

C. Крiоглобулiн, iнтерферон.

D. Церулоплазмiн,

трансферин.

E. *α2-макроглобулiн,

α1-антитрипсин.

24. У 19-мiсячної дитини iз

затримкою розвитку та

проявами самоагресiї,

вмiст сечової кислоти в

кровi - 1,96 ммоль/л. При

якому метаболiчному

порушеннi це спостерiгає-

ться ?

A. Хвороба Iценко-Кушинга.

B. Подагра.

C. Синдром набутого

iмунодефiциту.

D. Хвороба Гiрке.

E. *Синдром Леша-Нiхана.

25. Чоловiк 53-х рокiв

звернувся зi скаргами на

гострий бiль у правому

пiдребер’ї. При оглядi

лiкар звернув увагу на

пожовтiлi склери хворого.

Лабораторнi аналiзи

показали пiдвищену

активнiсть АЛТ та

негативну реакцiю на

стеркобiлiн у калi. Для

якого захворювання хара-

ктернi такi симптоми ?

A. Хронiчний гастрит.

B. Гемолiтична жовтяниця.

C. Гепатит.

D. Хронiчний колiт.

E. *Жовчнокам’яна хвороба.

26. Основними тригерами,

що включають ефекторнi

системи клiтини у

вiдповiдь на дiю гормонiв,

є протеїнкiнази, якi

змiнюють каталiтичну

активнiсть певних

регуляторних ферментiв

шляхом АТФ-залежного

фосфорилювання. Який iз

наведених ферментiв є

активним у фосфорильо-

ванiй формi?

A. Глiкогенсинтаза.

B. Ацетил-КоА-

карбоксилаза.

Page 172: КРОК-1 БІОХІМІЯ Збірник тестових завданьlib.sumdu.edu.ua/library/docs/rio/2020/m4695.pdf · 2 Крок-1. Біохімія Н: збірник тестових

172

C. ГОМГ-КоА-редуктаза.

D. Пiруваткiназа.

E. *Глiкогенфосфорилаза.

27. При алкаптонурiї

вiдбувається надмiрне

видiлення гомогентизи-

нової кислоти iз сечею. З

порушенням метаболiзму

якої амiнокислоти

пов’язано виникнення

цього захворювання?

A. Аспарагiн.

B. Фенiлаланiн.

C. Аланiн.

D. Метiонiн.

E. *Тирозин.

28. У хворого, що страждає

на спадкову хворобу

Хартнупа, спостерiгаються

пелагроподiбнi ураження

шкiри, порушення розумо-

вого розвитку в результатi

нестачi нiкотинової

кислоти. Причиною цього

захворювання є порушен-

ня такого процесу:

A. Всмоктування i

реабсорбцiя цистеїну.

B. Трансамiнування

фенiлаланiну.

C. Декарбоксилювання

триптофану.

D. Всмоктування i реабсорб-

цiя в нирках метiонiну.

E. *Всмоктування i

реабсорбцiя в нирках

триптофану.

29. Хворий помилково

прийняв надмiрну дозу

тироксину. До яких змiн

секрецiї тиреолiберину та

тиреотропiну це призведе ?

A. Секрецiя тиреотропiну

збiльшиться, тиреолiберину

– зменшиться.

B. Секрецiя гормонiв

збiльшиться.

C. Змiн секрецiї гормонiв не

буде.

D. Секрецiя тиреолiберину

збiльшиться, тиреотропiну –

зменшиться.

E. *Секрецiя гормонiв

зменшиться.

30. У чоловiка 32-х рокiв,

хворого на пневмонiю,

спостерiгається закупорка

харкотинням дихальних

шляхiв. В органiзмi

хворого при цьому буде

розвиватися така змiна

кислотно-лужної рiвно-

ваги:

A. Змiн не буде.

B. Метаболiчний ацидоз.

Page 173: КРОК-1 БІОХІМІЯ Збірник тестових завданьlib.sumdu.edu.ua/library/docs/rio/2020/m4695.pdf · 2 Крок-1. Біохімія Н: збірник тестових

173

C. Респiраторний алкалоз.

D. Метаболiчний алкалоз.

E. *Респiраторний ацидоз.

31. При регенерацiї

епiтелiю слизової оболонки

порожнини рота

(розмноження клiтин)

вiдбулася реплiкацiя (авто-

репродукцiя) ДНК за

напiвконсервативним ме-

ханiзмом. При цьому

нуклеотиди нової нитки

ДНК є комплементарними

до:

A. Ферменту РНК-полiме-

рази.

B. Змiстовних кодонiв.

C. Ферменту ДНК-полiме-

рази.

D. Iнтронних дiлянок гену.

E. *Материнської нитки.

32. Для профiлактики

атеросклерозу, iшемiчної

хвороби серця, порушень

мозкового кровообiгу

рекомендується спожи-

вання жирiв iз високим

вмiстом полiненасичених

жирних кислот. Однiєю з

таких жирних кислот є:

A. Стеаринова.

B. Олеїнова.

C. Лауринова.

D. Пальмiтоолеїнова.

E. *Лiнолева.

33. При обстеженнi жiнки

56-ти рокiв, що хвора на

цукровий дiабет 1-го типу,

виявлене порушення

бiлкового обмiну, що при

лабораторному дослiд-

женнi кровi проявляється

амiноацидемiєю та

клiнiчно - уповiльненням

загоєння ран i зменшенням

синтезу антитiл. Який з

перерахованих механiзмiв

викликає розвиток

амiноацидемiї?

A. Збiльшення лiпопротеїдiв

високої щiльностi.

B. Гiперпротеїнемiя.

C. Зменшення концентрацiї

амiнокислот у кровi.

D. Пiдвищення онкотичного

тиску в плазмi кровi.

E. *Пiдвищення протеолiзу.

34. Чоловiковi 58-ми рокiв

зроблено операцiю з

приводу раку простати.

Через 3 мiсяцi йому

проведено курс променевої

та хiмiотерапiї. До

комплексу лiкарських

препаратiв входив 5-фтор-

дезоксиуридин - iнгiбiтор

Page 174: КРОК-1 БІОХІМІЯ Збірник тестових завданьlib.sumdu.edu.ua/library/docs/rio/2020/m4695.pdf · 2 Крок-1. Біохімія Н: збірник тестових

174

тимiдилатсинтази. Синтез

якої речовини блокується

цим препаратом ?

A. мяРНК.

B. i-РНК.

C. р-РНК.

D. т-РНК.

E. *ДНК.

35. Стеатоз виникає

внаслiдок накопичення

триацилглiцеролiв у

гепатоцитах. Одним з

механiзмiв розвитку цього

захворювання є зменшен-

ня утилiзацiї нейтрального

жиру ЛПДНЩ. Якi

лiпотропнi речовини

попереджують розвиток

стеатозу?

A. Iзолейцин, B1, B2.

B. Аргiнiн, B2, B3.

C. Аланiн, B1, PP.

D. Валiн, B3, B2.

E. *Метiонiн, B6, B12.

36. У чоловiка 40-ка рокiв

внаслiдок посиленого

гемолiзу еритроцитiв

пiдвищився вмiст залiза в

плазмi кровi. Який бiлок

забезпечує його депо-

нування в тканинах?

A. Альбумiн.

B. Гаптоглобiн.

C. Трансферин.

D. Транскортин.

E. *Феритин.

37. Хвороба Андерсена

належить до групи

спадкових хвороб, що

розвиваються внаслiдок

уродженої недостатностi

синтезу певних ферментiв

глiкогенолiзу. Недостат-

нiсть якого ферменту є

молекулярною основою

цього глiкогенозу?

A. Фосфофруктокiназа.

B. Глiкогенсинтаза.

C. Глюкозо-6-фосфатаза.

D. Лiзосомальнi глiкозидази.

E. *Амiло(1,4-1,6)транс-

глiкозидаза.

38. У недоношеного

немовляти спостерiгається

жовтяниця. З нестачею

якого ферменту це

пов’язано ?

A. НАД+ - дегiдрогеназа.

B. Лужна фосфатаза.

C. Кисла фосфатаза.

D. Каталаза.

E. *УДФ-глюкуронiл-

трансфераза.

39. У жiнки 36-ти рокiв має

мiсце гiповiтамiноз B2.

Page 175: КРОК-1 БІОХІМІЯ Збірник тестових завданьlib.sumdu.edu.ua/library/docs/rio/2020/m4695.pdf · 2 Крок-1. Біохімія Н: збірник тестових

175

Причиною виникнення

специфiчних симптомiв

(ураження епiтелiю, слизо-

вих, шкiри, рогiвки ока)

iмовiрно є дефiцит:

A. Цитохрому С.

B. Цитохрому А1.

C. Цитохромоксидази.

D. Цитохрому В.

E. *Флавiнових

коферментiв.

40. Порушення процесiв

мiєлiнiзацiї нервових

волокон призводить до

неврологiчних розладiв i

розумової вiдсталостi. Такi

симптоми характернi для

спадкових i набутих

порушень обмiну:

A. Фосфатидної кислоти.

B. Нейтральних жирiв.

C. Вищих жирних кислот.

D. Холестерину.

E. *Сфiнголiпiдiв.

41. У хворого 15-ти рокiв

концентрацiя глюкози

натще 4,8 ммоль/л, через

годину пiсля цукрового

навантаження становить

9,0 ммоль/л, через 2 години

- 7,0 ммоль/л, через 3

години - 4,8 ммоль/л. Цi

показники характернi для

такого захворювання:

A. Хвороба Гірке.

B. Цукровий дiабет I типу.

C. Цукровий дiабет II типу.

D. Хвороба Iценко-Кушинга.

E.*Прихований цукровий

дiабет.

42. Спецiальний режим

харчування призвiв до

зменшення iонiв Ca2+ в

кровi. До збiльшення

секрецiї якого гормону це

призведе?

A. Тироксин.

B. Тирокальцитонiн.

C. Вазопресин.

D. Соматотропiн.

E. *Паратгормон.

43. Молода людина 25-ти

рокiв споживає надмiрну

кiлькiсть вуглеводiв (600 г

на добу), що перевищує її

енергетичнi потреби. Який

процес буде активуватися

в органiзмi людини у

даному випадку ?

A. Окиснення жирних

кислот.

B. Глiколiз.

C. Лiполiз.

D. Глюконеогенез.

E. *Лiпогенез.

Page 176: КРОК-1 БІОХІМІЯ Збірник тестових завданьlib.sumdu.edu.ua/library/docs/rio/2020/m4695.pdf · 2 Крок-1. Біохімія Н: збірник тестових

176

2.9. КРОК-2015

1. У хворого нормально

забарвлений кал, у складі

якого є велика кількість

вільних жирних кислот.

Причиною цього є

порушення:

А. Жовчоутворення.

В. Жовчовиділення.

С. Гідролізу жирів.

D. Секреції ліпаз.

E. *Всмоктування жирів.

2. У пацієнта стома-

тологічного відділення

виявлено хворобу

Педжета, що супроводжу-

ється деградацією

колагену. Вирішальним

фактом для постановки

діагнозу було виявлення у

сечі хворого підвищеного

рівня:

А. Аланіну.

В. Аргініну.

С. Триптофану.

D. Серину.

E. *Оксипроліну.

3. Після переходу до

змішаного харчування у

новонародженої дитини

виникла диспепсія з

діареєю, метеоризмом,

відставанням у розвитку.

Біохімічна основа даної

патології полягає у

недостатності:

А. Целюлази.

В. Ліпази та креатинкінази.

С. Лактази та целобіази.

D. Трипсину та

хімотрипсину.

E. *Сахарази та ізомальтази.

4. Сучасні антиатеро-

склеротичні препарати

застосовуються з метою

профілактики та

лікування атеросклерозу.

Такі препарати як

гемфіброзил та фенфібрат

гальмують біосинтез

холестеролу шляхом

інгібування ферменту:

А. Глюкозо-6-фосфатаза.

В. Ацилтрансфераза.

С. Ацил-КоА-холестерин-

ацилтрансфераза.

D. Гексокіназа.

E. *β-ГОМК-редуктаза.

5. При зниженні актив-

ності ферментів анти-

оксидантного захисту

посилюються процеси

перекисного окиснення

ліпідів клітинних мембран.

При нестачі якого

Page 177: КРОК-1 БІОХІМІЯ Збірник тестових завданьlib.sumdu.edu.ua/library/docs/rio/2020/m4695.pdf · 2 Крок-1. Біохімія Н: збірник тестових

177

мікроелементу знижується

активність глутатіон-

пероксидази ?

А. Мідь.

В. Кобальт.

С. Молібден.

D. Марганець.

E. *Селен.

6. Пацієнт 46-років зверну-

вся до лікаря зі скаргами

на болі в дрібних суглобах

ніг та рук. Суглоби

збільшені, мають вигляд

потовщених вузлів. У

сироватці встановлено

підвищений вміст уратів.

Це може бути спричинене:

А. Порушенням обміну

амінокислот.

В. Порушенням обміну

вуглеводів.

С. Порушенням обміну

ліпідів.

D. Порушенням обміну

піримідинів.

E. *Порушенням обміну

пуринів.

7. При напруженій фізич-

ній роботі у мязовій

тканині накопичується

молочна кислота, яка

дифундує в кров і

підхоплюється печінкою

та серцем. Який процес

забезпечує відновлення

запасів глікогену в мязах?

А. Пентозофосфатний шлях.

В. Орнітиновий цикл.

С. Цикл лимонної кислоти.

D. Цикл трикарбонових

кислот.

E. *Цикл Корі.

8. В ході експерименту

було продемонстровано

підвищення активності -

галактозидази після

внесення лактози до

культурального середо-

вища з E.coli. Яка ділянка

лактозного оперону стає

розблокованою від

репресору за цих умов ?

А. Промотор.

В. Праймер.

С. Структурний ген.

D. Регуляторний ген.

E. *Оператор.

9. До складу харчових

раціонів обовязково

входять продукти, в яких є

клітковина. Відомо, що

вона не перетравлюється

ферментами травного

тракту й не засвоюється

організмом. Яку роль

відіграє ця речовина ?

Page 178: КРОК-1 БІОХІМІЯ Збірник тестових завданьlib.sumdu.edu.ua/library/docs/rio/2020/m4695.pdf · 2 Крок-1. Біохімія Н: збірник тестових

178

А. Гальмує всмоктувальну

функцію травного каналу.

В. Гальмує процеси

виділення ферментів

травних соків.

С. Гальмує секреторну

функцію травного каналу.

D. Гальмує моторну

функцію травного каналу.

E. *Стимулює моторну

функцію травного каналу.

10. У пацієнта встановлено

гіповітаміноз фолієвої

кислоти, що може

призвести до порушення

синтезу:

А. Гема та креатину.

В. Тимідилових нуклеотидів

та жирних кислот.

С. Пуринових нуклеотидів

та холестерину.

D. Цитрату та кетонових тіл.

E. *Пуринових та

тимідилових нуклеотидів.

11. У пятирічного

хлопчика спостерігалися

малий зріст, розумове

відставання, обмежені

рухи, грубі риси обличчя.

Ці особливості стали

помітними з 18-місячного

віку. У нього виявили

дефіцит L-ідуронідази.

Обмін яких сполук

порушено ?

А. Нуклеотиди.

В. Вітаміни.

С. Білки.

D. Фосфоліпіди.

Е. *Глікозаміноглікани.

12. У синтезі пуринових

нуклеотидів беруть участь

деякі амінокислоти,

похідні вітамінів, фосфорні

ефіри рибози. Кофермент-

на форма якого вітаміну є

переносником одновугле-

цевих фрагментів в цьому

синтезі ?

А. Рибофлавін.

В. Пантотенова кислота.

С. Піридоксин.

D. Нікотинова кислота.

Е. *Фолієва кислота.

13. Для лiкування

урогенiтальних iнфекцiй

використовують хiнолони

- iнгiбiтори ферменту

ДНК-гiрази. Який процес

порушується пiд дiєю

хiнолонiв у першу чергу ?

А. Рекомбiнацiягенiв.

В. Зворотна транскрипцiя.

С. Амплiфiкацiягенiв.

D. Репарацiя ДНК.

Е. *Реплiкацiя ДНК.

Page 179: КРОК-1 БІОХІМІЯ Збірник тестових завданьlib.sumdu.edu.ua/library/docs/rio/2020/m4695.pdf · 2 Крок-1. Біохімія Н: збірник тестових

179

14. У чоловiка 35-ти рокiв

феохромоцитома. В кровi

спостерiгається пiдвище-

ний рiвень адреналiну та

норадреналiну, концен-

трацiя вiльних жирних

кислотзросла в 11 разiв.

Активацiя якого ферменту

пiд впливом адреналiну

пiдвищує лiполiз ?

А. Лiпопротеїдлiпаза.

В. Фосфолiпаза А2.

С. Фосфолiпаза С.

D. Холестеролестераза.

Е. *ТАГ-лiпаза.

15. У хлопчика 2-х рокiв

спостерiгається збiльшен-

ня в розмiрах печiнки та

селезiнки, катаракта. В

кровi пiдвищена

концентрацiя цукру, але

тест толерантностi до

глюкози в нормi. Спадкове

порушення обмiну якої

речовини є причиною

цього стану ?

А. Сахароза.

В. Мальтоза.

С. Глюкоза.

D. Фруктоза.

Е. *Галактоза.

16. В сечi новонародженого

визначається цитрулiн та

високий рiвень амiаку.

Вкажiть, утворення якої

речовини, найiмовiрнiше,

порушене у цього малюка:

А. Амiак.

В. Креатинiн.

С. Креатин.

D. Сечова кислота.

Е. *Сечовина.

17. У хворого з дихальною

недостатнiстю рН кровi

7,35. Визначення pCO2

показало наявнiсть

гiперкапнiї. При

дослiдженнi рН сечi

вiдзначається пiдвищення

її кислотностi. Яка форма

порушення кислотно-

основного стану в даному

випадку ?

А. Алкалоз газовий,

декомпенсований

В. Алкалоз газовий,

компенсований

С. Ацидоз метаболiчний,

декомпенсований

D. Ацидоз метаболiчний,

компенсований

Е. *Ацидоз газовий,

компенсований.

18. У хворого з

жовтяницею встановлено:

пiдвищення у плазмi кровi

Page 180: КРОК-1 БІОХІМІЯ Збірник тестових завданьlib.sumdu.edu.ua/library/docs/rio/2020/m4695.pdf · 2 Крок-1. Біохімія Н: збірник тестових

180

вмiсту загального

бiлiрубiну за рахунок

непрямого (вiльного), в

калi та сечi – високий

вмiст стеркобiлiну, рiвень

прямого (зв’язаного)

бiлiрубiну в плазмi кровi в

межах норми. Про який

вид жовтяницi можна

думати ?

A. Паренхiматозна

(печiнкова).

B. Механiчна.

C. Хвороба Жильбера.

D. Жовтяниця немовлят.

E. *Гемолiтична.

19. У хворого

вiдзначаються перiодичнi

напади серцебиття

(пароксизми), сильне

потовидiлення, напади

головного болю. При

обстеженнi виявлена

гiпертензiя, гiперглiкемiя,

пiдвищення основного

обмiну, тахiардiя. При якiй

патологiї наднирникiв

спостерiгається подiбна

картина?

A. Первинний

альдостеронiзм.

B. Гiпофункцiя мозкового

шару.

C. Гiперфункцiя кори

наднирникiв.

D. Гiпофункцiя кори

наднирникiв.

E. *Гiперфункцiя мозкового

шару.

20. Знешкодження ксено-

бiотикiв (лiкарських

засобiв, епоксидiв,

ареноксидiв, альдегiдiв,

нiтропохiдних тощо) та

ендогенних метаболiтiв

(естрадiолу, простагланди-

нiв, лейкотрiєнiв)

вiдбувається в печiнцi

шляхом їх кон’югацiї з:

A. Фосфоаденозином.

B. Аспарагiновою кислотою.

C. Глiцином.

D. S-Аденозилметiонiном.

E. *Глутатiоном.

21. Бiосинтез пуринового

кiльця вiдбувається на

рибозо-5-фосфатi шляхом

поступового нарощення

атомiв азоту i вуглецю та

замикання кiлець.

Джерелом рибозофосфату є

наступний процес:

A. Глiкогенолiз.

B. Глiколiз.

C. Глiконеогенез.

D. Глюконеогенез.

E. *Пентозофосфатний цикл.

Page 181: КРОК-1 БІОХІМІЯ Збірник тестових завданьlib.sumdu.edu.ua/library/docs/rio/2020/m4695.pdf · 2 Крок-1. Біохімія Н: збірник тестових

181

22. У пацiєнта у результатi

тривалого блювання

вiдбувається значна

втрата шлункового соку,

що є причиною порушення

кислотно-лужного стану в

органiзмi. Яка з

перерахованих форм

порушення кислотно-

лужного стану має мiсце ? A. Метаболiчний ацидоз.

B. Газовий ацидоз.

C. Негазовий ацидоз.

D. Газовий алкалоз.

E. *Негазовий алкалоз.

23. Хворий 55-ти рокiв

хворiє на хронiчний

гломерулонефрит на

протязі 15-ти рокiв. Якi

змiни складу кровi або сечi

найбiльш характерно

свiдчать про обмеження

секреторної функцiї

нирок ?

А. Гiпо-, iзостенурiя.

B. Гiперглiкемiя.

C. Гiпопротеїнемiя.

D. Протеїнурiя.

E. *Гiперазотемiя.

24. До клiнiки доставлено

хвору на цукровий дiабет,

госпiталiзовано у

прекоматозному станi

кетоацидотичного типу.

Збiльшення вмiсту якого

метаболiту до цього

призвело ?

A. Аспартат.

B. Цитрат.

C. α-Кетоглутарат.

D. Малонат.

E. *Ацетоацетат.

25. Основними тригерами,

що вмикають ефекторнi

системи клiтини у

вiдповiдь на дiю гормонiв,

є протеїнкiнази, якi

змiнюють каталiтичну

активнiсть певних

регуляторних ферментiв

шляхом АТФ-залежного

фосфорилювання. Який з

наведених ферментiв є

активним у фосфорильо-

ванiй формi ?

A. Глiкогенсинтаза.

B. Ацетил-КоА-карбокси-

лаза.

C. ГОМГ-КоА-редуктаза.

D. Пiруваткiназа.

E. *Глiкогенфосфорилаза.

26. Iнозитолтрифосфати в

тканинах органiзму

утворюються в результатi

гiдролiзу фосфатидил-

iнозитолдифосфатiв i

Page 182: КРОК-1 БІОХІМІЯ Збірник тестових завданьlib.sumdu.edu.ua/library/docs/rio/2020/m4695.pdf · 2 Крок-1. Біохімія Н: збірник тестових

182

вiдiграють роль вторинних

посередникiв (месендже-

рiв) в механiзмi дiї

гормонiв. Їхнiй вплив у

клiтинi спрямований на:

A. Гальмування протеїн-

кiнази С.

B. Активацiю аденiлат-

циклази.

C. АктивацiюпротеїнкiназиА

D. Гальмування

фосфодiестерази.

E. *Вивiльнення iонiв

кальцiю з клiтинних депо.

27. У дитини спостерiгає-

ться затримка фiзичного

та розумового розвитку,

глибокi порушення з боку

сполучної тканини

внутрiшнiх органiв, у сечi

виявлено кератан-

сульфати. Обмiн яких

речовин порушений ?

A. Гiалуронова кислота.

B. Колаген.

C. Еластин.

D. Фiбронектин.

E. *Глiкозамiноглiкани.

28. До лiкарнi доставлено

хворого з отруєнням

iнсектицидом - ротеноном.

Яка дiлянка мiто-

хондрiального ланцюга

переносу електронiв

блокується цiєю

речовиною ?

A. АТФ-синтетаза.

B. Сукцинат-коензим Q-

редуктаза.

C. Коензим Q-цитохром С-

редуктаза.

D. Цитохром С-оксидаза.

E. *НАДН-коензим Q-

редуктаза.

29. В ходi регенерацiї

епiтелiю слизової оболонки

порожнини рота (поділ

клiтин) вiдбулася

реплiкацiя (авто-

репродукцiя) ДНК за

напiвконсервативним

механiзмом. При цьому

нуклеотиди нової нитки

ДНК є комплементарними

до:

A. Ферменту РНК-полiме-

рази.

B. Змiстовних кодонiв.

C. Ферменту ДНК-полiме-

рази.

D. Iнтронних дiлянок гену.

E. *Материнської нитки.

30. Серед антиатеро-

склеротичних препаратiв,

що застосовуються з

метою профiлактики та

Page 183: КРОК-1 БІОХІМІЯ Збірник тестових завданьlib.sumdu.edu.ua/library/docs/rio/2020/m4695.pdf · 2 Крок-1. Біохімія Н: збірник тестових

183

лiкування атеросклерозу, є

левостатин. Вiн дiє

шляхом: A. Пригнiчення всмоктуван-

ня холестерину в

кишкiвнику.

B. Усiма наведеними

шляхами.

C. Активацiї метаболiзму

холестерину.

D. Стимулювання екскрецiї

холестерину з органiзму.

E. *Гальмування бiосинтезу

холестерину.

31. При обстеженнi жiнки

56-ти рокiв, що хвора на

цукровий дiабет 1-го типу,

виявлене порушення

бiлкового обмiну, щопри

лабораторному дослiджен-

нi кровi проявляється

амiноацидемiєю, а клiнiчно

- уповiльненням загоєння

ран i зменшенням синтезу

антитiл. Який з

перерахованих механiзмiв

викликає розвиток

амiноацидемiї ?

A. Збiльшення лiпопротеїдiв

високої щiльностi.

B. Гiперпротеїнемiя.

C. Зменшення концентрацiї

амiнокислот у кровi.

D. Пiдвищення онкотичного

тиску в плазмi кровi.

E. *Пiдвищення протеолiзу.

32. Недостатнiсть в

органiзмi мiкроелементу

селену проявляється

кардiомiопатiєю. Iмовiр-

ною причиною такого

стану є зниження

активностi такого селен-

вмiсного ферменту:

A. Сукцинатдегiдрогеназа.

B. Лактатдегiдрогеназа.

C. Каталаза.

D. Цитохромоксидаза.

E. *Глутатiонпероксидаза.

33. Фенiлкетонурiя – це

захворювання, яке

зумовлено рецесивним

геном, що локалiзується в

аутосомi. Батьки є

гетерозиготами за цим

геном. Вони вже мають

двох хворих синiв i одну

здорову доньку. Яка

iмовiрнiсть, що четверта

дитина, яку вони

очiкують, народиться теж

хворою ?

A. 100%.

B. 0%.

C. 50% .

D. 75%.

E. *25%.

Page 184: КРОК-1 БІОХІМІЯ Збірник тестових завданьlib.sumdu.edu.ua/library/docs/rio/2020/m4695.pdf · 2 Крок-1. Біохімія Н: збірник тестових

184

34. У жiнки 36-ти рокiв має

мiсце гiповiтамiноз B2.

Причиною виникнення

специфiчних симптомiв

(ураження епiтелiю,

слизових оболонок, шкiри,

рогiвки ока) iмовiрно є

дефiцит:

A. Цитохрому С.

B. Цитохрому А1.

C. Цитохромоксидази.

D. ЦитохромуВ.

E. *Флавiнових

коферментiв.

35. У хворого 40-ка рокiв

ознаки гiрської хвороби:

запаморочення, задишка,

тахiкардiя, рН кровi - 7,50,

pCO2 – 30 мм рт. ст., зсув

буферних основ + 4

ммоль/л. Яке порушення

кислотно-основного стану

має мiсце ?

A. Видiльний ацидоз.

B. Негазовий алкалоз.

C. Негазовий ацидоз.

D. Газовий ацидоз.

E. *Газовий алкалоз.

36. Пiдшлункова залоза –

орган змiшаної секрецiї.

Ендокринно продукує

бета-клiтинами гормон

iнсулiн, який впливає на

обмiн вуглеводiв. Як вiн

впливає на активнiсть

глiкогенфосфорилази (ГФ)

i глiкогенсинтетази (ГС) ?

A. Не впливає на активнiсть

ГФ i ГС.

B. Активує ГФ i.ГС.

C. Пригнiчує ГФ i ГС.

D. Активує ГФ, пригнiчує

ГС.

E. *Пригнiчує ГФ, активує

ГС.

37. Тривале лiкування

гiпофункцiї щитоподiбної

залози спричинило

загальну дистрофiю, карiєс

зубiв, тахiкардiю, тремор

кiнцiвок. Який лiкарський

засiб викликав зазначенi

побiчнi ефекти ?

A. Преднiзолон.

B. Хумулiн.

C. Паратиреоїдин.

D. Тирокальцитонiн.

E. *L-Тироксин.

38. Молода людина 25-ти

рокiв споживає надмiрну

кiлькiсть вуглеводiв (600 г

на добу), що перевищує її

енергетичнi потреби. Який

процес буде активуватися

в органiзмi людини у

даному випадку ?

Page 185: КРОК-1 БІОХІМІЯ Збірник тестових завданьlib.sumdu.edu.ua/library/docs/rio/2020/m4695.pdf · 2 Крок-1. Біохімія Н: збірник тестових

185

A. Окиснення жирних

кислот.

B. Глiколiз.

C. Лiполiз.

D. Глюконеогенез.

E. *Лiпогенез.

39. До лiкарнi звернувся

чоловiк 50-ти роiв з

розладами пам’ятi,

болiсними вiдчуттями по

ходу нервових стовбурiв,

зниженням iнтелектуаль-

них функцiй, порушен-

нями з боку серцево-

судинної системи i

явищами диспепсiї. В

анамнезi хронiчний

алкоголiзм. Дефiцит якого

вiтамiну може викликати

цi симптоми ?

A. Рибофлавiн.

B. Нiацин.

C. Ретинол.

D. Кальциферол.

E. *Тiамiн.

40. По приїздi групи

експертiв на мiсце злочину

виявлено тiло без ознак

життя. В ходi дослiдження

кровi загиблого виявлена

велика концентрацiя iонiв

цiанової кислоти.

Iнгiбування якого

комплексу дихального

ланцюга мiтохондрiй стало

причиною смертi ?

A. V.

B. I.

C. II.

D. III.

E. *IV.

41. Пацiєнт звернувся зi

скаргами на гострий бiль у

правому пiдребер’ї. При

оглядi лiкар звернув увагу

на пожовтiння склер

хворого. Лабораторно:

пiдвищена активнiсть

АлАТ та негативна

реакцiя на стеркобiлiн в

калi. Для якого

захворювання характернi

такi симптоми ?

A. Хронiчний гастрит.

B. Гемолiтична жовтяниця.

C. Хронiчний

гастродуоденiт.

D. Хронiчний колiт.

E. *Гепатит.

42. У туриста пiд час

тривалого перебування на

спекотi вiдбулася значна

втрата води, що

супроводжувалося рiзким

зниженням дiурезу.

Посилення секрецiї яких

Page 186: КРОК-1 БІОХІМІЯ Збірник тестових завданьlib.sumdu.edu.ua/library/docs/rio/2020/m4695.pdf · 2 Крок-1. Біохімія Н: збірник тестових

186

гормонiв вiдбувається при

цьому ?

A. Серотонiн i дофамiн.

B. Адреналiн i норадреналiн.

C. Глюкокортикоїди та

iнсулiн.

D. Тироксин i трийод-

тиронiн.

E. *Вазопресин та

альдостерон.

2.10. КРОК-2016

1. У хворого iз захворю-

ванням печiнки виявлено

зниження вмiсту протром-

бiну в кровi. Це призведе,

перш за все, до порушення:

A. Антикоагулянтних

властивостей кровi.

B. Першої фази

коагуляцiйного гемостазу.

C. Судинно-тромбоцитарно-

го гемостазу.

D. Фiбринолiзу.

E. *Другої фази

коагуляцiйного гемостазу.

2. При патологiчних

процесах, якi супровод-

жуються гiпоксiєю,

вiдбувається неповне

вiдновлення молекули

кисню в дихальному

ланцюзi i накопичення

пероксиду водню. Вкажiть

фермент, який забезпечує

його руйнування:

A. Аконiтаза.

B. Цитохромоксидаза.

C. Сукцинатдегiдрогеназа.

D. Альфа-кетоглутарат-

дегiдрогеназа.

E. *Каталаза.

3. У жiнки обмежений

кровотiк у нирках,

пiдвищений артерiальний

тиск. Гiперсекрецiя якого

гормону зумовила

пiдвищений тиск ?

A. Вазопресин.

B. Адреналiн.

C. Норадреналiн.

D. Еритропоетин.

E. *Ренiн.

4. У хворої внаслiдок

запалення порушена

ендокринна функцiя

фолiкулярних клiтин

фолiкулiв яєчника. Синтез

яких гормонiв буде

пригнiчений ?

A. Фолiстатин.

B. Прогестерон.

C. Лютропiн.

D. Фолiкулостимулюючий

гормон.

Page 187: КРОК-1 БІОХІМІЯ Збірник тестових завданьlib.sumdu.edu.ua/library/docs/rio/2020/m4695.pdf · 2 Крок-1. Біохімія Н: збірник тестових

187

E. *Естрогени.

5. Хворому з метою

попередження жирової

дистрофiї печiнки лiкар

призначив лiпотропний

препарат-донор метильних

груп. Це iмовiрно:

A. Глюкоза.

B. Холестерин.

C. Бiлiрубiн.

D. Валiн.

E. *S-Аденозилметiонiн.

6. У клiнiчнiй практицi

застосовують для

лiкування туберкульозу

препарат iзонiазид-

антивiтамiн, який здатний

проникати у туберкульоз-

ну паличку. Туберкуло-

статичний ефект

обумовлений порушенням

процесiв реплiкацiї,

окисно-вiдновних реакцiй

завдяки утворенню

несправжнього

коферменту з:

A. КоQ.

B. ФАД.

C. ФМН.

D. ТДФ.

E. *НАД.

7. Синтез i-РНК проходить

на матрицi ДНК з

урахуванням принципу

комплементарностi. Якщо

триплети у ДНК наступнi -

АТГ-ЦГТ, то вiдповiднi

кодони i-РНК будуть:

A. ТАГ-УГУ.

B. АУГ-ЦГУ.

C. АТГ-ЦГТ.

D. УАГ-ЦГУ.

E. *УАЦ-ГЦА.

8. У процесi фiбринолiзу

кров’яний тромб

розсмоктується. Розщеп-

лення нерозчинного

фiбрину вiдбувається

шляхом його гiдролiзу пiд

дiєю протеолiтичного

ферменту плазмiну, який

наявний у кровi в

неактивнiй формi

плазмiногену. Активується

плазмiноген шляхом

обмеженого протеолiзу за

участю фермента:

A. Хiмотрипсин.

B. Пепсин.

C. Трипсин.

D. Ентерокiназа.

E. *Урокiназа.

9. Пацiєнту, який

знаходився в клiнiцi з

Page 188: КРОК-1 БІОХІМІЯ Збірник тестових завданьlib.sumdu.edu.ua/library/docs/rio/2020/m4695.pdf · 2 Крок-1. Біохімія Н: збірник тестових

188

приводу пневмонiї,

ускладненої плевритом, у

складi комплексної терапiї

вводили преднiзолон.

Протизапальна дiя цього

синтетичного глюко-

кортикоїда пов’язана з

блокуванням вивiльнення

арахiдонової кислоти

шляхом гальмування:

A. Циклооксигенази.

B. Пероксидази.

C. Фосфолiпази С.

D. Лiпоксигенази.

E. * Фосфолiпази А2.

10. Спадкова гiперлiпо-

протеїнемiя I типу

обумовлена недостатнiстю

лiпопротеїнлiпази. Пiдви-

щення рiвня яких

транспортних форм лiпiдiв

в плазмi, навiть натще-

серце, є характерним?

A. Модифiкованi

лiпопротеїни.

B.Лiпопротеїни низької

густини.

C.Лiпопротеїни дуже

низької густини.

D. Лiпопротеїни високої

густини.

E. *Хiломiкрони.

11. До лiкарнi звернувся

чоловiк 50-ти рокiв з

розладами пам’ятi,

болiсними вiдчуттями по

ходу нервових стовбурiв,

зниженням iнтелектуаль-

них функцiй, порушен-

нями з боку серцево-

судинної системи i

явищами диспепсiї. В

анамнезi хронiчний

алкоголiзм. Дефiцит якого

вiтамiну може викликати

цi симптоми?

A. Рибофлавiн.

B. Нiацин.

C. Ретинол.

D. Кальциферол.

E. *Тiамiн.

12. Пацiєнт звернувся зi

скаргами на гострий бiль у

правому пiдребер’ї. При

оглядi лiкар звернув увагу

на пожовтiння склер

хворого. Лабораторно:

пiдвищена активнiсть

АлАТ та негативна

реакцiя на стеркобiлiн в

калi. Для якого

захворювання характернi

такi симптоми ?

A. Хронiчний гастрит.

B. Гемолiтична жовтяниця.

Page 189: КРОК-1 БІОХІМІЯ Збірник тестових завданьlib.sumdu.edu.ua/library/docs/rio/2020/m4695.pdf · 2 Крок-1. Біохімія Н: збірник тестових

189

C. Хронiчний

гастродуоденiт.

D. Хронiчний колiт.

E. *Гепатит.

13. У хворого 41-го року

вiдзначається гiпо-

натрiємiя, гiперкалiємiя,

дегiдратацiя, зниження

артерiального тиску,

м’язова слабкiсть, бради-

кардiя, аритмiя. З

порушенням функцiй яких

гормонiв це пов’язано ?

A. Гормони мозкової

речовини наднирникiв.

B. Тиреоїднi.

C. Гормони пiдшлункової

залози.

D. Статевi гормони.

E. .*Кортикостероїди.

14. Хворому з ревматоїд-

ним артритом тривалий

час вводили гiдро-

кортизон. У нього

з’явилися гiперглiкемiя,

полiурiя, глюкозурiя,

спрага. Цi ускладнення

лiкування є наслiдком

активацiї процесу:

A. Лiполiз.

B. Глiкогенолiз.

C. Глiкогенез.

D. Глiколiз.

E. *Глюконеогенез.

15. Хворому перед

операцiєю було введено

дитилiн (лiстенон) i

проведено iнтубацiю.

Дефiцит якого ферменту

ворганiзмi хворого

подовжує дiю м’язового

релаксанту ?

A. K-Na-АТФ-аза.

B. Сукцинатдегiдрогеназа.

C. Карбангiдраза.

D. N-ацетилтрансфераза.

E. *Псевдохолiнестераза.

16. У немовляти спостерi-

гаються епiлептиформнi

судоми, викликанi

дефiцитом вiтамiну В6. Це

спричинено зменшенням у

нервовiй тканинi гальмiв-

ного медiатора - γ-амiно-

масляної кислоти. Актив-

нiсть якого ферменту

знижена при цьому ? A. Глутаматсинтетаза.

B. Аланiнамiнотрансфераза.

C. Глутаматдегiдрогеназа.

D. Пiридоксалькiназа .

E. *Глутаматдекарбокси-

лаза.

17. При обстеженнi хворого

виявлена характерна

Page 190: КРОК-1 БІОХІМІЯ Збірник тестових завданьlib.sumdu.edu.ua/library/docs/rio/2020/m4695.pdf · 2 Крок-1. Біохімія Н: збірник тестових

190

клiнiка колагенозу.

Вкажiть, збiльшення якого

показника сечi характерне

для цiєї патологiї:

A. Солi амонiю.

B. Аргiнiн.

C. Глюкоза.

D. Мiнеральнi солi.

E. *Гiдроксипролiн.

18. Електрофоретичне

дослiдження сироватки

кровi хворого пневмонiєю

показало збiльшення одної

з бiлкових фракцiй.

Вкажiть її: A. β-глобулiни.

B. Альбумiни.

C. α1-глобулiни.

D. α2-глобулiни.

E. *γ-глобулiни.

19. У чоловiка 53-х рокiв

дiагностовано сечокам’яну

хворобу з утворенням

уратiв. Цьому пацiєнту

призначено аллопурiнол,

який є конкурентним

iнгiбiтором ферменту:

A. Уридiлтрансфераза.

B. Уреаза.

C. Уратоксидаза.

D. Дигiдроурацилдегiдро-

геназа.

E. *Ксантиноксидаза.

20. Мати зауважила

занадто темну сечу у її 5-

рiчної дитини. Дитина

скарг не висловлює.

Жовчних пiгментiв у сечi

не виявлено. Поставлено

дiагноз алкаптонурiя.

Дефiцит якого ферменту

має мiсце у дитини ?

A. Декарбоксилаза фенiл-

пiрувату.

B. Фенiлаланiнгiдроксилаза.

C. Тирозиназа.

D. Оксидаза оксифенiл-

пiрувату.

E. *Оксидаза гомогентизи-

нової кислоти.

21. У вiддiлення реанiмацiї

надiйшов чоловiк 47-ми

рокiв з дiагнозом iнфаркт

мiокарда. Яка з фракцiй

лактатдегiдрогенази (ЛДГ)

буде переважати в

сироватцi кровi впродовж

перших двох дiб

захворювання ?

A. ЛДГ5.

B. ЛДГ4.

C. ЛДГ3.

D. ЛДГ2.

E. *ЛДГ1.

22. Глiкоген, що надiйшов

з їжею, гiдролiзується у

Page 191: КРОК-1 БІОХІМІЯ Збірник тестових завданьlib.sumdu.edu.ua/library/docs/rio/2020/m4695.pdf · 2 Крок-1. Біохімія Н: збірник тестових

191

шлунково-кишковому

трактi. Який кiнцевий

продукт утворюється в

результатi цього процесу ? A. Фруктоза.

B. Лактат.

C. Лактоза.

D. Галактоза.

E. *Глюкоза.

23. Хворому з прогресую-

чою м’язовою дистрофiєю

було проведено бiохiмiчне

дослiдження сечi. Поява

якої речовини у великiй

кiлькості в сечi може

пiдтвердити захворювання

м’язiв у даного хворого ?

A. Креатинiн.

B. Порфiрини.

C. Сечовина.

D. Гiпуровакислота.

E. *Креатин.

24. У жiнки обмежений

кровотiк у нирках,

пiдвищений артерiальний

тиск. Гiперсекрецiя якого

гормону зумовила пiдви-

щений тиск?

А. Вазопресин.

B. Адреналiн.

C. Норадреналiн.

D. Еритропоетин.

E. *Ренiн.

25. Пацiєнт 16-ти рокiв, що

страждає на хворобу

Iценко-Кушинга, консу-

льтований з приводу

надмiрної ваги тiла. При

опитуваннi з’ясувалося,

що енергетична цiннiсть

спожитої їжi складає 1700-

1900 ккал/добу. Яка

провiдна причина

ожирiння у даному

випадку ?

A. Гiподинамiя.

B. Нестача iнсулiну.

C. Надлишок iнсулiну.

D. Нестача глюкокорти-

коїдiв.

E. *Надлишок глюкокорти-

коїдiв.

26. У хворого 37-ми рокiв

на фонi тривалого

застосування антибiотикiв

спостерiгається пiдвищена

кровоточивiсть при

невеликих пошкодженнях.

У кровi - зниження

активностi факторiв

згортання кровi II, VII, IX,

X, подовження часу

згортання кровi. Недостат-

нiстю якого вiтамiну

обумовленi такi змiни ?

A. Вiтамiн E.

B. Вiтамiн A.

Page 192: КРОК-1 БІОХІМІЯ Збірник тестових завданьlib.sumdu.edu.ua/library/docs/rio/2020/m4695.pdf · 2 Крок-1. Біохімія Н: збірник тестових

192

C. Вiтамiн C.

D. Вiтамiн D.

E. *Вiтамiн K.

27. У юнака 20-ти рокiв

дiагностовано спадковий

дефiцит УДФ- глюкуронiл-

трансферази. Пiдвищення

якого показника кровi

пiдтверджує дiагноз?

A. Тваринний iндикан.

B. Прямий (кон’югований)

бiлiрубiн.

C. Уробiлiн.

D. Стеркобiлiноген.

E. *Непрямий (некон’югова-

ний) бiлiру бiн.

28. У людини порушено

всмоктування продуктiв

гiдролiзу жирiв. Причиною

цього може бути дефiцит у

порожнинi тонкої кишки:

A. Жиророзчинних

вiтамiнiв.

B. Жовчних пiгментiв.

C. Лiполiтичних ферментiв.

D. Iонiв натрiю.

E. *Жовчних кислот.

29. У клiнiку госпiталi-

зовано хворого з дiагнозом

карциноїду кишечника.

Аналiз виявив пiдвищену

продукцiю серотонiну,

який утво-рюється з

амiнокислоти триптофан.

Який бiохiмiчний механiзм

лежить в основi даного

процесу ?

A. Утворення парних

сполук.

B. Дезамiнування.

C. Мiкросомальне

окиснення.

D. Трансамiнування.

E. *Декарбоксилювання.

30. У хворого струс

головного мозку, що

супроводжується повтор-

ним блюванням i

задишкою. При обстеженнi

вiдзначено: рН-7,62; pCO2

– 40 мм рт. ст. Яке

порушення кислотно-

основного стану є у

хворого ?

A. Респіраторний ацидоз.

B. Газовий алкалоз.

C. Негазовий ацидоз.

D. Газовий ацидоз.

E. *Негазовий алкалоз.

31. Катiоннi глiкопротеїни

є основними компонен-

тами слини привушних

залоз. Якi амiнокислоти

обумовлюють їх позитив-

ний заряд ?

Page 193: КРОК-1 БІОХІМІЯ Збірник тестових завданьlib.sumdu.edu.ua/library/docs/rio/2020/m4695.pdf · 2 Крок-1. Біохімія Н: збірник тестових

193

A. Цистеїн, глiцин, пролiн.

B. Аспартат, глутамат,

глiцин.

C. Аспартат, аргiнiн,

глутамат.

D. Глутамат, валiн, лейцин.

E. *Лiзин, аргiнiн, гiстидин.

32. При пошкодженнi

клiтини iонiзуючим

випромiнюванням вми-

каються механiзми захисту

i адаптацiї. Який механiзм

вiдновлення порушеного

внутрiшньоклiтинного

гомеостазу реалiзується

при цьому ?

A. Гiпертрофiя мiтохондрiй.

B. Активацiя Ca-опосередко-

ваних клiтинних функцiй.

C. Накопичення Na+ в

клiтинах.

D. Пригнiчення аденiлат-

циклази

E. *Активацiя антиоксидан-

тної системи.

33. При лабораторному

дослiдженнi дитини

виявлено пiдвищений

вмiст у кровi та сечi

лейцину, валiну, ізолей-

цину та їх кетопохiдних.

Сеча має характерний

запах кленового сиропу.

Недостатнiсть якого

ферменту характерна для

цього захворювання ?

A. Фосфофруктомутаза.

B. Амiнотрансфераза.

C. Глюкозо-6-фосфатаза.

D. Фосфофруктокiназа.

E. *Дегiдрогеназа розгалу-

жених амiнокислот.

34. У чоловiка 32-х рокiв,

хворого на пневмонiю,

спостерiгається закупорка

харкотинням дихальних

шляхiв. В органiзмi

хворого при цьому буде

розвиватися така змiна

кислотно-лужної рiвно-

ваги:

A. Змiн не буде.

B. Метаболiчний ацидоз.

C. Респiраторний алкалоз.

D. Метаболiчний алкалоз.

E. *Респiраторний ацидоз.

35. Стресовий стан i

больове вiдчуття у

пацiєнта перед вiзитом до

стоматолога супроводжую-

ться анурiєю (вiдсутнiстю

сечовидiлення). Це явище

зумовлене збiльшенням:

A. Секрецiї адреналiну та

зменшенням вазопресину.

Page 194: КРОК-1 БІОХІМІЯ Збірник тестових завданьlib.sumdu.edu.ua/library/docs/rio/2020/m4695.pdf · 2 Крок-1. Біохімія Н: збірник тестових

194

B. Активностi парасимпа-

тичної нервової системи.

C. Активностi антиноцiце-

птивної системи.

D. Секрецiї вазопресину та

зменшенням адреналiну.

E. *Секрецiї вазопресину та

адреналiну.

36. Для профiлактики

атеросклерозу, iшемiчної

хвороби серця, порушень

мозкового кровообiгу

рекомендується споживан-

ня жирiв iз високим

вмiстом полiненасичених

жирних кислот. Однiєю з

таких жирних кислот є:

A. Стеаринова.

B. Олеїнова.

C. Лауринова.

D. Пальмiтоолеїнова.

E. *Лiнолева.

37. Чоловiковi 58-ми рокiв

зроблено операцiю з

приводу раку простати.

Через 3 мiсяцi йому

проведено курс променевої

та хiмiотерапiї. До

комплексу лiкарських

препаратiв входив 5-фтор-

дезоксиуридин – iнгiбiтор

тимiдилатсинтази. Синтез

якої речовини блокується

цим препаратом ?

A. т-РНК.

B. i-РНК.

C. р-РНК.

D. Білків.

E. *ДНК.

38. При диспансерному

обстеженнi у хворого

знайдено цукор в сечi.

Який найбiльш iмовiрний

механiзм виявлених змiн,

якщо вмiст цукру в кровi

нормальний ?

A. Гiперпродукцiя глюко-

кортикоїдiв наднирниками.

B. Порушення фiльтрацiї

глюкози в клубочковому

вiддiлi нефрона.

C. Недостатня продукцiя

iнсулiну пiдшлунковою

залозою.

D. Iнсулiнорезистентнiсть

рецепторiв клiтин.

E. *Порушення реабсорбцiї

глюкози в канальцях

нефрона.

39. У хворого 15-ти рокiв

концентрацiя глюкози

натще - 4,8 ммоль/л, через

годину пiсля цукрового

навантаження становить

9,0 ммоль/л, через 2 години

Page 195: КРОК-1 БІОХІМІЯ Збірник тестових завданьlib.sumdu.edu.ua/library/docs/rio/2020/m4695.pdf · 2 Крок-1. Біохімія Н: збірник тестових

195

- 7,0 ммоль/л, через 3

години - 4,8 ммоль/л. Цi

показники характернi для

такого за-хворювання:

A. Хвороба Аддісона.

B. Цукровий дiабет I типу.

C. Цукровий дiабет II типу.

D. Хвороба Iценко-Кушiнга.

E. *Прихований цукровий

дiабет.

40. Спецiальний режим

харчування призвiв до

зменшення iонiв Ca2+ в

кровi. До збiльшення

секрецiї якого гормону це

призведе ?

A. Тироксин.

B. Тирокальцитонiн

C. Вазопресин.

D. Соматотропiн.

E. *Паратгормон.

41. Спадкова гiперлiпо-

протеїнемiя I типу

обумовлена недостатнiстю

лiпопротеїнлiпази. Пiдви-

щення рiвня яких

транспортних форм лiпiдiв

в плазмi навiть

натщесерце є характерним

для цієї патології ?

A. Модифiкованi лiпо-

протеїни.

B. Лiпопротеїни низької

густини.

C. Лiпопротеїни дуже

низької густини.

D. Лiпопротеїни високої

густини.

E. *Хiломiкрони.

2.11. КРОК-2017

1. Хворому із больовим

синдромом в суглобах

постійно призначають

аспірин. Який з

перерахованих ферментів

він пригнічує ?

А. Ліпооксигеназа.

В. Фосфоліпаза С.

С. Фосфоліпаза D.

D. Фосфоліпаза А2.

Е. *Циклооксигеназа.

2. У хворого виявлено

ожиріння, гірсутизм,

«місяцеподібне» обличчя,

рубці багряного кольору

на шкірі стегон.

Артеріальний тиск –

180/110 мм.рт.ст., глюкоза

крові – 17,2 ммоль/л. При

якій зміні продукції

гормонів наднирників

можлива така картина ?

Page 196: КРОК-1 БІОХІМІЯ Збірник тестових завданьlib.sumdu.edu.ua/library/docs/rio/2020/m4695.pdf · 2 Крок-1. Біохімія Н: збірник тестових

196

А. Гіпопродукція

глюкокортикоїдів.

В. Гіперпродукція статевих

гормонів.

С. Гіперпродукція

мінералокортикоїдів.

D. Гіпопродукція

мінералокортикоїдів.

Е. *Гіперпродукція

глюкокортикоїдів.

3. Реакції міжмолекуляр-

ного транспорту одно-

вуглецевих радикалів є

необхідними для синтезу

білків та нуклеїнових

кислот. З якого з

наведених нижче вітамінів

утворюється кофермент,

потрібний для вказаних

вище реакцій ?

А. Аскорбінова кислота.

В. Пантотенова кислота.

С. Тіамін.

D. Рибофлавін.

Е. *Фолієва кислота.

4. Щоденно в організмі

людини 0,5% всього

гемоглобіну перетво-

рюється на метгемоглобін.

Який фермент, що

міститься в еритроцитах,

каталізує відновлення

метгемоглобіну до

гемоглобіну ?

А. Глюкоронілтрансфераза.

В. Метгемоглобін-

трансфераза.

С. Гемоксигеназа.

D. Білівердинредуктаза.

Е. *Метгемоглобін-

редуктаза.

5. У чоловіка 25-ти років з

переломом основи черепа

виділяється великий об’єм

сечі з низькою відносною

щільністю. Причиною змін

сечоутворення є

порушення синтезу такого

гормону:

А. Соматотропний гормон.

В. Адренокортикотропний

гормон.

С. Тиреотропний гормон.

D. Окситоцин.

Е. *Вазопресин.

6. Введення тварині

екстракту тканини

передсердя посилює

виділення натрію з сечею.

Дія якої біологічно

активної речовини стала

причиною такого стану ?

А. Адреналін.

В. Калікреїн.

С. Глюкокортикоїд.

Page 197: КРОК-1 БІОХІМІЯ Збірник тестових завданьlib.sumdu.edu.ua/library/docs/rio/2020/m4695.pdf · 2 Крок-1. Біохімія Н: збірник тестових

197

D. Серотонін.

Е. *Натрійуретичний

гормон.

7. Хворому з ревматоїдним

артитом тривалий час

вводили гідрокортизон. У

нього з’явилися

гіперглікемія, поліурія,

глюкозурія, спрага. Ці

ускладнення лікування є

наслідком активації

такого процесу:

А. Глікогеноліз.

В. Гліколіз.

С. Глікогенез.

D. Ліполіз.

Е. *Глюконеогенез.

8. У хворого має місце

хронічний запальний

процес мигдаликів. За

рахунок якого біохімічного

процесу у вогнищі

запалення підтримується

концентрація НАДФН,

необхідного для реалізації

механізму фагоцитозу ?

А. Цикл Корі.

В. Цикл Кребса.

С. Орнітиновий цикл.

D. Гліколіз.

Е. *Пентозо-фосфатний

шлях.

9. Дослідженнями останніх

десятиліть встановлено,

що безпосередніми

«виконавцями» апоптозу в

клітині є особливі

ферменти – каспази. В

утворенні одного з них

бере участь цитохром С.

Вкажіть його функцію в

нормальній клітині:

А. Компонент

піруватдегідрогеназної

системи.

В. Фермент бета-окиснення

жирних кислот.

С. Компонент Н+ - АТФ-

азної системи.

D. Фермент ЦТК.

Е.*Фермент дихального

ланцюга переносу

електронів.

10. У молодого чоловіка

внаслідок подразнення

сонячного сплетення

запальним процесом

(солярит) підвищена

функціональна активність

залоз шлунка, що

виражається, зокрема, у

збільшенні продукції

хлоридної кислоти. Яка з

вказаних нижче речовин

викликає гіперхлоргідрію

у даному випадку ?

Page 198: КРОК-1 БІОХІМІЯ Збірник тестових завданьlib.sumdu.edu.ua/library/docs/rio/2020/m4695.pdf · 2 Крок-1. Біохімія Н: збірник тестових

198

А. Калікреїн.

В. Глюкагон.

С. Урогастрон.

D. Гастроінгібуючий пептид.

Е. *Гастрин.

11. В регуляції фізіоло-

гічних функцій беруть

участь іони металів. Один

із них отримав назву

«король месенджерів».

Таким біоелементом

посередником є:

А.Fe3+.

В. Na+.

С. Zn2+.

D. K+.

Е. *Ca2+.

12. Після операції на

кишечнику у хворого

з’явились симптоми

отруєння аміаком за типом

печінкової коми. Який

механізм дії аміаку на

енергозабезпечення ЦНС ?

А. Інактивація ферментів

дихального ланцюга.

В. Гальмування бета-

окиснення жирних кислот.

С. Гальмування гліколізу.

D. Роз’єднання окисного

фосфорилювання.

Е. *Гальмування ЦТК в

результаті зв’язування

альфа-кетоглутарату.

13. Прокаріотичні та

еукаріотичні клітини

характеризуються здат-

ністю до поділу. Поділ

прокаріотичних клітин

відрізняється від поділу

еукаріотичних, але існує

молекулярний процес,

який лежить в основі цих

поділів. Який це процес ?

А. Ампліфікація генів.

В. Репарація.

С. Транскрипція.

D. Трансляція.

Е. *Реплікація ДНК.

14. У хворого спосте-

рігається тремтіння рук,

що пов’язане з хворобою

Паркінсона. Дефіцит якого

медіатора в стріопалідар-

них структурах призво-

дить до таких симптомів ?

А. Субстанція Р.

В. ГАМК.

С. Серотонін.

D. Норадреналін.

Е. *Дофамін.

15. Препарати групи

вітаміну В2 призначають

Page 199: КРОК-1 БІОХІМІЯ Збірник тестових завданьlib.sumdu.edu.ua/library/docs/rio/2020/m4695.pdf · 2 Крок-1. Біохімія Н: збірник тестових

199

при захворюваннях шкіри.

Завдяки наявності якої

структури в його складі

визначається здатність до

окиснення-відновлення ?

А. Залишок фосфорної

кислоти.

В. Аденін.

С. Рибозофосфат.

D. Рибітол.

Е. *Ізоалоксазин.

16. При пошкодженні

клітини іонізуючим

випромінюванням вми-

каються механізми захисту

і адаптації. Який механізм

відновлення порушеного

внутрішньо-клітинного

гомеостазу реалізується

при цьому ?

А. Пригнічення

аденілатциклази.

В. Активація Са-

опосередкованих клітинних

функцій.

С. Накопичення Na+ в

клітинах.

D. Гіпертрофія мітохондрій.

Е. *Активація

антиоксидантної системи.

17. При недостатності

вітаміну С спостерігається

порушення структури

колагенових волокон. Яка

стадія їх синтезу

порушується при цьому ?

А. Приєднання глюкозних і

галактозних залишків.

В. Відщеплення сигнальних

олігопептидів.

С. Агрегація тропоколагену і

утворення фібрил.

D. Утворення гама-

карбоксиглутамату.

Е. *Утворення гідрокси-

проліну і гідроксилізину.

18. Який вітамінний

препарат доцільно

призначити пацієнту зі

скаргами на зниження

гостроти зору в сутінках,

сухість шкіри, часті

застудні захворювання,

ламкість волосся ?

А. Нікотинова кислота.

В. Ергокальциферол.

С. Тіаміну хлорид.

D. Пантотенова кислота.

Е. *Ретинолу ацетат.

19. Хворому призначили

антибіотик хлорамфенікол

(левомецитин), який

порушує у мікроорганізмів

синтез білку шляхом

гальмування процесу:

А. Транскрипція.

Page 200: КРОК-1 БІОХІМІЯ Збірник тестових завданьlib.sumdu.edu.ua/library/docs/rio/2020/m4695.pdf · 2 Крок-1. Біохімія Н: збірник тестових

200

В. Утворення полірибосом.

С. Процесінг.

D. Ампліфікація генів.

Е. *Елонгація трансляції.

20. У доношеного ново-

народженого спостерігає-

ться жовте забарвлення

шкіри та слизових оболо-

нок. Імовірною причиною

цього стану може бути

тимчасова нестача такого

ферменту:

A. Білівердинредуктаза.

B. Уридинтрансфераза.

C. Гемсинтетаза.

D. Гемоксигеназа.

E. *УДФ-глюкуроніл-

трансфераза.

21. У немовляти

спостерігаються епіле-

птиформні судоми,

викликані дефіцитом

вітаміну B6. Це спричинено

зменшенням у нервовій

тканині гальмівного

медіатора - 7-

аміномасляної кислоти.

Активність якого

ферменту знижена при

цьому ?

A. Глутаматсинтетаза.

B. Аланінамінотрансфераза.

C. Глутаматдегідрогеназа.

D. Піридоксалькіназа.

E. *Глутамат-

декарбоксилаза.

22. У пацієнта, що

звернувся до лікаря,

спостерігається жовте

забарвлення шкіри, сеча -

темна, кал темно-жовтого

кольору. Підвищення

концентрації якої

речовини буде спостеріга-

тися в сироватці крові ?

A. Білівердин.

B. Кон’югований білірубін.

C. Мезобілірубін.

D. Вердоглобін.

E. *Вільний білірубін.

23. В результат виснажую-

чої м’язової роботи у робо-

чого значно зменшилась

буферна ємність крові.

Надходженням якої

речовини у кров можна

пояснити це явище ?

A. 3-Фосфоглицерат.

B. Піруват.

C. 1,3-Бісфосфогліцерат.

D. Альфа-кетоглутарат.

E. *Лактат.

24. Хворий з діагнозом

цукровий діабет, вранці

натще отримав

Page 201: КРОК-1 БІОХІМІЯ Збірник тестових завданьlib.sumdu.edu.ua/library/docs/rio/2020/m4695.pdf · 2 Крок-1. Біохімія Н: збірник тестових

201

призначену дозу інсуліну

пролонгованої дії.

Пропустив черговий

прийом їжі і невдовзі

відчув слабкість, біль

голови, запаморочення,

пітливість, тремтіння тіла,

судоми, відчуття голоду,

явища гіпоглікемії.

Застосування глюкози

стан не полегшило. Який

препарат необхідно ввести

для купірування даного

стану ?

A. Гідрокортизон.

B. Тріамцінолон.

C. Норадреналін.

D. Пренізолон.

E. *Адреналін.

25. У чоловіка 25-ти років

з переломом основи черепа

виділяється великий об’єм

сечі з низькою відносною

щільністю. Причиною змін

сечоутворення є

порушення синтезу такого

гормону:

A. Соматотропний гормон.

B. Тиреотропний гормон.

C. Адренокортикотропний

гормон.

D. Окситоцин.

E. *Вазопресин.

26. У дитини, що страждає

на пілоростеноз, що

супроводжується частим

блюванням, розвинулись

ознаки зневоднення

організму. Яка форма

порушення кислотно-

основного стану може

розвиватися у даному

випадку ?

A. Метаболічний ацидоз.

B. Негазовий ацидоз.

C. Газовий ацидоз.

D. Газовий алкалоз.

E. *Негазовий алкалоз.

27. У 12-річного хлопчика

в сечі виявлено високий

вміст усіх амінокислот

аліфатичного ряду. При

цьому відмічена найбільш

висока екскреція цистину

та цистеїну. Крім того,

УЗД нирок показало

наявність каменів у них.

Виберіть можливу

патологію:

A. Хвороба Хартнупа.

B. Алкаптонурія.

C. Цистит.

D. Фенілкетонурія.

E. *Цистинурія.

28. Цикл Кребса відіграє

важливу роль у реалізації

Page 202: КРОК-1 БІОХІМІЯ Збірник тестових завданьlib.sumdu.edu.ua/library/docs/rio/2020/m4695.pdf · 2 Крок-1. Біохімія Н: збірник тестових

202

глюкопластичного ефекту

амінокислот. Це зумовлено

обов’язковим перетво-

ренням безазотистого

залишку амінокислот у:

A. Цитрат.

B. Малат.

C. Сукцинат.

D. Фумарат.

E. *Оксалоацетат.

29. У пацієнта 65-ти років з

тривалими скаргами,

характерними для

хронічного гастриту, у

периферичній крові

виявлені мегалоцити, у

кістковому мозку мегало-

бластичний еритропоез.

Який найбільш імовірний

діагноз ?

A. Залізодефіцитна анемія.

B. Апластична анемія.

C. Гіпопластична анемія.

D. Гемолітична анемія.

E. *B12-фолієводефіцитна

анемія.

30. У хворого з

жовтяницею встановлено:

підвищення у плазмі крові

вмісту загального

білірубіну за рахунок

непрямого (вільного), в

калі та сечі - високий вміст

стеркобіліну, рівень

прямого (зв’язаного)

білірубіну в плазмі крові в

межах норми. Про який

вид жовтяниці можна

думати ?

A. Хвороба Жильбера.

B. Паренхіматозна

(печінкова).

C. Механічна.

D. Жовтяниця немовлят.

E. *Гемолітична.

31. Кухар в результаті

необачності обпік руку

парою. Підвищення

концентрації якої

речовини викликало

почервоніння, набряклість

та болючість ураженої

ділянки шкіри ?

A. Галактозамін.

B. Тіамін.

C. Глутамін

D. Лізин.

E. *Гістамін.

32. У хворого із

запаленням легень

спостерігається підвищен-

ня температури тіла. Яка

біологічно активна

речовина відіграє провідну

роль у виникненні цього

прояву хвороби ?

Page 203: КРОК-1 БІОХІМІЯ Збірник тестових завданьlib.sumdu.edu.ua/library/docs/rio/2020/m4695.pdf · 2 Крок-1. Біохімія Н: збірник тестових

203

A. Інтерлейкін-I.

B. Гістамін.

C. Брадикінін.

D. Серотонін.

E. *Лейкотрієни.

33. Хворий 12-ти років

поступив в клініку з

гемартрозом колінного

суглоба, з раннього дитин-

ства страждає кровоточи-

вістю. Яка хвороба у

хлопчика ?

A. Тромбоцитопенічна

пурпура.

B. Геморагічний васкуліт.

C. Гемолітична анемія.

D. B12 фолієво-дефіцитна

анемія.

E. *Гемофілія.

34. У хворого видалено 12-

палу кишку. Це призведе

до зменшення секреції,

перш за все, такого

гормону:

A. Нейротензин.

B. Гастрин.

C. Гістамін.

D. Соматостатин.

E. *Холецистокінін-

секретин.

35. Вітамін A у комплексі

зі специфічними цито-

рецепторами проникає

через ядерні мембрани,

індукує процеси

транскрипції, що

стимулює ріст та

диференціювання клітин.

Ця біологічна функція

реалізується наступною

формою вітаміну A:

A. Каротин.

B. Транс –ретиналь.

C. Цис-ретиналь.

D. Ретинол.

E. *Транс-ретиноєва

кислота.

36. Одна з форм вродженої

патології супроводжується

гальмуванням перетво-

рення фенілаланіну в

тирозин. Біохімічною

ознакою хвороби є

накопичення в організмі

деяких органічних кислот,

у тому числі такої

кислоти:

A. Глутамінова.

B. Лимонна.

C. Піровиноградна.

D. Молочна.

E. *Фенілпіровиноградна.

37. Встановлено ураження

ВІЛ Т-лімфоцитів. При

цьому фермент вірусу

Page 204: КРОК-1 БІОХІМІЯ Збірник тестових завданьlib.sumdu.edu.ua/library/docs/rio/2020/m4695.pdf · 2 Крок-1. Біохімія Н: збірник тестових

204

зворотня транскриптаза

(РНК-залежна ДНК-

полімераза) каталізує

синтез:

A. і-РНК на матриці

вірусного білку.

B. Вірусної і-РНК на

матриці ДНК.

C. ДНК на вірусній р-РНК.

D. Вірусної РНК на матриці

ДНК.

E. *ДНК на матриці вірусної

і-РНК.

38. У хворого спосте-

рігається гемералопія

(куряча сліпота). Яка з

перерахованих речовин

володітиме лікувальною

дією ?

A. Карнозин.

B. Кератин

C. Креатин

D. Карнітин

E. *Каротин.

39. Внаслідок тривалого

голодування в організмі

людини швидко зникають

резерви вуглеводів. Який з

процесів метаболізму за

цих умов поновлює вміст

глюкози в крові ?

A. Пентозофосфатний шлях.

B. Анаеробний гліколіз.

C. Аеробний гліколіз.

D. Ілікогеноліз.

E. *Глюконеогенез.

40. У чоловіка 40-ка років

було встановлено діагноз:

серпоподібноклітинна

анемія. Який механізм

приводить до зменшення

кількості еритроцитів в

крові у цього хворого ?

A. Нестача білка.

B. Внутрішньосудинний

гемоліз.

C. Нестача заліза в організмі

D. Нестача вітаміну B12 і

фолієвої кислоти.

E. *Позасудинний гемоліз.

41. В поліклініку до лікаря

звернулася жінка 32-х

років зі скаргами на

відсутність в неї лактації

після народження дитини.

Дефіцитом якого гормону,

найбільш імовірно, можна

пояснити дане порушення?

A. Глюкагон.

B. Соматотропін.

C. Вазопресин.

D. Тиреокальцитонін.

E. *Пролактин.

42. Жінка 52-х років, хвора

на рак молочної залози,

Page 205: КРОК-1 БІОХІМІЯ Збірник тестових завданьlib.sumdu.edu.ua/library/docs/rio/2020/m4695.pdf · 2 Крок-1. Біохімія Н: збірник тестових

205

пройшла курс променевої

терапії. Розмір пухлини

зменшився. Який з

наведених механізмів

ушкодження клітини

найбільше обумовлює

ефективність променевої

терапії ?

A. Мутагенез.

B. Гіпертермія.

C. Лізис NK-клітинами.

D. Тромбоз судин.

E. *Утворення вільних

радикалів.

43. Студент використав

консервовану донорську

кров для визначення часу

її зсідання. Однак, будь-

якого позитивного

результату він отримати

не зміг. Причиною цього є

відсутність в крові:

A. Вітаміну K.

B. Фактора Хагемана.

C. Тромбопластину.

D. Фі6риногєну.

E. *Іонізованого кальцію.

44. Введення тварині

екстракту тканини

передсердя посилює

виділення натрію з сечею.

Дія якої біологічно

активної речовини стала

причиною такого стану ?

A. Калійкреїн.

B. Глюкокортикоїд.

C. Адреналін.

D. Серотонін.

E. *Натрійуретичний

гормон.

45. У альпініста, що

піднявся на висоту 5200 м,

розвинувся газовий

алкалоз. Що є причиною

його розвитку?

A. Зниження температури

навколишнього середовища.

B. Гіповентиляція легенів.

C. Гіпероксемія.

D. Гіпоксемія.

E. *Гіпервентиляція легенів

46. Прокаріотичні та

еукаріотичні клітини

характеризуються здатніс-

тю до поділу. Поділ

прокаріотичних клітин

відрізняється від поділу

еукаріотичних, але існує

молекулярний процес,

який лежить в основі цих

поділів. Який це процес ?

A. Ампліфікація генів.

B. Транскрипція.

C. Репарація.

D. Трансляція.

Page 206: КРОК-1 БІОХІМІЯ Збірник тестових завданьlib.sumdu.edu.ua/library/docs/rio/2020/m4695.pdf · 2 Крок-1. Біохімія Н: збірник тестових

206

E. *Реплікація ДНК.

47. У хворого з синдромом

Іценко-Кушинга спостері-

гаються стійка гіпер-

глікемія та глюкозурія.

Синтез та секреція якого

гормону збільшені у цього

хворого ?

A. Альдостерон.

B. Адреналін.

C. Глюкагон.

D. Тироксин.

E. *Кортизол.

48. У хворого з

гемолітичною анемією

виявлено дефіцит

піруваткінази в

еритроцитах. За цих умов

причиною розвитку

гемолізу еритроцитів є: A. Дефіцит спектрину.

B. Нестача Na+ в

еритроцитах.

C. Надлишок К+ в

еритроцитах.

D. Генетичні дефекти

глікофорину А.

E. *Зменшення активності

Na+, К+ -АТФ-ази

2.12 КРОК-2018

1. До травматологiчного

пункту доставлено

постраждалого пiсля ДТП

з дiагнозом: Закритий

перелом середньої третини

стегна зi змiщенням. З

метою репозицiї кiсткових

уламкiв хворому введено

10 мл 2% розчину

дитилiну в/в, внаслiдок

чого розвинулося тривале

апное та мiорелаксацiя.

Дефiцитом якого ферменту

зумовлена вказана

фармакогенетична

ферментопатiя ?

A. Уридіндифосфоглюкуро-

нілтрансфераза.

B. N-ацетилтрансфераза.

C. Глюкозо-6-фосфат-

дегiдрогеназа

D. Метгемоглобiнредуктаза.

E. *Псевдохолiнестераза.

2. Захворювання берi-берi-

це класична форма

недостатностi вiтамiну

тiамiну. Активна форма

його синтезується за

допомогою ферменту з

класу:

A. Iзомераз.

B. Оксидоредуктаз.

Page 207: КРОК-1 БІОХІМІЯ Збірник тестових завданьlib.sumdu.edu.ua/library/docs/rio/2020/m4695.pdf · 2 Крок-1. Біохімія Н: збірник тестових

207

C. Гiдролаз.

D. Лiаз.

E. *Трансфераз.

3. У хворого iнфаркт

мiокарда. Активнiсть

якого ферменту буде

значно пiдвищена в

сироватцi кровi хворого в

першi години ?

A. АЛТ.

B. ЛДГ4.

C. AСT.

D. ЛДГ5.

E.*Креатинфосфокiназа МВ.

4. У хворого cпостерi-

гається атонiя м’язiв.

Назвiть фермент м’язової

тканини, активнiсть якого

може бути знижена при

такому станi:

A. Каталаза.

B. Амiлаза.

C. Транскетолаза.

D. Глутамiнтрансфераза.

E. *Креатинфосфокiназа.

5. Пiд час патологiчних

процесiв, якi супровод-

жуються гiпоксiєю,

вiдбувається неповне

вiдновлення молекули

кисню в дихальному

ланцюзi i накопичення

пероксиду водню. Вкажiть

фермент, який забезпечує

його руйнування:

A. Аконiтаза.

B. Цитохромоксидаза.

C. Сукцинатдегiдрогеназа.

D. Кетоглутарат-дегiдро-

геназа.

E. *Каталаза.

6. У вагiтної жiнки

розвинувся токсикоз з

тяжким повторним

блюванням шлунковим

вмiстом протягом доби.

Наприкiнцi доби почали

проявлятися тетанiчнi

судоми та зневоднення

органiзму. Який розлад

кислотно-лужної рiвно-

ваги викликав данi змiни ?

A. Негазовий видiльний

ацидоз.

B. Газовий алкалоз.

C. Газовий ацидоз.

D. Негазовий метаболiчний

ацидоз.

E. *Негазовий видiльний

алкалоз.

7. У лiкарню надiйшла

робiтниця хiмiчного

пiдприємства з ознаками

отруєння. У волоссi цiєї

жiнки знайдено пiдвищену

Page 208: КРОК-1 БІОХІМІЯ Збірник тестових завданьlib.sumdu.edu.ua/library/docs/rio/2020/m4695.pdf · 2 Крок-1. Біохімія Н: збірник тестових

208

концентрацiю арсенату,

який блокує лiпоєву

кислоту. Вкажiть,

порушення якого процесу

є iмовiрною причиною

отруєння:

A. Знешкодження

супероксидних iонiв.

B. Мiкросомальне

окиснення.

C. Вiдновлення

метгемоглобiну.

D. Вiдновлення органiчних

перекисiв.

E. *Окислювальне

декарбоксилювання ПВК.

8. З метою аналгезiї

можуть бути використанi

речовини, що iмiтують

ефекти морфiну, але

виробляються в ЦНС.

Вкажiть таку речовину:

A. Соматолiберин.

B. Окситоцин.

C. Вазопресин.

D. Кальцитонiн.

E. *β-ендорфiн.

9. У хворого через 12 годин

пiсля гострого нападу

загрудинного болю

спостерiгається рiзке

пiдвищення активностi

АсАТ в сироватцi кровi.

Вкажiть патологiю, для

якої це характерно:

A. Нецукровий дiабет.

B. Вiруснийгепатит.

C. Колагеноз.

D. Цукровий дiабет.

E. *Iнфаркт мiокарду.

10. Пiсля курсу терапiї

хворому на виразку

дванадцятипалої кишки

лiкар пропонує вживати

соки з капусти та картоплi.

Вмiст яких речовин в цих

продуктах сприяє

профiлактицi та загоєнню

виразок ?

A. Вiтамiн K.

B. Пантотенова кислота.

C. Вiтамiн C.

D. Вiтамiн B1.

E. *Вiтамiн U.

11. Хворому з больовим

синдромом в суглобах

постiйно призначають

аспiрин. Який з

перерахованих ферментiв

вiн пригнiчує ?

A. Фосфолiпаза С.

B. Лiпооксигеназа.

C. Фосфолiпаза А2.

D. Фосфолiпаза D.

E. *Циклооксигеназа.

Page 209: КРОК-1 БІОХІМІЯ Збірник тестових завданьlib.sumdu.edu.ua/library/docs/rio/2020/m4695.pdf · 2 Крок-1. Біохімія Н: збірник тестових

209

12. У дитини 6-ти мiсяцiв

спостерiгається рiзке

вiдставання в психомотор-

ному розвитку, блiда

шкiра з екзематозними

змiнами, бiляве волосся,

блакитнi очi, напади

судом. Найточнiше

встановити дiагнозу цiєї

дитини дозволить

визначення в кровi i сечi

концентрацiї такої

речовини:

A. Валiн.

B. Триптофан.

C. Гiстидин.

D. Лейцин.

E. *Фенiлпiруват.

13. Аналiз кровi хворого на

цукровий дiабет показав

наявнiсть молочної

кислоти у концентрацiї 2,5

ммоль/л. Яка кома

розвинулася у хворого ?

A. Гiперкетонемiчна.

B. Гiперглiкемiчна.

C. Гiпоглiкемiчна.

D. Гiперосмолярна.

E. *Лактацидемiчна.

14. Щоденно в органiзмi

людини 0,5% всього

гемоглобiну перетво-

рюється на метгемоглобiн.

Який фермент, що

мiститься в еритроцитах,

каталiзує вiдновлення

метгемоглобiну до

гемоглобiну ?

A. Бiлiвердинредуктаза.

B. Метгемоглобiн-

трансфераза.

C. Глюкуронiлтрансфераза.

D. Гемоксигеназа.

E. *Метгемоглобiн-

редуктаза.

15. У хворого виявлена

гемералопiя (куряча

слiпота). Яка з

перерахованих речовин

матиме лiкувальну дiю ?

A. Карнозин.

B. Кератин.

C. Креатин.

D. Карнiтин.

E. *Каротин.

16. У юнака 20-ти рокiв

дiагностовано спадковий

дефiцит УДФ-глюкуронiл-

трансферази. Пiдвищення

якого показника кровi

пiдтверджує цей дiагноз ?

A. Тваринний iндикан.

B. Прямий (кон’югований)

бiлiрубiн.

C. Уробiлiн.

D. Стеркобiлiноген.

Page 210: КРОК-1 БІОХІМІЯ Збірник тестових завданьlib.sumdu.edu.ua/library/docs/rio/2020/m4695.pdf · 2 Крок-1. Біохімія Н: збірник тестових

210

E. *Непрямий

(некон’югований)бiлiрубiн.

17. Чоловiк 55-ти рокiв, що

скаржиться на бiль в

дiлянцi нирок, надiйшов в

лiкарню. В ходi ультра-

звукового обстеження

пацiєнта виявлено

наявнiсть ниркових

каменiв. Наявнiсть в сечi

якої з наведених речовин є

найбiльш iмовiрною

причиною утворення

каменiв у даного пацiєнта?

A. Креатинiн.

B. Бiлiрубiн.

C. Бiлiвердин.

D. Уробiлiн.

E. *Сечова кислота.

18. В ходi визначення

енерговитрат органiзму

людини встановлено, що

дихальний коефiцiєнт

дорiвнює 1,0. Це означає,

що у клiтинах

дослiджуваного переважно

окиснюються:

A. Вуглеводи та жири.

B. Бiлки.

C.Жири.

D. Бiлки i вуглеводи.

E. *Вуглеводи.

19. Важливим джерелом

утворення амiаку в

головному мозку є

дезамiнування АМФ. Яка

амiнокислота вiдiграє

основну роль у зв’язуваннi

амiаку в нервовiй тканинi?

A. Аланiн.

B. Лiзин.

C. Iзолейцин.

D. Аргiнiн.

E. *Глутамат.

20. У 19-мiсячної дитини iз

затримкою розвитку та

проявами самоагресiї,

вмiст сечової кислоти в

кровi - 1,96 ммоль/л. При

якому метаболiчному

порушеннi це спостерi-

гається ?

A. Хвороба Iценко-Кушинга.

B. Подагра.

C. Синдром набутого

iмунодефiциту.

D. Хвороба Гiрке.

E. *Синдром Леша-Нiхана.

21. Дитина 9-ти мiсяцiв

харчується сумiшами,

незбалансованими за

вмiстом вiтамiну B6. У

дитини спостерiгається

пелагроподiбний дерматит,

судоми, анемiя. Розвиток

Page 211: КРОК-1 БІОХІМІЯ Збірник тестових завданьlib.sumdu.edu.ua/library/docs/rio/2020/m4695.pdf · 2 Крок-1. Біохімія Н: збірник тестових

211

судом може бути

пов’язаний з дефiцитом

утворення: A. Дофамiну.

B. Гiстамiну.

C. Серотонiну.

D. ДОФА.

E. *ГАМК.

22. Хворому на гепатит

для попередження уражень

печiнки призначили

вiтамiноподiбну речовину

холiн. Його лiкувальний

ефект пов’язаний з:

A. Пригнiченням синтезу

ацетонових тiл.

B. Активацiєю глiкоген-

синтази.

C. Активацiєю глiкоген-

фосфорилази.

D. Пригнiченням синтезу

холестерину.

E. *Лiпотропною дiєю.

23. Дослiдженнями остан-

нiх десятилiть встанов-

лено, що безпосереднiми

”виконавцями” апоптозу в

клiтинi є особливi

ферменти - каспази. В

утвореннi одного з них

бере участь цитохром С.

Вкажiть його функцiю в

нормальнiй клiтинi:

A. Компонент пiруват-

дегiдрогеназної системи.

B. Фермент ЦТК.

C. Фермент β-окиснення

жирних кислот.

D. Компонент H+-АТФ-азної

системи.

E. *Фермент дихального

ланцюга переносу

електронiв.

24. Процес бiосинтезу бiлка

є енергозалежним.

Вкажiть, який

макроергiчний субстрат

безпосередньо використо-

вується в цьому процесi на

стадiї елонгацiї:

A. ЦТФ.

B. АТФ.

C. АДФ.

D. УТФ.

E. *ГТФ.

25. У новонародженої

дитини спостерiгається

зниження iнтенсивностi

смоктання, часте

блювання, гiпотонiя. У

сечi та кровi значно

пiдвищена концентрацiя

цитрулiну. Який

метаболiчний процес

порушений ? A. Цикл Корi.

Page 212: КРОК-1 БІОХІМІЯ Збірник тестових завданьlib.sumdu.edu.ua/library/docs/rio/2020/m4695.pdf · 2 Крок-1. Біохімія Н: збірник тестових

212

B. ЦТК.

C. Глiколiз.

D. Глюконеогенез.

E. *Орнiтиновий цикл.

26. Пiсля споживання

солоної їжi у людини

значно зменшилася

кiлькiсть сечi. Пiдвищена

секрецiя якого гормону

призвела до зменшення

дiурезу ?

A. Ренiн.

B. Натрiйуретичний.

C. Ангiотензин-II.

D. Альдостерон.

E. *Вазопресин.

27. Чоловiк 70-ти рокiв

хворiє на атеросклероз

судин нижнiх кiнцiвок та

iшемiчну хворобу серця.

Пiд час обстеження

виявлено порушення

лiпiдного складу кровi.

Надлишок яких

лiпопротеїнiв є головною

ланкою в патогенезi

атеросклерозу ?

A. Хiломiкронiв.

B. Холестерину.

C. Високої щiльностi.

D. Промiжної щiльностi.

E. *Низької щiльностi.

28. Пiсля видалення зуба у

пацiєнта виникла

кровотеча. Аналiз кровi

виявив зниження

протромбiнового iндексу.

Дефiцит якого вiтамiну

може бути причиною

такого стану ?

A. A.

B. D.

C. C.

D. B.

E. *K.

29. Аспiрин iнгiбує синтез

простагландинiв, завдяки

блокуванню активностi

циклооксигенази. Яка

жирна кислота необхiдна

для цього синтезу?

A. Пальмiтинова.

B. Лiнолева.

C. Лiноленова.

D. Стеаринова.

E. *Арахiдонова.

30. У хворого з яскраво

вираженою жовтушнiстю

шкiри, склер та слизових

оболонок, сеча має колiр

темного пива, кал свiтлий.

У кровi пiдвищений вмiст

прямого бiлiрубiну, в сечi

визначається бiлiрубiн.

Page 213: КРОК-1 БІОХІМІЯ Збірник тестових завданьlib.sumdu.edu.ua/library/docs/rio/2020/m4695.pdf · 2 Крок-1. Біохімія Н: збірник тестових

213

Який тип жовтяницi у

хворого ?

A. Екскрецiйна.

B. Паренхiматозна.

C. Гемолiтична.

D. Кон’югацiйна.

E. *Обтурацiйна.

31. У вiддiлення надiйшла

дитина з носовою

кровотечею та меленою

(кровю) в калi. Зi слiв

матерi, вiдбулося отруєння

дитини кумаринами, якi

застосовувалися для

боротьби з щурами.

Введення якого засобу

припинить кровотечу у

дитини ?

A. Адреналiн.

B. Фраксипарин.

C. Фепранон.

D. Тромбiн.

E. *Вiкасол.

32. Внаслiдок дефiциту

УФО-ендонуклеази

порушується репарацiя

ДНК i виникає таке

захворювання:

A. Серпоподiбноклiтинна

анемiя.

B. Подагра.

C. Фенiлкетонурiя.

D. Альбiнiзм.

E. *Пiгментна ксеродермiя.

33. Пiд час реплiкацiї ДНК

один iз її ланцюгiв

синтезується iз запiзнен-

ням. Що визначає дану

особливiсть синтезу ?

A. Необхiднiсть репарацiї.

B. Комплiментарнiсть

ланцюгiв.

C. Вiдсутнiсть

трифосфонуклеотидiв.

D. Великi розмiрамиДНК-

полiмерази.

E. *Антипаралельнiсть

ланцюгiв.

34. Фермент оксидаза

D-амiнокислот каталiзує

дезамiнування тiльки

D-амiнокислот. Яка

властивiсть ферментiв

виявляється при цьому ?

A. Абсолютна

специфiчнiсть.

B. Термолабiльнiсть.

C. Вiдносна специфiчнiсть.

D. Залежнiсть вiд рН.

E. *Стереохiмiчна

специфiчнiсть.

35. An experiment has

demonstrated that after

exposure to ultraviolet

radiation the dermal cells of

Page 214: КРОК-1 БІОХІМІЯ Збірник тестових завданьlib.sumdu.edu.ua/library/docs/rio/2020/m4695.pdf · 2 Крок-1. Біохімія Н: збірник тестових

214

the patients with xeroderma

Pigmentosum are slower to

restore the native DNA

structure than they are in

the healthy individuals due

to deficiency of the DNA

repair enzyme. What

enzyme takes part in the

repair process ?

A. DNA gyrase.

B. RNA ligase.

C. Primase.

D. DNA polymerase III

holoenzyme.

E. *Endonuclease.

36. Blood serum analysis of

the patient with acute

hepatitis show sincreased

levels of alanine

aminotransferase (ALT) and

aspartate aminotransferase

(AST). What changes on the

cellular level can resultin

such developments ?

A. Disturbed intercellular

interactions.

B. Disturbed energy supply to

the cells.

C. Disturbed cellular enzyme

systems.

D. Damage to the genetic

apparatus of the cells.

E. *Cell destruction.

37. In the morning a man

diagnosed with diabetes

mellitus received a

prescribed dose of long-

acting insulin on an empty

stomach. He missed his

regular meal and soon after

that he developed weakness,

headache, and vertigo, body

tremors, convulsions, feeling

of hunger, and signs of

hypoglycemia. Glucose

administration did not

improve the patient’s

condition. What medicine

should be administered to

provide quick relief to the

patient ?

A. Hydrocortisone

B. Triamcinolone

C. Noradrenaline

D. Prednisolone

E. *Adrenaline.

38. A 36-year-old woman

suffers from a connective

tissue disease (collagenosis).

What metabolite is the most

likely to be increased in her

urine ?

A. Urobilinogen.

B. Indican.

C. Creatinine.

D. Urea.

E. *Oxyproline.

Page 215: КРОК-1 БІОХІМІЯ Збірник тестових завданьlib.sumdu.edu.ua/library/docs/rio/2020/m4695.pdf · 2 Крок-1. Біохімія Н: збірник тестових

215

39. The cell was exposed to

mutagenic factor which

resulted in DNA molecule

losing 2 nucleotide pairs.

What type of mutation

occurred in the DNA ?

A. Replication.

B. Duplication.

C. Inversion.

D. Translocation.

E. *Deletion.

40. An unconscious patient

was brought into the

hospital. The smell of

acetone can be detected from

the patient’s mouth. Blood

glucose – 25 mmol/L, ketone

bodies – 0,57 mmol/L. What

hormone deficiency can

result in the development of

this condition ?

A. Somatotropin

B. Thyroxin.

C. Glucocorticoids.

D. Aldosterone.

E. *Insulin.

41. A patient has developed

systemic (megaloblastic)

anemia despite eating a

balanced diet. The day

before he underwent a

gastric surgical resection.

The anemia in this patient is

caused by the deficiency of:

A. Folic acid.

B. Vitamin B1.

C. Vitamin B6.

D. Protein.

E. *Castle factor.

42. A patient was

hospitalized with diagnosis

of an intestinal carcinoid.

Laboratory analysis detects

increased synthesis of

serotonin from tryptophan.

This process is based on the

following biological

mechanism:

A. Formation of paired

compounds.

B. Deaminization.

C. Microsomaloxidation.

D. Transamination.

E. *Decarboxylation.

2.13 КРОК-2019

1. У головному мозку

людини утворюються

ендогенні пептиди, подібні

до морфію, які здатні

зменшувати больові

відчуття. До них

відносяться:

A. Окситоцин.

Page 216: КРОК-1 БІОХІМІЯ Збірник тестових завданьlib.sumdu.edu.ua/library/docs/rio/2020/m4695.pdf · 2 Крок-1. Біохімія Н: збірник тестових

216

B. Ліберини.

C. Статини.

D. Вазопресин.

E. *Ендорфіни.

2. У хлопчика 7-ми

років діагностована

анемія. Лабораторно

встановлено дефіцит

піруваткінази в еритро-

цитах. Порушення якого

процесу відіграє головну

роль в розвитку анемії у

хлопчика ?

A. Дезамінування

амінокислот.

B. Декарбоксилювання

амінокислот.

C. Анаеробний глікогеноліз.

D. Глюконеогенез.

E. *Анаеробний гліколіз.

3. У новонародженої

дитини спостерігаються

зниження інтенсивності

смоктання, часте

блювання, гіпотонія. У

сечі та крові значно

підвищена концентрація

цитруліну. Який

метаболічний процес

порушений ?

A. Цикл Корі.

B. ЦТК.

C. Гліколіз.

D. Глюконеогенез.

E. *Орнітиновий цикл.

4. У хворого 27-ми років

виявлено патологічні

зміни печінки і головного

мозку. У плазмі крові

виявлено різке зниження, а

в сечі підвищення вмісту

міді. Поставлено діагноз:

хвороба Вільсона.

Активність якого фермен-

ту в сироватці крові

необхідно дослідити для

підтвердження діагнозу ?

A. Лейцинамінопептидаза.

B. Карбоангідраза.

C. Ксантиноксидаза.

D. Алкогольдегідрогеназа.

E. *Церулоплазмін.

5. Пацієнтам з ішемічною

хворобою серця

призначають невеликі

дози аспірину, який інгібує

синтез активатора

агрегації тромбоцитів

тромбоксану А2. З якої

речовини утворюється

тромбоксан А2 ?

A. Оцтова кислота.

B. Глутамінова кислота.

C. Малонова кислота.

D. Гомогентизинова

кислота.

Page 217: КРОК-1 БІОХІМІЯ Збірник тестових завданьlib.sumdu.edu.ua/library/docs/rio/2020/m4695.pdf · 2 Крок-1. Біохімія Н: збірник тестових

217

E. *Арахідонова кислота.

6. При хворобі Верніке-

Корсакова, що часто

спостерігається у

хронічних алкоголіків,

харчовий раціон яких

містить мало вітамінів,

визначається зниження

активності транскетолази.

Нестача якого вітаміну

призводить до цього ?

A. Ніацин.

B. Ретинол.

C. Рибофлавін.

D. Кобаламін.

E. *Тіамін.

7. При цукровому діабеті і

голодуванні в крові

збільшується вміст

ацетонових тіл, що

використовуються в якості

енергетичного матеріалу.

Назвіть речовину, з якої

вони синтезуються:

A. Кетоглутарат.

B. Малат.

C. Сукциніл-КоА.

D. Цитрат.

E. *Ацетил-КоА.

8. У пацієнта, який довгий

час знаходився на

незбалансованому харчу-

ванні з недостатньою

кількістю білка,

розвинулась жирова

інфільтрація печінки.

Назвіть речовину,

відсутність якої у їжі могла

бути причиною цього

стану:

A. Біотин.

B. Холестерин.

C. Аланін.

D. Оцтова кислота.

E. *Метіонін.

9. У пацієнта тривале

блювання призвело до

зневоднення організму.

Підвищення секреції якого

гормону за цих умов, перш

за все, забезпечує

збереження води в

організмі ?

A. Натрійуретичний.

B. Кальцитонін.

C. Альдостерон.

D. Адреналін.

E. *Вазопресин.

10. При хронічному

передозуванні глюко-

кортикоїдів у хворого

розвивається гіпер-

глікемія. Назвіть процес

вуглеводного обміну, за

рахунок якого при цьому

Page 218: КРОК-1 БІОХІМІЯ Збірник тестових завданьlib.sumdu.edu.ua/library/docs/rio/2020/m4695.pdf · 2 Крок-1. Біохімія Н: збірник тестових

218

збільшується концентра-

ція глюкози у крові:

A. Пентозофосфатний цикл.

B. Глікогенез.

C. Аеробний гліколіз.

D. Глікогеноліз.

E. *Глюконеогенез.

11. У щура з хронічною

нирковою недостатністю

виявлено остеопороз,

патологічна кальцифіка-

ція внутрішніх органів,

артеріальна гіпертензія. З

посиленням дії якого

гормону повязані ці

порушення ?

A. Кальцитонін.

B. Трийодтиронін.

C. Адреналін.

D. Тироксин.

E. *Паратгормон.

12. В гематологічному

відділенні хворому на

лейкоз лікар призначив

5-фторурацил, який:

A. Прискорює реплікацію.

B. Стимулює ДНК-азу.

C. Інгібує трансляцію.

D. Інгібує транскрипцію.

E. *Інгібує синтез ДНК.

13. У 3-річної дитини з

підвищеною температурою

тіла після прийому

аспірину спостерігається

гемоліз еритроцитів.

Вроджена недостатність

якого ферменту могла

викликати у дитини

гемолітичну анемію ?

A. Гамма-глутаміл-

трансфераза.

B. Глікогефосфорилаза.

C. Гліцеролфосфат-дегідро-

геназа.

D. Глюкозо-6-фосфатаза.

E. *Глюкозо-6-фосфат-

дегідрогеназа.

14. При обстеженні хворого

виявлено зменшення

кількості лейкоцитів,

еритроцитів і гемоглобіну

в периферичній крові,

появу великих клітин

(мегалобластів). Дефіцит

якого вітаміну може до

цього призводити ?

A. Біотин.

B. Ніацин.

C. Рибофлавін.

D. Аскорбінова кислота.

E. *Фолієва кислота.

15. У жінки 45-ти років

відсутні симптоми діабету,

але натще визначається

підвищений вміст глюкози

Page 219: КРОК-1 БІОХІМІЯ Збірник тестових завданьlib.sumdu.edu.ua/library/docs/rio/2020/m4695.pdf · 2 Крок-1. Біохімія Н: збірник тестових

219

в крові (7,2 ммоль,л).

Наступним тестом має

бути визначення:

A. Сечовини в крові.

B. Залишкового азоту в

крові.

C. Глікозильованого

гемоглобіну.

D. Глюкози у сечі.

E. *Толерантності до

глюкози.

16. У хворого виявлено

зниження кількості іонів

магнію, які потрібні для

прикріплення рибосом до

гранулярної ендоплазма-

тичної сітки. Відомо, що це

призводить до порушення

біосинтезу білка.

Порушення відбувається

на такому етапі:

A. Реплікація.

B. Процесинг.

C. Транскрипція.

D. Активація амінокислот.

E. *Трансляція.

17. Вагітній жінці, яка

мала в анмнезі декілька

викиднів, призначена

терапія, яка містить

вітамінні препарати.

Вкажіть вітамін, який

сприяє виношуванню

вагітності:

A. Піридоксальфосфат.

B. Фолієва кислота.

C. Рутин.

D. Ціанокобаламін.

E. *Альфа-токоферол.

18. Хворого доставили до

клініки у коматозному

стані. В анамнезі:

цукровий діабет II типу

впродовж 5-ти років.

Об’єктивно: дихання

шумне, глибоке, у

видихуваному повітрі

чутно запах ацетону. Вміст

глюкози у крові

15,2 ммоль/л, кетонових

тіл – 100 мкмоль/л. Для

якого ускладнення даного

захворювання характерні

такі розлади ?

A. Печінкова кома.

B. Гіпоглікемічна кома.

C. Гіперглікемічна кома.

D. Гіперосмолярна кома.

E. *Кетоацидотична кома.

19. У жінки 45-ти років має

місце недостатня секреція

ферменту ентерокінази.

Порушення якої травної

функції може викликати

дефіцит ентерокінази ?

Page 220: КРОК-1 БІОХІМІЯ Збірник тестових завданьlib.sumdu.edu.ua/library/docs/rio/2020/m4695.pdf · 2 Крок-1. Біохімія Н: збірник тестових

220

A. Гідроліз вуглеводів.

B. Всмоктування вітамінів.

C. Гідроліз жирів.

D. Всмоктування жирів.

E. *Гідроліз білків.

20. У хворих на колагеноз

має місце процес

деструкції сполучної

тканини. Зростання вмісту

яких сполук в крові це

підтверджує ?

A. Урати.

B. Креатин та креатинін.

C. Трансамінази.

D. Ізоферменти ЛДГ.

E. *Оксипролін та

оксилізин.

21. У хворого на цукровий

діабет після ін’єкції

інсуліну настала втрата

свідомості, судоми. Який

результат дав біохімічний

аналіз крові на вміст

глюкози ?

A. 10 ммоль/л.

B. 8,0 ммоль/л.

C. 5,5 ммоль/л.

D. 3,3 ммоль/л.

E. *2,5 ммоль/л.

22. У хворого виявлена

стеаторея. З порушенням

надходження у кишечник

яких речовин це може бути

пов’язане ?

A. Вуглеводи.

B. Амілаза.

C. Хімотрипсин.

D. Трипсин.

E. *Жовчні кислоти.

23. 40-річному чоловікові,

який хворіє на

туберкульоз легень,

призначено ізоніазид.

Нестача якого вітаміну

може розвинутися

внаслідок тривалого

вживання даного препа-

рату ?

A. Фолієва кислота.

B. Кобаламін.

C. Біотин.

D. Тіамін.

E. *Піридоксин.

24. Генний апарат людини

містить близько 30 тисяч

генів. А кількість

варіантів антитіл сягає

мільйонів. Який механізм

використовується для

утворення нових генів, що

відповідають за синтез

такої кількості антитіл ?

A. Реплікація ДНК.

B. Утворення фрагментів

Оказакі.

Page 221: КРОК-1 БІОХІМІЯ Збірник тестових завданьlib.sumdu.edu.ua/library/docs/rio/2020/m4695.pdf · 2 Крок-1. Біохімія Н: збірник тестових

221

C. Ампліфікація генів.

D. Репарація ДНК.

E. *Рекомбінація генів.

25. У чоловіка 39-ти років

спостерігається підвище-

ний ризик розвитку

інфекційних процесів,

гіперкератоз, порушення

сутінкового зору. Який

вітамінний препарат

необхідно призначити ?

A. Токоферолу ацетат.

B. Рибофлавін.

C. Піридоксину гідрохлорид.

D. Ергокальциферол.

E. *Ретинолу ацетат.

26. 34-річний чоловік

звернувся до стоматолога з

приводу зубного болю.

Після видалення декількох

зубів розпочинається

кровотеча, що триває

більше 15 хв. З анамнезу

відомо, що пацієнт хворіє

на хронічний гепатит С.

Яка причина подовження

часу кровотечі є найбільш

імовірною ?

A. Гіперкальциемія.

B. Зниження вмісту

альбумінів в крові.

C. Тромбоцитопенія.

D. Гіпокальциемія.

E. *Зниження вмісту

фібриногену в крові.

27. Хворий на сімейну

гіперліпідемію, викликану

дефіцитом рецепторв до

ЛПНЩ, вживав інгібітор

-гідроксиметил-глутарил-

КоА-редуктази. Цей

препарат сприяє:

A. Зниженню клітинного

вмісту бета-ГМК-КоА.

B. Підвищенню рівня

триацилгліцеролів крові.

C. Підвищенню активності

клітинної ацилхолестерол-

ацилтрансферази.

D. Підвищенню рівня

сквалена в клітинах

E. *Зниженню рівня

холестеролу крові.

28. Під дією УФ-опромі-

нення та інших факторів

можуть відбуватися зміни

в структурі ДНК.

Репарація молекули ДНК

досягається узгодженою

дією всіх наступних

ферментів, ЗА

ВИНЯТКОМ:

A. Ендонуклеаза.

B. ДНК-полімераза.

C. ДНК-глікозидаза.

D. ДНК-лігаза.

Page 222: КРОК-1 БІОХІМІЯ Збірник тестових завданьlib.sumdu.edu.ua/library/docs/rio/2020/m4695.pdf · 2 Крок-1. Біохімія Н: збірник тестових

222

E. *Аміноацил-тРНК-

синтетаза.

29. У новонародженої

дитини після годування

молоком спостерігаються

диспептичні розлади

(диспепсія, блювання).

При годуванні розчином

глюкози ці явища

зникають. Вкажіть

фермент, що бере участь в

перетравленні вуглеводів,

недостатня активність

якого призводить до

вказаних розладів:

A. Ізомальтаза.

B. Сахараза.

C. Мальтаза.

D. Амілаза.

E. *Лактаза.

30. Жінка літнього віку

перенесла сильний стрес.

У крові різко збільшилась

концентрація адреналіну і

норадреналіну. Які

ферменти каталізують

процес інактивації

катехоламінів ?

A. Карбоксилази.

B. Глікозидази.

C. Пептидази.

D. Тирозиназа.

E. *Моноамінооксидази.

31. Хворому на

туберкульоз призначено

антибіотик олігоміцин.

Назвіть процес, який

інгібується цим

препаратом у дихальному

ланцюзі мітохондрій при

розмноженні туберку-

льозної палички:

A. Ініціація.

B. Трансляція.

C. Реплікація.

D. Трансамінування

E. *Окисне фосфорилю-

вання.

32. Людина вживає

надмірну кількість

вуглеводів. Який

метаболічний процес

активується в організмі

при цьому ?

A. Сечовиноутворення.

B. Синтез нуклеїнових

кислот.

C. Глікогеноліз.

D. Ліполіз.

E. *Ліпогенез.

33. У хворого збільшений

основний обмін, підвищена

температура тіла,

тахікардія у стані спокою.

Причиною цього може

бути підвищена функція:

Page 223: КРОК-1 БІОХІМІЯ Збірник тестових завданьlib.sumdu.edu.ua/library/docs/rio/2020/m4695.pdf · 2 Крок-1. Біохімія Н: збірник тестових

223

A. Підшлункової залози.

B. Нейрогіпофізу.

C. Статевих залоз.

D. Кіркової речовини

наднирників.

E. *Щитоподібної залози.

34. Жінка 45-ти років

звернулася до свого лікаря

зі скаргами на надмірну

втому та слабкість. Вона

повідомила, що симптоми

тривають вже місяць. За

останні 2 тижні втратила 3

кілограми. Об’єктивно:

втомленого вигляду

худорлява жінка. На

багатьох ділянках тіла

виявлено гіперпігментацію

шкіри, найбільш помітну

на обличчі, шиї та тильній

стороні кистей (відкриті до

світла ділянки).

Підвищена продукція

якого з гомноів найбільш

імовірно є причиною

гіперпігментації у жінки ?

A. Гонадотропний гормон.

B. β-ліпотропний гормон.

C. Соматотропний гормон.

D. Тиреотропний гормон.

E.* Меланоцитостимулюю-

чий гормон (МСГ).

35. A 37-year-old man is

admitted to hospital with

mental confusion and

disorientation. His wife

reports he became more

irritable and forgetful in the

past year. In addition, she

notes that he became a vegan

a year ago, and currently,

his diet consists of starchy

foods like potatoes, corn, and

leafy vegetables. GI

symptoms include anorexia,

diarrhea and vomiting. He

has glossitis and skin lesions

that appear as vesicles over

the extremities. Eczema-like

lesions around the mouth, as

well as desquamation and

roughened skin over the

hands are also present.

Neurologic examination

reveals symmetrical

hypesthesia for all types of

sensation in both upper and

lower extremities in a

“gloves and socks”

distribution. Deficiency in

diet of which of the following

amino acids is the most

likely cause of this

condition?

A. Arginine.

B. Threonine.

C. Lysine.

Page 224: КРОК-1 БІОХІМІЯ Збірник тестових завданьlib.sumdu.edu.ua/library/docs/rio/2020/m4695.pdf · 2 Крок-1. Біохімія Н: збірник тестових

224

D. Histidine.

E. *Tryptophan.

36. A mother of a 4-month-

old male infant brought him

to pediatrician with

complaints of food rejection

and weight loss. He started

having trouble latching onto

his bottle. Examination

reveals diminished muscle

tone in all four limbs, and

hepatosplenomegaly. An

opthalmoscopic exam reveals

macular cherry red spots.

During the next few weeks,

hepatosplenomegaly

progresses, the boy fails to

thrive, and he continues to

reject food. ChestX-ray

shows a reticulonodular

pattern and calcified

nodules. Biopsy of the liver

shows foamy histiocytes. A

Niemann-Pick disease is

suspected. Which of the

following is the most likely

deficient enzyme in this

patient ?

A. Galactocerebrosidase.

B. Phenylalanine-hydroxylase.

C. Glucose-6-phosphatese.

D. Glucocerebrosidase.

E. *Sphingomyelinase.

37. On your physiology class,

the professor asks you to

report about the effects of

various body hormones and

neurotransmitters on the

metabolism of glucose. You

begin your report with the

statement that the use of

glucose by the cells is

preceded by absorbtion

through the plasma

membrane from the

extracellular matrix into cell.

Which of the following

hormones is most likely

responsible for the glucose

uptake by the cell ?

A. Thyroxine.

B. Glucagon.

C. Glucose-6-phosphatase.

D. Epinephrine.

E. *Insulin.

Page 225: КРОК-1 БІОХІМІЯ Збірник тестових завданьlib.sumdu.edu.ua/library/docs/rio/2020/m4695.pdf · 2 Крок-1. Біохімія Н: збірник тестових

225

Навчальне видання

Крок-1

Біохімія Збірник тестових завдань

для студентів спеціальностей

222 «Медицина» та 228 «Педіатрія»

денної форми навчання

Відповідальний за випуск Л. Ф. Суходуб

Редактор Н. М. Іншина

Комп’ютерне верстання Н. М. Іншиної

Формат 60×84/16. Ум. друк. арк. 13,02. Обл.-вид. арк. 14,15.

Видавець і виготовлювач

Сумський державний університет,

вул. Римського-Корсакова, 2, м. Суми, 40007

Свідоцтво суб’єкта видавничої справи ДК № 3062 від 17.12.2017.